You are on page 1of 247

Modern Geometry ii

No portion of this learning materials may be copied or

reproduced in books, pamphlets, outlines, or notes- whether

printed, mimeographed, typewritten, photocopied or in any form

– for distribution or sale, without the written permission of the

author.
Modern Geometry iii

Note to the Reader

This endeavour is an attempt to simplify and addressed the impasse

on comprehensible materials on the said field of study. Majority of the

contents of these learning materials were adapted from the book of

George A. Jennings entitled “ Modern Geometry with Applications”. The

topics included in these learning materials were restricted only to

Euclidean Geometry, Spherical Geometry, Conics, Projective Geometry

and Special Relativity. The author acknowledges and would be glad to

hear your feedback about this work text. Comments and suggestions might

be given to further improve this work text. The author understands that

there is always a room for improvement.

The Author
Modern Geometry iv

Overview of the Learning Materials

This learning material has 7 basic parts. These parts include Introduction, Pre-

Competency Checklist, Learning Resources, Explore, Discussion Board,

Post-Competency Checklist and Assignment. The Introduction contains the

short description of the activities for a particular topic as well as the learning

competencies. The Pre-Competency Checklist contains Self-Check that spells

out the Intended Learning Outcomes (ILO) for the specific module. The Learning

Resources identifies the reference materials involved in the particular module. It

may contain books, electronic resources and the likes. The Explore pertains to

the detailed discussion of the topic in the specified module. The Discussion

Board serves as the “Chat room”. Questions about the topic that was discussed

in Explore part were posted in this area. Students would respond to every

question and they may submit their responses either online or offline depending

upon the instruction of the Faculty – In- Charge (FIC). The Post-Competency

Checklist includes Self-Assessment Question that intends to determine

whether the students understood the discussion and gained the expected

competencies. Lastly, the Assignment serves as the reinforcement of the

students learning. It includes items from all of the subtopics included in the entire

module.
Modern Geometry v

TABLE OF CONTENTS

Note to the Reader iii

Overview of the Learning Materials iv

Table of Contents v

Module 1. Euclidean Geometry 1

Euclidean Space 2

Self-Assessment Question 1.1 4

Isometries and Congruence 5

Self-Assessment Question 1.2 7

Reflections in a Plane 8

Self-Assessment Question 1.3 10

Reflections in Space 11

Self-Assessment Question 1.4 14

Translation 15

Self-Assessment Question 1.5 16

Rotations 17

Self-Assessment Question 1.6 19

Applications and Examples of Geometric Concepts 20

Self-Assessment Question 1.7 25

Some Key Concepts of High School Geometry 26

Self-Assessment Question 1.8 34

Some Triangles Congruence Postulates 34


Modern Geometry vi

Self-Assessment Question 1.9 36

General Isometry 37

Self-Assessment Question 1.10 40

Assignment 40

Module 2. Spherical Geometry 44

Geodesics 45

Self-Assessment Question 2.1 50

Geodesics on Spheres 50

Self-Assessment Question 2.2 54

The Six Angles of Spherical Triangles 55

Self-Assessment Question 2.3 62

Law of Cosines for Sides 62

Self-Assessment Question 2.4 64

The Dual Spherical Triangle 65

Self-Assessment Question 2.5 69

Law of Cosines for Angles 70

Self-Assessment Question 2.6 72

Law of Sines for Spherical Triangles 73

Self-Assessment Question 2.7 74

Some Navigation Problems 75

Self-Assessment Question 2.8 79

Mapmaking 80
Modern Geometry vii

Self-Assessment Question 2.9 92

Applications of Stereographic Projection 92

Self-Assessment Question 2.10 99

Assignment 99

Module 3. Conics 101

Conics Sections 102

Self-Assessment Question 3.1 105

Foci of Ellipses and Hyperbolas 106

Self-Assessment Question 3.2 109

Eccentricity and Directrix 109

Self-Assessment Question 3.3 112

Tangent Lines 113

Self-Assessment Question 3.4 118

Properties of Conics 118

Self-Assessment Question 3.5 121

Review Exercises for Standard Equations for Smooth Conics 122

Self-Assessment Question 3.6 124

LORAN Navigation 124

Self-Assessment Question 3.7 126

Kepler’s Law of Planetary Motion 127

Self-Assessment Question 3.8 137

Assignment 138
Modern Geometry viii

Module 4. Projective Geometry 140

Perspective Drawing 141

Self-Assessment Question 4.1 148

Projective Space 149

Self-Assessment Question 4.2 153

Desargues Theorem 154

Self-Assessment Question 4.3 157

Cross Ratios 158

Self-Assessment Question 4.4 166

Projections in Coordinates 167

Self-Assessment Question 4.5 170

Homogenous Coordinates and Duality 171

Self-Assessment Question 4.6 175

Homogenous Polynomials and Algebraic Curves 176

Self-Assessment Question 4.7 179

Tangent 180

Self-Assessment Question 4.8 181

Dual Curves 182

Self-Assessment Question 4.9 185

Pascal’s and Brianchon’s Theorem 186

Self-Assessment Question 4.10 190

Assignment 190
Modern Geometry ix

Module 5. Special Relativity 192

Spacetime 193

Self-Assessment Question 5.1 197

Galilean Transformations 198

Self-Assessment Question 5.2 202

Failure of Galilean Transformations 203

Self-Assessment Question 5.3 204

Lorentz Transformations 205

Self-Assessment Question 5.4 214

Relativistic Addition of Velocities 215

Self-Assessment Question 5.5 216

Lorentz-FitzGerald Contractions 217

Self-Assessment Question 5.6 220

Minkowski Geometry 221

Self-Assessment Question 5.7 226

The Slowest Path is a Line 227

Self-Assessment Question 5.8 231

Hyperbolic Angles and Velocity Addition Formula 232

Self-Assessment Question 5.9 234

Assignment 235

References 237

About the Author 238


Module 1. Euclidean Geometry 1

1
Euclidean Geometry
We are familiar with points, lines and planes during our high school mathematics.
The concept and postulates that we had learned in high school are infallible, that is, what
is true about points, lines and planes are still true at present. In this section, we will try to
recall some of the familiar postulates and theorems related to geometry. But this time, we
will try to use a more concise and axiomatic approach.

SELF-CHECK!!!

After finishing this module, you are expected to achieve the following
objectives
1. Define Euclidean Space.
2. Describe Isometries and Congruence.
3. Illustrate Reflections in a Plane.
4. Describe Reflections in Space.
5. Define Translation in Space.
6. Illustrate Rotation in a Plane and in Space.
7. Solve some Exercises and Applications of Geometric concepts.
8. Recall Some Key Concepts of High School Geometry.
9. Describe Some Triangle Congruence Postulates.
10. Illustrate the General Isometry.
Module 1. Euclidean Geometry 2

LEARNING RESOURCES
For further readings, you may refer to the following book;
George A. Jennings Modern Geometry With Applications
(Universitext)

Or watch video at www.youtube.com/moderngeometry

EXPLORE!!!!
We will now try to build up our understanding about Modern

Geometry. Let us start learning the basic concepts relevant to our

course of study.

EUCLIDEAN SPACE
The ideas about Space that we had learned in high school mathematics

are also true about Euclidean Space. George A. Jennings in his book “Modern

Geometry with Applications” had a concise discussion about Euclidean Space

which we will try to adopt. According to him, Euclidean Space is the space that

contains the ordinary objects of high school geometry, which includes lines,

circles, spheres and so many more. The notation represents the -

dimensional space of real numbers (where stands for real numbers and for

-dimensional). Thus, is a two-dimensional space of real numbers which we

sometimes name each dimension as and , which in turn, results to -plane or

rectangular coordinates ( ), or Cartesian Plane (in honour to Rene Descartes).

The notation represents the -dimensional Euclidean space ( for Euclid and
Module 1. Euclidean Geometry 3

for -dimensional). The notations and are essentially the same. Their

difference is that, comes equipped with special system of coordinates and a

fixed point (origin), while (Euclidean Space) has no natural coordinates or

distinguished points. Therefore, to avoid confusion, we shall refer to geometrical

space as when coordinates was used and when coordinates was not

involved.

Euclidean space is absolutely uniform. Every place in a Euclidean space

looks the same as every other place. This uniformity enables one to move

objects around inside Euclidean space without bending, stretching or distorting.

Distant objects can be compared without changing their shapes by bringing them

together and placing one on top of the other. Reliable measurements can also be

observed in Euclidean space since we can move measuring instruments from

place to place without destroying their accuracy. The most important measuring

tools for Euclidean geometers are ruler (for measuring length/distances), the

protractor (for measuring angles), and sense of orientation or rotational direction

for distinguishing between clockwise and counter clockwise rotations. Objects

are moved from place to place in space by the action of functions. A function

(read as “ is a function from to ” ) takes each point and moves it to

a new position ( ) . If is a set of points, then

( ) * ( ) +

is the set that results from applying the function to all the points in .
Module 1. Euclidean Geometry 4

DISCUSSION BOARD
We will now try to interact with each other in a two way

process at a least possible way! I will post a question/s and try to answer it on

your own.

1. How do we represent -dimensional space of real numbers?

2. Illustrate -dimensional space of real numbers.

3. What about -dimensional Euclidean space?

4. Illustrate -dimensional Euclidean space.

5. What tips could you give to your fellow students who were taking the same

course to make this topic easier to understand?

POST COMPETENCY CHECKLISTS


Let us try to determine if you learned something from our topic. Answer the
following Self-Assessment Question.

SELF – ASSESSMENT QUESTION 1.1


1. In your own words, define Euclidean Space.
2. Explain possible advantages and disadvantages of -dimensional
Euclidean Space over -dimensional Geometrical Space.
3. Identify at least two examples of Euclidean Space and at least two
examples of Geometrical Space.

EXPLORE!!!!
Perhaps, up to this moment you were able to recall some of
Module 1. Euclidean Geometry 5

the concepts discussed in your high school mathematics. It will help you

the most at this point. Essential concepts in mathematics are intertwined with

other concepts, so try to recall them all! We will continue on another concept –

Isometries and Congruence.

ISOMETRIES AND CONGRUENCE

As what our high school teacher taught us, the notation means “the

distance from the point to the point ”. This claim was still true here in our

discussion. There were a lot of functions on Euclidean Space, however, our

focus will be the function that preserve distance.

Definition 1.1 Isometry. A function 𝑓 𝐸𝑛 𝐸𝑛 is an


isometry if, for all points 𝑃 and 𝑄 in 𝐸𝑛 ,
𝑓 (𝑃 )𝑓 (𝑄 ) 𝑃𝑄.

To visually illustrate isometry, we will try to espouse the figure illustrated by

George A. Jennings on his book, as shown below.

Figure 1. Functions on Euclidean Space


Module 1. Euclidean Geometry 6

There are several names for isometries on Euclidean space. Some

authors used the term “Euclidean isometries”, “Euclidean motions” or “Euclidean

transformations” to represent isometries on Euclidean space.

The geometric qualities of figures such as length, area, volume, the size of

angles, and others, can be derived from distances. The length of a polygon is the

sum of the distances between its adjacent vertices and the lengths of more

general curves are computed by approximating them with polygons. Angles can

be measured by the length of arc subtended by the angle on the unit circle. The

area of a rectangle is the product of its length and its width, the volume of a

rectangular box is( ) ( ) ( ), and the areas and volumes of

more general regions can be approximated by filling them up with little rectangles

or boxes.

Isometries preserve lengths, angles, areas and volumes. In short,

isometries preserve the size and shape of every geometric figure.

Example 1.1 The line segment ̅̅̅̅ is the shortest curve connecting to . If is

an isometry then it follows that (̅̅̅̅ ) must be the shortest curve connecting ( )

to ( ), since isometries preserve lengths. Therefore (̅̅̅̅ ) also is a line

segment

(̅̅̅̅ ) ̅̅̅̅̅̅̅̅̅̅̅̅̅
( ) ( ).

Definition 1.2 Congruence. Subsets 𝐴 𝐵 ⊆ 𝐸𝑛 are


congruent (in symbols 𝐴 𝐵) if there is an isometry 𝑓 such
that 𝑓 (𝐴) 𝐵.
𝑓 (𝑃 )𝑓 (𝑄 ) 𝑃𝑄.
Module 1. Euclidean Geometry 7

The idea of congruence that we learned in number theory and set theory also

holds true in modern geometry.

DISCUSSION BOARD
We will now try to interact with each other in a two way

process at a least possible way! I will post a question/s and try to answer it on

your own.

1. What are other names for isometries?

2. How do we describe isometry?

3. What tips could you give to your fellow students who were taking the same

course to make this topic easier to understand?

POST COMPETENCY CHECKLISTS


Let us try to determine if you learned something from our topic. Answer the
following Self-Assessment Question.

SELF – ASSESSMENT QUESTION 1.2


1. Show that every isometry is a one – to – one function. (In other words,
show that if is an isometry and and are points with then
( ) ( ))
2. Assume that is an isometry and that it has an inverse function .
Show that is also an isometry.
3. Let be the circle with center and radius . Prove that if
is an isometry then ( ) is the circle with center ( ) and
radius . (Hint: prove that if is the circle with the center ( ) and
radius then ( ) ⊆ and ( ) ⊆ . You may assume that has
an inverse and that is an isometry.
Module 1. Euclidean Geometry 8

EXPLORE!!!!
Your knowledge about logic and set theory will be helpful in

establishing proof on exercises in this module. So, as a piece of advice, try to

read your notes again! Meanwhile, we will now move forward to another concept

in Euclidean Geometry.

REFLECTIONS IN A PLANE

We have to bear in mind that all Euclidean isometry is a combination of

three fundamental types: reflections, translations and rotations. We will deal with

these fundamental types of isometry in detailed. For the meantime, let us

consider the discussion and illustration of Jennings (1997).

Let be a point and a line in . Drop a perpendicular from to .

The reflection of in is the point such that lies on the opposite side

of from and is the same distance from as . ( if )

Figure 2. Reflection in a Line

We still have three other ways to reflect a figure which will lead to the same

result.
Module 1. Euclidean Geometry 9

1. Flip the plane over onto its back by rotating it around in three

dimensional space. Equivalently, fold the plane over along the line , then

trace the figure on the other side. This illustration will explain why

reflections are isometries, since it is obvious that merely rotating the plane

does not change the size or shape of any figure in it.

Figure 3. Reflecting by Flipping Over

2. A “ruler and compass construction”. If , draw two circles with their

centers on , intersecting at . The two circles will intersect at another

point on the opposite side of ; this point is the reflection of .

Figure 4. Ruler and Compass Construction

3. Reflection in the -axis in is given by the formula

( ) ( )

DISCUSSION BOARD
We will now try to interact with each other in a two way
Module 1. Euclidean Geometry 10

process at a least possible way! I will post a question/s and try to answer it on

your own.

1. What happens when we reflect an object in a plane?

2. Create your own illustration of a reflection of an object in a plane.

3. What advice could you give to your fellow students who were taking the

same course to make this topic easier to understand?

POST COMPETENCY CHECKLISTS


Let us try to determine if you learned something from our topic. Answer the
following Self-Assessment Question.

SELF – ASSESSMENT QUESTION 1.3


1. Illustrate the reflection of a curve in the -axis.
2. Find a formula in terms of , and for reflection in an arbitrary line
in .
3. Prove that any two circles in the plane intersect in exactly two, one or
zero points, depending on the distance between their centers and the
sizes of their radii. (Hint: Write equations for the circles and solve them
simultaneously. This is not hard to do if you set up coordinates so that
the -axis passes through the centers of both circles).

EXPLORE!!!!

I hope that you are doing better at this moment. As we

continue, you will notice that your knowledge in high school geometry will be of

great help. We proceed with Reflections in Space.


Module 1. Euclidean Geometry 11

REFLECTIONS IN SPACE

The familiar geometric space coordinates, , will help us facilitate

understanding of reflections in space in an easy way. Let us try to analyse

reflection in space with the aid of illustrations for visualization.

Let be a point and a plane in . Drop a perpendicular from to .

The reflection of in is the point such that lies on the opposite side

of from and is the same distance from as . ( if ).

Figure 5. Reflection in a Plane

Reflection in the -plane in is given by the formula

( ) ( )

When we deal with space, it is inevitable to talk about vectors. So in this section,

we will try to include vectors. And, I advise you to read your notes on Linear

Algebra for your reference.

Vectors

A vector is a directed line segment that points from one point to another in

or . Vectors are equivalent if they have the same length and point in the

same direction. It is advisable that we should use notation for vectors. We will

use the notation ( ) to denote the point ( ) and the


Module 1. Euclidean Geometry 12

Figure 6. Vectors and Points in

vector that points from the origin to the point ( ) in .We add vectors

and subtract vectors in a usual way, by placing the vectors end to end. Vectors

can be added to points. If and are points in and ⃗⃗⃗⃗ is a vector that points

from to , then

⃗⃗⃗ , ⃗⃗⃗ and ⃗⃗⃗ ⃗⃗⃗⃗⃗

Figure 7. Adding Vectors to Points and Vectors

Since we are talking about reflection, we should include orientation as well. This

enables us to have the same understanding of figures and illustrations.

Orientation in the Plane

When we deal with plane, reflection produce mirror images of objects, with

the line in which one is reflecting serving as the mirror. Reflected writings has the
Module 1. Euclidean Geometry 13

same size and shape as the original, but it is still hard to read because its

orientation has been changed. In plane, orientation refers to a choice of direction

of rotation- clockwise or counter clockwise in the plane. Reflections reverse

orientation by changing clockwise rotations to counter clockwise rotations and

vice-versa.

Figure 8. Reflections Reverse Orientation

Orientation in Space

As we observe in a plane, the orientation was reversed by the reflection.

However, in space, there is a difference. Orientation in is determined by using

the “right hand rule”

Figure 9. Right-hand Rule

Let (⃗⃗⃗ ⃗⃗⃗ ⃗⃗⃗ ) be an ordered triple of nonzero vectors in , not all parallel to the

same plane. (⃗⃗⃗ ⃗⃗⃗ ⃗⃗⃗ ) is positively oriented if, when you point the thumb of your

right hand in the ⃗⃗⃗ direction and your first finger in the ⃗⃗⃗ direction, then the rest

of the fingers of your right hand curl toward ⃗⃗⃗ direction. (⃗⃗⃗ ⃗⃗⃗ ⃗⃗⃗ ) is negatively
Module 1. Euclidean Geometry 14

oriented if the rest of your fingers point in the ⃗⃗⃗ direction. Reflections in space

reverse orientations by mapping positively oriented triples of vectors to negatively

oriented triples and vice-versa.

DISCUSSION BOARD
We will now try to interact with each other in a two way
process at a least possible way! I will post a question/s
and try to answer it on your own.
1. How do we illustrate reflection in space?

2. How does vectors relate to reflection in space?

3. Does right-hand rule helps us understand the concepts better?

4. What advice could you give to your fellow students who were taking the

same course to make this topic easier to understand?

POST COMPETENCY CHECKLISTS


Let us try to determine if you learned something from our topic. Answer the
following Self-Assessment Question.

SELF – ASSESSMENT QUESTION 1.4


1. In your own words, describe Reflections in Space.
2. Show that every reflection (in or ) is its own inverse: if
is a reflection ( ) then
3. Illustrate the reflection of a square in a plane
4. Draw the reflection of a globe about -plane in
Module 1. Euclidean Geometry 15

EXPLORE!!!!

I am confident that you are all right! Personally, I find it hard to

draw objects, but I know that you are much better than me! We will now continue

on another interesting concept in Euclidean geometry-Translation.

TRANSLATION

When we move objects in space without changing its qualities we had

created another concept in Euclidean geometry- the translation. Translations

moves objects along a straight line through space without rotating the objects or

changing their orientation. There is a one – to – one correspondence between

vectors in and translations: the translation associated to a vector ⃗⃗⃗

acts by adding ⃗⃗⃗ to each point,

( ) ⃗⃗⃗

⃗⃗⃗ is the displacement vector for the translation .

Figure 10. A Translation

In coordinates, if ⃗⃗⃗ ( ), then

( ) ( )
Module 1. Euclidean Geometry 16

DISCUSSION BOARD
We will now try to interact with each other in a two way
process at a least possible way! I will post a question/s
and try to answer it on your own.

1. How do we illustrate translation?

2. Can you name some of the applications of translation in our lives?

3. What possible study guide could you give to your fellow students who

were taking the same course to make this topic easier to understand?

POST COMPETENCY CHECKLISTS


Let us try to determine if you learned something from our topic. Answer the
following Self-Assessment Question.

SELF – ASSESSMENT QUESTION 1.5


1. In your own words, define Translation in Space.
2. Let be a square whose vertices are ( ), ( ), ( ) and ( ) and
let be the translation associated to the vector ⃗⃗⃗ ( ). Find ( ).
3. Let ( ) be a point and ⃗⃗⃗ ( ) and ⃗⃗⃗ ( ) be vectors
in .
a. If the vector ⃗⃗⃗ is associated with translation , find ( ).
b. If the vector ⃗⃗⃗ is associated with translation , find ( ).

EXPLORE!!!!

Perhaps, you deserve a break! You may take a cup of tea or


Module 1. Euclidean Geometry 17

coffee before we continue on another concept in Euclidean geometry. A 5-minute

break would enable your neurons to take a little relaxation. After the break we will

continue on rotations. Okay, time is up! As a piece of advice, you should read

your notes in trigonometry, because you will need those concepts you learned, in

this section.

ROTATIONS

The formulas for trigonometric functions, sum and difference of angles,

twice angles and half-angles would be used in rotations. We will start with

rotations in plane.

Rotations in Plane

We can perform rotation in the plane by revolving the plane around a

given point (the center of rotation) through a given angle. We need a point and

angle to describe rotation in a plane in . We have to take note that counter

clockwise rotations sweep out positive angles, and clockwise rotations sweep out

negative angles.

Figure 11. A Rotation


Module 1. Euclidean Geometry 18

For us to be guided, we will use the following notation in this particular topic;

( )

Given the polar coordinates for ( ),

The formula for rotation in the - plane through an angle about the origin is

given by,

( ) ( )

I will not include the derivation of the formula in this section. I will challenge you

to derive it on your own using your previous knowledge in trigonometry.

The inverse of a rotation is a rotation with the same center and the

opposite angle,

( )

You have to note that the formula for rotation in plane was already given and it

was not so complicated to comprehend. In the next section, I will ask you to

compare by yourself, the two rotations – rotation in plane and rotation in space,

for you to determine if there is a difference or similarities between them.

Rotations in Space

We will use the formula to denote the rotation in three – dimensional

space. In three – dimensional space, represents an angle and for axis. is a

directed line – a line with sense of direction. The direction of the axis determines

the direction of positive rotation by using the right – hand rule (refer to Figure 9).
Module 1. Euclidean Geometry 19

DISCUSSION BOARD
We will now try to interact with each other in a two way
process at a least possible way! I will post a question/s
and try to answer it on your own.

1. How do we describe rotation in plane? In space?

2. What skills do we need to have to help us learn this topic easier?

3. Can you name some advice for your fellow students to understand this

topic at ease?

POST COMPETENCY CHECKLISTS


Let us try to determine if you learned something from our topic. Answer the
following Self-Assessment Question.

SELF – ASSESSMENT QUESTION 1.6


1. Illustrate Rotation in Plane and in Space. Create your own examples.
2. Find a formula for rotation through an angle around an arbitrary
point ( )
3. Let , be two lines intersecting at a point in . Let be the
reflection in and let be the reflection in . Show that the
composition is a rotation around , and the angle of rotation is
equal to twice the angle formed by the two lines.

EXPLORE!!!!

We had examined several geometric concepts in the previous


Module 1. Euclidean Geometry 20

sections, and we will now try to explore some applications and examples of

geometric concepts. In this section, we will try to integrate geometric concepts in

some real life situations.

APPLICATIONS AND EXAMPLES OF GEOMETRIC CONCEPTS

In this portion, we will try to expound the concepts in geometry that we

had discussed.

Example 1.2 A boy intends to collect some water from a stream and then carry it

home. He wants to find the shortest path that takes him to the stream and then to

his house. What route should he take?

Figure 12. Shortest Path

Obviously, he should go straight to some point on the bank of the stream, fill his

bucket, and then go straight home. To which point on the stream should he go?

Solution. Assume the boy is at , his home is at , and the bank of the

stream forms line . Assume further that and are on the same side of

(since the solution is obvious if they are on opposite sides).

Claim. Let be the reflection of in and let


Module 1. Euclidean Geometry 21

̅̅̅̅̅

Then is the shortest path.

Proof. If is any other point on ,then . Since reflection

preserves distance, . Thus,

Likewise, , so

Clearly,

because , and lie in a straight line while , and do not. Hence,

which proves the claim.

Example 1.3 Two mirrors meet in a angle. Someone stands between the

mirrors with her eye at point searching for images of an object that is located at

Figure 13. Multiple Reflections


Module 1. Euclidean Geometry 22

point . How many images of the object will she see? In what direction and how

far away will they appear to be?

For simplicity’s sake assume that and lie in a plane that is

perpendicular to the line where the two mirrors intersect. This enables us to treat

the problem as a problem in plane geometry: because is perpendicular to both

mirrors a light ray reflecting from to will remain in for its entire trip.

The figure shows five images of and their apparent locations as they

appear from an observer at . itself is one of the “images” since observer could

look directly at . Another image is formed by light that goes from to the first

mirror, , reflects, and then goes to . This image appears to be located at the

reflection of in because the reflected light comes from the same direction

as and it travels a distance equal to . Similarly, light travelling along the

path forms an image that appears to be located at , the reflection of in

The image is formed as follows. is the reflection of the mirror

in . is the reflection of in . ̅̅̅̅̅̅̅ intersects at and at .

Reflecting ̅̅̅̅̅̅̅̅ in gets ̅̅̅̅̅̅̅, then reflecting the path in one

obtains the path . Thus light reflects from to along the path , and

the image appears to be located at .

Similarly, light reflects along the path producing an image at .

is the reflection of in , while is the reflection in . ̅̅̅̅̅̅̅

, ̅̅̅̅̅̅̅ , ̅̅̅̅̅̅̅ reflects to ̅̅̅̅̅ in , and reflects to in .


Module 1. Euclidean Geometry 23

One can construct all these reflections by folding the plane over. For

instance the path results from folding ̅̅̅̅̅̅̅ once over and then over

. Or one could think of ̅̅̅̅̅̅̅ as the result of unfolding , once over

and then over .

This folding construction makes it easy to see why no reflections are

possible in this example except the five that were constructed above. When one

unfolds a path, the points where the light reflects become points where the

unfolded line crosses a mirror. As unfolding progresses the successive images of

all lie in the same side of the line that connects to the point where the two

mirrors intersect. No more images are possible because it is impossible to

produce any more images of without passing from one side of this line to the

other.

The number of reflections, and hence the number of images seen by the

observer, varies depending on the angle between the two mirrors, the position of

the viewer and the position of the object relative to the mirrors.

Example 1. 4 A road is proposed that will connect two towns and on

Figure 14. Bridge Between Two Towns


Module 1. Euclidean Geometry 24

opposite sides of river. The road will cross the river in a bridge that is

perpendicular to the riverbanks. Where the bridge should be placed so as to

minimize the length of the road?

Solution. Let ⃗⃗⃗ be the vector that points from the riverbank on the side of the

river to the bank on the side, perpendicular to the river. Set ⃗⃗⃗ . The

bridge should be built at the point where ̅̅̅̅̅ crosses the riverbank on the

side of the river.

Proof. We must show that if one puts the bridge anywhere else he will get a

longer road. A bridge at extends from to the point ⃗⃗⃗ on the

opposite riverbank. With the bridge at , the length of the road is

|⃗⃗⃗ | , where |⃗⃗⃗ | is the length of vector ⃗⃗⃗ .

Let be any other point on the side of the river and let ⃗⃗⃗ be the

corresponding point on the opposite riverbank. If the bridge were built at then

the roads length will be

|⃗⃗⃗ | .

Translation by ⃗⃗⃗ maps ̅̅̅̅ to ̅̅̅̅̅̅ and ̅̅̅̅ to ̅̅̅̅̅̅. Thus

|⃗⃗⃗ | |⃗⃗⃗ |

And

|⃗⃗⃗ | |⃗⃗⃗ | .

Clearly,
Module 1. Euclidean Geometry 25

since points , and lie on a line. Hence |⃗⃗⃗ | |⃗⃗⃗ |

, so is the shortest road.

DISCUSSION BOARD
We will now try to interact with each other in a two way
process at a least possible way! I will post a question/s
and try to answer it on your own.

1. Aside from the examples given, can you mention at least two of the

Geometric concepts and their corresponding applications?

2. What possible tips could you give to your fellow students who were taking

the same course to make this topic easier to learn?

POST COMPETENCY CHECKLISTS


Let us try to determine if you learned something from our topic. Answer the
following Self-Assessment Question.

SELF – ASSESSMENT QUESTION 1.7


1. Find all possible paths that light could take in Example 1.3 if the angle

between the mirrors were

a. 900

b. 600

c. 500
Module 1. Euclidean Geometry 26

2. Find a formula for the number of paths from to as a function of the

angle between the mirrors and the positions of the points and in

Figure 13.

3. What modifications should be made in the discussion of Example 1.3 if

and do not lie in a plane that is perpendicular to the intersection of the

two mirrors?

EXPLORE!!!!

I hope that you are still doing well this time! Well, we may

sometimes find it hard to connect our previous knowledge with the new ideas! I

had also experience the same scenario especially in Geometry. For now, we will

continue exploring Some Key Concepts of High School Geometry.

I do not own all the knowledge in this universe, so I had to ask for help

from other individual who were more knowledgeable in this field. I would like to

mention the discussion of George A. Jennings in this section, from his book

“Modern Geometry with Applications”.

Two lines in are parallel if and only if they are equal or they lie in

common plane and do not intersect. We will now state the controversial parallel

postulate.

THE PARALLEL POSTULATE: Given a line and a point in there is

exactly one line through that is parallel to .


Module 1. Euclidean Geometry 27

If is a translation that takes some point on to , then ( ) is parallel to

Figure 15. Parallel Postulate

The parallel postulate was considered to be the most controversial of Euclid’s

postulates for Geometry. A lot of scholars felt that it should be possible to deduce

the parallel postulate from Euclid’s other postulates. However, it was later proved

to be impossible to deduce the parallel postulate from the other postulates, and

efforts to do so led to the invention of various “non- Euclidean geometries” in

which the parallel postulate was debased. Aside from being interesting to

mathematicians, some non-Euclidean goemetries have practical application, like

the famous being in Einstein’s General Theory of Relativity.

ALTERNATE INTERIOR - EXTERIOR ANGLES

As previously discussed, two figures and are congruent (in

symbols ) if there is an isometry such that ( ) .

Figure 16. Alternate Interior – Exterior Angles


Module 1. Euclidean Geometry 28

Alternate interior – exterior angles and are formed when parallel lines

intersect a transversal. Alternate interior – exterior angles are congruent since a

translation maps one of them to the other. Conversely, if then the two

lines are parallel.

VERTICAL ANGLES

When two lines intersect, it forms pairs of vertical angles.

Figure 17. Vertical Angles

Vertical angles are congruent are congruent since a 1800 rotation about the point

of intersection maps one angle to the other.

THEOREM 1. The sum of the angles of a Euclidean triangle is 1800.

Proof. Given , draw a line through parallel to ̅̅̅̅ . Label angles as in

Figure 18. and are vertical angles so , while and

because they are alternate interior – exterior angles. Clearly, +

+ = .

Figure 18. Sum of Angles = 1800


Module 1. Euclidean Geometry 29

Theorem 1 depends on parallel postulate – it is not true in non-Euclidean

geometries where the space is curved. The difference between the sum of the

angles of a triangle and 1800 is a measure of the curvature of space; if the

difference is not zero the space is curved rather than flat. Our own physical

universe is a curved space: if you build a very large triangle by joining three

vertices together with curves of minimal length then the angles of the triangle

generally will not add up to 1800. The General Theory of Relativity explains that

this bending of space is a manifestation of gravitation.

Majority of key concepts in high school geometry derived from elementary

facts about area. Mostly, the area inside a figure can be computed by cutting the

figure up and rearranging the pieces until one obtains a figure of known area. For

example, from the fact that area of rectangle is the length of its base times its

height, one finds that:

Figure 19. Area of Parallelogram

( )

Figure 20. Area of Triangle


Module 1. Euclidean Geometry 30

( )

Figure 21. Area of a Circle

PROPOSITION 1. If is parallel to ̅̅̅̅ , and , , then and

have the same area.

Proof. Both triangles have the same base and height.

Figure 22. The Triangles Have Equal Area

Definition 1.3 Similar Triangles. 𝐴𝐵𝐶 and 𝐴 𝐵 𝐶 are


similar if 𝐴 𝐴, 𝐵 𝐵 and 𝐶 𝐶

THEOREM 2. Similar Triangles are Proportional.

If and are similar then


Module 1. Euclidean Geometry 31

Proof. Without loss of generality, we may assume that . Since

there exists an isometry such that (⃗⃗⃗⃗⃗⃗⃗⃗ ) ⃗⃗⃗⃗⃗ and (⃗⃗⃗⃗⃗⃗⃗⃗ ) ⃗⃗⃗⃗⃗ .

By applying we may arrange the triangles as in Figure 23, where ,

̅̅̅̅ , and ̅̅̅̅

Figure 23.

Regard and as triangles with their bases on ⃡⃗⃗⃗⃗ and their heights

equal to the distance from ⃡⃗⃗⃗⃗ to . The formula for the area of a triangle says

that,

( )
( )

Similarly,

( )
( )

Now ⃡⃗⃗⃗⃗⃗⃗⃗ is parallel to ⃡⃗⃗⃗⃗ since , so the distance from to the line ⃡⃗⃗⃗⃗⃗⃗⃗

equals the distance from to ⃡⃗⃗⃗⃗⃗⃗⃗ . Hence,

( ) ( )

But,

( ) ( ) ( )

And

( ) ( ) ( ).
Module 1. Euclidean Geometry 32

So,

( ) ( )

Simplifying the results,

THE PYTHAGOREN THEOREM. The following generalization of Pythagorean

Theorem was proved by Pappus of Alexandria in the 4th century A.D.

THEOREM 3. Let be an arbitrary triangle in , not necessarily a right

triangle. Erect parallelograms ABDE and ACFG on the outside of so that

ABDE meets along the edge ̅̅̅̅ and ACFG meets along the edge

̅̅̅̅ . Let be the point where the lines ⃡⃗⃗⃗⃗ and ⃡⃗⃗⃗⃗ meet. Erect a third

parallelogram BCHI on the outside of so that the vectors

⃗⃗⃗⃗⃗ ⃗⃗⃗⃗

are equal. Then,

( ) ( ) ( )

Figure 24. The Generalized Pythagorean Theorem

Proof. Let ⃡⃗⃗⃗⃗ ⃡⃗⃗⃗ , ⃡⃗⃗⃗⃗ ⃡⃗⃗⃗⃗ , ⃡⃗⃗⃗⃗ ⃡⃗⃗⃗⃗ and ⃡⃗⃗⃗⃗ ⃡⃗⃗⃗ . ̅̅̅̅ , ̅̅̅̅, ̅̅̅̅

and ̅̅̅ are parallel. Two parallelograms with the same base and height have the
Module 1. Euclidean Geometry 33

same area, so

( ) ( ) ( ),

and

( ) ( ) ( )

Clearly,

( ) ( ) ( )

so the proof is complete.

DISCUSSION BOARD
We will now try to interact with each other in a two way
process at a least possible way! I will post a question/s
and try to answer it on your own.

1. Can you reiterate some of the key concepts in high school geometry

previously mentioned?

2. Aside from the concepts mentioned, can you name at least two key

concepts in high school geometry?

3. What possible tips could you give to your fellow students who were taking

the same course to make this topic easier to learn/recall?

POST COMPETENCY CHECKLISTS


Let us try to determine if you learned something from our topic. Answer the
following Self-Assessment Question.
Module 1. Euclidean Geometry 34

SELF – ASSESSMENT QUESTION 1.8


1. Deduce the Pythagorean Theorem from Theorem 3, if , then

( ) +( ) =( )

2. Make a scale drawing and use it to find the height of any object near your

place of residence.

3. From two points and on their side of a river, a group of sightseers can

see a dock on the opposite bank. Using a protractor and a tape

measure, they find that , and . Make

a scale drawing and use it to find the width of the river.

EXPLORE!!!!

I hope that you are still alright at this moment! Meanwhile, let

us try to continue on Some Triangles Congruence Postulates. It has been

observed that much of school geometry was taken up with the study of

elementary relations (“angle – side – angle”, “side – side – side”, etc.) between

congruent triangles. This relation as well as the rest of Euclidean geometry

follows from properties of isometries.

PROPOSITION 2. Side – Angle – Side (SAS)


If , , and , then .
Module 1. Euclidean Geometry 35

Figure 25. Side – Angle - Side

Proof. Since is congruent to there is an isometry such that ( )

. By composing it with a reflection if necessary we may assume that takes

⃗⃗⃗⃗⃗ to ⃗⃗⃗⃗⃗⃗⃗⃗ and ⃗⃗⃗⃗⃗ to ⃗⃗⃗⃗⃗⃗⃗⃗ . Because and it follows that

( ) and ( ) , since isometries preserve distances. Therefore maps

to .

PROPOSITION 3. Angle – Side – Angle (ASA)

If , , and , then .

Proof. . By law of cosines,

( )( ) ( )( )

Thus, by SAS the triangles are congruent.

PROPOSITION 4. Side – Side – Side (SSS)

If , and then .

Proof. By law of cosines,

( ) ( ) ( )
( )( )

( ) ( ) ( )
( )( )

Hence, , so by SAS the triangles are congruent.


Module 1. Euclidean Geometry 36

DISCUSSION BOARD
We will now try to interact with each other in a two way
process at a least possible way! I will post a question/s
and try to answer it on your own.

1. Can you reiterate some of the triangle congruence postulates previously

mentioned?

2. Aside from the three triangle congruence postulate discussed previously,

can you discuss at least two other triangle congruence postulate?

3. What possible tips could you give to your fellow students who were taking

the same course to make this topic easier to learn/recall?

POST COMPETENCY CHECKLISTS


Let us try to determine if you learned something from our topic. Answer the
following Self-Assessment Question.

SELF – ASSESSMENT QUESTION 1.9


1. The navigator of a ship saw landmarks in the distance at three points ,

and . Taking sightings from the deck of the ship she found that

, , and . Then she located the

points , and on a map and find the exact location of her ship. How

did she do it?

2. Prove that if ⃡⃗⃗⃗⃗ and ⃡⃗⃗⃗⃗ are tangent to a circle at distinct points and

respectively, then and the bisector of passes through the


Module 1. Euclidean Geometry 37

center of the circle.

3. State and prove a corresponding result for the lines tangent to a sphere.

EXPLORE!!!!

So far, we had explored some of the concepts relevant to

Euclidean geometry. We are now about to end our journey in Euclidean

geometry, however it points out that we will again start a new journey towards

examining our course of study- the Modern Geometry.

At this point in time, we will try to analyse General Isometry. Again, I would

like to reiterate, I do not own all the knowledge in this universe, so I have to ask

for help from more knowledgeable person. I would like to emphasize the

discussion of George A. Jennings in this section.

THEOREM 4. Every isometry is a composition of rotations,

translations, and reflections.

Sketch of Proof. We will try to sketch the proof for the case , the general

case may be treated in much the same way.

We set up a system of coordinates on in the usual way. Choose a pair

of perpendicular lines for the coordinate axes, label the four quadrants, and then

assign coordinates ( ) to each point in by measuring the distance ( )

from the point to one of the axes (the -axis) and the distance ( ) from the

point to the other axis (the -axis). The plus or minus sign depend on which

quadrant it is that contains the point.


Module 1. Euclidean Geometry 38

The positive end of the -axis is the set of points with positive

coordinates on the -axis. The positive end of the -axis is defined in similar way.

Let

the - axis

the -axis.

An isometry maps and to another pair of perpendicular lines. Let

( )

( ).

We shall regard and as a new pair of coordinate axes, labelling the

quadrants so that maps the -th quadrant in the , coordinate system onto

the -th quadrant in the , coordinate system for each .

Since isometries preserve distance, the distance from to any point

equals the distance from to ( ). Similarly, the distance to equals

the distance from to ( ). It follows that if a given point has

coordinates ( ) in the , coordinate system then the point ( ) has the same

coordinates ( ) in the , coordinate system.

Consequently, one can compute ( ) for every point as soon as

one knows what does on the coordinate axes. In particular, two isometries are

the same if they both have the same effect on the coordinate axes.

Figure 26
Module 1. Euclidean Geometry 39

Let be the translation such that

( ) ( ).

maps to a line through ( ). Choose a rotation around ( ) that maps

( ) to . There are two such rotations, each differing by ; let be the one

that takes points on the positive end of to points on the positive end of . The

composition maps to . Since ( )( )) preserves angles, it must also

map to .

If maps points on the positive end of to points on the positive end

of then has the same effect on and as does, which proves that

. Otherwise, maps points on the positive end of to points on the

negative end of . In this case let be reflection in . ( ) maps the

positive end of to the positive end of and the positive end of to the positive

end of , so . In either case is a composition of translations,

rotations, and reflections, which completes the proof.

nce, , so by SAS the triangles are congruent.

DISCUSSION BOARD
We will now try to interact with each other in a two way
process at a least possible way! I will post a question/s
and try to answer it on your own.

1. Can you discuss General Isometry in your own way?

2. Illustrate/draw concrete objects that exhibit General Isometry.


Module 1. Euclidean Geometry 40

3. What possible tips could you give to your fellow students who were taking

the same course to make this topic easier to learn?

POST COMPETENCY CHECKLISTS


Let us try to determine if you learned something from our topic. Answer the
following Self-Assessment Question.

SELF – ASSESSMENT QUESTION 1.10


1. Show that if is an orientation – preserving isometry then is a

translation or a rotation.

2. Show that every isometry on or has an inverse.

ASSIGNMENT

1. The best seat in a certain theatre is the seat marked “A” in Figure 27. Find

all other points from which the stage subtends the same angle as it does

for the viewer in seat A.

Figure 27

2. A line is tangent to a circle if and only if it intersects the circle at exactly

one point. Let be a circle with center , , and a line through .

Prove that is tangent to if and only if is perpendicular to ̅̅̅̅.


Module 1. Euclidean Geometry 41

3. A belt wraps around two pulleys, which are mounted with their centers 4

feet apart. If the radius of one pulley is 1 ft. and the radius of the other is 2

ft., how long is the belt? (Assume the belt has zero thickness and does not

cross itself)

Figure 28

4. Prove that the bisectors of the angles of a triangle all meet at the same

point , and there is a circle centered at that is tangent to all three sides

of the triangle.

Figure 29. The Inscribed Circle

5. What is the largest sphere that will pass through a triangular hole with

sides 7 in., 8 in., and 9 in. long? (refer to Figure 29 for visualization)

6. A gardener cut a piece of sod to fill a triangular hole. When he tried to put

it in the hole he found out that it fit perfectly, but only with the wrong side
Module 1. Euclidean Geometry 42

up. How can he cut the triangle into three pieces so that the shape of each

piece is not changed when he turns it over?

7. A globe is supported by a triangular stand set on a table. Pads are placed

on the sides of the stand to prevent the globe from getting scratched. The

stand is 3 in. tall; its sides forms a - - triangle whose shortest

side is 1 ft long. Exactly where on the triangle should each pad be placed?

If the globe is 2 ft. in diameter, how high is the top of the globe above the

table? (Neglect the thickness of the pads)

Figure 30. Globe on a Stand

8. Prove that the perpendicular bisectors of the sides of any triangle all meet

at a single point , and there is a circle centered at that passes through

all three vertices of the triangle.

Figure 31. The Circumscribed Circle

9. A thin iron triangle with angles , and is accidentally dropped

into a hemispherical tank. The tank is 10 feet deep and filled with water. It
Module 1. Euclidean Geometry 43

is proposed to fish the triangle out the tank by lowering a powerful magnet

into the tank with a rope, allowing the magnet to attach itself to the

triangle, and then pulling it up. To enable the magnet to reach the triangle

the rope must be long enough to reach the triangle from the surface of the

water.

What is the minimum length of the rope that is required to ensure

that one can reach the triangle if the shortest side of the triangle is 10 feet

long?

Figure 32. Retrieving a Triangle

10. Given a triangle with sides of length , , opposite vertices , ,

respectively, prove that

Where is the diameter of the circle that circumscribes the triangle.


Module 2. Spherical Geometry 44

2
Spherical Geometry
We are familiar with circle, cone, cylinder and sphere during our high school
mathematics. The concepts, theorems and postulates that we had learned in high school
about these figures are dependable, that is, what is true about these figures are still true at
present. In this section, we will try to explore one of the unfamiliar topics related to
modern geometry- Spherical Geometry. Take note that we will be using a more concise
and axiomatic approach.

SELF-CHECK!!!

After finishing this module, you are expected to achieve the following
objectives
1. Define Geodesics.
2. Describe Geodesics on Spheres.
3. Illustrate the Six Angles of a Spherical Triangle.
4. Solve for Law of Cosines for Sides.
5. Define the Dual Spherical Triangle.
6. Solve for the Law of Cosines for Angles.
7. Illustrate the Law of Sines for Spherical triangle.
8. Solve some Navigation Problems
9. Describe Mapmaking.
10. Solve some exercises related to Applications of Stereographic Projection.
Module 2. Spherical Geometry 45

LEARNING RESOURCES
For further readings, you may refer to the following book;
George A. Jennings Modern Geometry With Applications
(Universitext)

Or watch video at www.youtube.com/moderngeometry

EXPLORE!!!!
We will now try to continue enhancing our understanding

about Modern Geometry. We had started with Euclidean Geometry, and we will

continue on another topic relevant to our course of study – Spherical Geometry.

As I previously mentioned in Module 1, majority of my discussion here was lifted

from the work of George A. Jennings in his book Modern Geometry with

Applications. The starting point of our learning journey would be – Geodesics.

GEODESICS

As mentioned by Jennings, in any geometrical setting where it makes

sense to talk about the distance between points, the most important curves are

the geodesics.

Definition 2.1 Geodesics. The shortest curve connecting


two points in a space is a geodesic in that space.
Module 2. Spherical Geometry 46

Example 2.1. A geodesic connecting two points on the globe can be found by

stretching a piece of string across the globe between the points and pulling it

tight. The geodesic connecting Los Angeles to London passes northeast through

central Canada, turns east across the southern tip of Greenland, and arrives in

London heading southeast. Ships and airliners save fuels by following such

“great circle routes” when travelling long distances.

It should be noted that one looks only at curves that lie entirely in the

space when searching for geodesics in space . The fact that there may be

shorter curves outside of is irrelevant- we should treat as if it were the entire

universe. For instance one could find a shorter path from Los Angeles to London

than the one in Example 2.1 by burrowing through the earth, but that does not

matter since such path would take one out of the “universe” which, in this case, is

the surface of the globe.

There may be more than one geodesic connecting a given pair of points.

For instance, there are infinitely many geodesics connecting the north and south

poles on the globe.

Figure 1. Geodesic connecting Los Angeles to London


Module 2. Spherical Geometry 47

We can connect shorter geodesics together to get longer ones in the same way

that we join line segments to get a line. In general, we say that a curve is a

“geodesic” if every sufficiently small segment of the curves has minimum length

in the sense of Definition 2.1. Such geodesics minimize length over long

distances.

Example 2.2.If you roll up a piece of paper into a cylinder, connect two points on

the cylinder with a piece of string, and pull the string tight, you will get a geodesic

connecting that points. There are infinitely many ways to do this, depending on

the number of times the string winds around the cylinder, but only one (or in

certain cases, two) of these geodesics minimizes the length between the two

points.

A geodesic may intersect itself. You can create an example of this by

wrapping a string tightly around a cone.

Figure 2. Geodesics

Definition 2.2 Geodesics Triangle. A geodesic triangle is a


triangle which consists of three vertices connected by
geodesics .
Module 2. Spherical Geometry 48

We have to remember that the angles of a geodesic triangle on a curved surface

need not add up to .

Example 2.3. Make a paper cone by joining the edges of a circular sector. Mark

three points , , and on the cone, and join them with geodesic segments by

flattening the cone out on a table and connecting the points with line segments.

These line segments remain shortest curves on the paper even when it is lifted

off the table, unflattened, and bent into a cone, because flattening or unflattening

the paper does not stretch or shrink it, and so does not distort lengths within the

paper. Thus is a geodesic triangle on the cone.

Let be the angle subtended by the circular sector. If the vertex of the

cone lies in the interior of , you will find that

In particular if .

Figure 3. Triangle on a Cone

You can make a circular sector that subtends an angle by gluing

together two smaller sectors. If you join together the edges of this sector you will

get a saddle – shaped surface. The equation previously mentioned was

applicable to this also. The angles of a geodesic triangle on a saddle – shaped


Module 2. Spherical Geometry 49

surface add up to less than if the vertex of the sector is in the triangle’s

interior.

Figure 4. Triangle on a saddle

Based on the examples, we might guess that the angles of a geodesic triangle

on a surface add up to less than if the surface id saddle – shaped, or more

than if the surface is a bowl – shaped like a cone. Such a result would

enable two – dimensional beings who live inside the surface to discover whether

it is saddle – or bowl – shaped, without ever going outside the surface by adding

up the angles of a geodesic triangle and comparing the sum with . A similar

procedure would enable three – dimensional beings such as ourselves to

discover whether or not their universe is curved without having to leave their

universe to make measurements.

DISCUSSION BOARD
We will now try to interact with each other in a two way

process at a least possible way! I will post a question/s and try to answer it on

your own.

1. How do we describe Geodesics?

2. Can you reiterate the description of Geodesic Triangle?


Module 2. Spherical Geometry 50

3. What tips could you give to your fellow students who were taking the same

course to make this topic easier to understand?

POST COMPETENCY CHECKLISTS


Let us try to determine if you learned something from our topic. Answer the
following Self-Assessment Question.

SELF – ASSESSMENT QUESTION 2.1


1. Execute the steps in Example 2.2. Make sure to capture all the
sequence of steps. Upload the photos of your activity. Indicate
captions for each and arrange them properly.
2. Do the steps in Example 2.3. Make sure to capture all the sequence of
steps. Upload the photos of your activity. Indicate captions for each
and arrange them properly.
3. Prove the equation mentioned in Example 2.3 for surfaces built from
sectors subtending an arbitrary angle . Show that if
then no geodesic triangle in the surface has the vertex of the
sector in its interior. What is the sum of the angles of a geodesic
triangle on these surfaces if the vertex of the surface is not in its
interior?

EXPLORE!!!!
After going through the first subtopic, I hope that you can

visualize geodesic triangle. If not so, it’s okay! You just need some time. Take it

slowly but surely! Meanwhile, we will proceed to our next concept to deal with –

Geodesics on Spheres.

GEODESICS ON SPHERES

We will now try to explore spheres. I advise you to scan your notes on
Module 2. Spherical Geometry 51

solid geometry for it will be helpful at this moment.

Definition 2.3 Great Circle. A great circle is the


intersection of a sphere and a plane that passes through the
center of the sphere. All other circles on the sphere are
“small circles”.

Figure 5.

Let and be two points on the sphere and let be its center. The arc

subtended by the is a segment of a great circle with length,

( ) ( )

where is the radius of the sphere and is measured in radians.

It is convenient to use spherical coordinates when computing the lengths

of curves in the sphere. Set up a system of rectangular coordinates on with

the origin at the center of the sphere and the positive - axis piercing the

sphere at . The spherical coordinates of a point on the sphere are ( )

where,

is the radius of the sphere,

, and

is the angle between the positive - axis and the projection of ⃗⃗⃗⃗⃗ into the
Module 2. Spherical Geometry 52

plane

Elementary trigonometry shows that the spherical coordinates are related

to rectangular coordinates by the formulas:

Figure 6. Spherical Coordinates

THEOREM 1. The shortest path between two points on a sphere is an arc of a

great circle.

Proof. Let , - be a parametrized curved in with

( ) and ( ) .

Write ( ) ( ( ) ( ) ( )) in rectangular coordinates. The length of the curve

is given by the formula

( ) ∫ √ ( ) ( ) ( )

In spherical coordinates,

( ) ( ) ( )

( ) ( ) ( )

( ) ( ).

Plug the derivatives,


Module 2. Spherical Geometry 53

( ) ( ( ) ( ) ( ) ( ) ( ) ( ))

( ) ( ( ) ( ) ( ) ( ) ( ) ( ))

( ) ( ) ( )

into the integrand and get,

( ) ∫ √ ( ) ( ) ( )

∫ ( )

( ( ) ( ))

( )

( )

with strict inequality unless ( ) or ( ) for all , that is, unless

never leaves the arc . This completes the proof.

The geometry of geodesics on the sphere is different from the geometry of

lines in the plane. For instance a sphere has no parallel geodesics since two

great circles always intersect at a pair of diametrically opposite points.

Figure 7. Great Circles always intersect

DISCUSSION BOARD
We will now try to interact with each other in a two way
Module 2. Spherical Geometry 54

process at a least possible way! I will post a question/s and try to answer it on

your own.

1. How do we describe Great Circle of a Sphere?

2. How do we transform the coordinate of a point from Spherical Coordinates

to Rectangular Coordinates?

3. What advise could you give to your fellow students who were taking the

same course to make this topic easier to understand?

POST COMPETENCY CHECKLISTS


Let us try to determine if you learned something from our topic. Answer the
following Self-Assessment Question.

SELF – ASSESSMENT QUESTION 2.2


1. Given the following coordinates of points in Spherical Coordinates,
transform them into Rectangular Coordinates and draw the indicated
points.
a. . /
b. . /
c. . /

2. Derive a formula/s to transform points from Rectangular Coordinates to


Spherical Coordinates.

EXPLORE!!!!

So far, we had examined Geodesic on Spheres and Spherical


Module 2. Spherical Geometry 55

Coordinate System. I am confident that you all got it correctly! If not so, don’t be

too much worried! After all, “learning is an evolutionary process” as Dr. Brenda

Corpuz and Dr. Gloria Salandanan said. Just be patience! For the meantime, we

will continue on The Six Angles of Spherical Triangles.

THE SIX ANGLES OF A SPHERICAL TRIANGLE

A Spherical Triangle is a geodesic triangle on the surface of a sphere. Let

be a spherical triangle with side opposite vertex , side opposite vertex

, and side opposite vertex , on a sphere with center at . has six

angles: three arc angles , and and three vertex angles , and .

Figure 8. Six Angles

Arc angles measure the angles subtended by the sides of the triangle.

(⃗⃗⃗⃗⃗ ⃗⃗⃗⃗⃗ ) , (⃗⃗⃗⃗⃗ ⃗⃗⃗⃗⃗ ), and (⃗⃗⃗⃗⃗ ⃗⃗⃗⃗⃗ ),

where:

(⃗⃗⃗⃗⃗ ⃗⃗⃗⃗⃗ ) denotes the angle between vectors ⃗⃗⃗⃗⃗ and ⃗⃗⃗⃗⃗

(⃗⃗⃗⃗⃗ ⃗⃗⃗⃗⃗ ) denotes the angle between vectors ⃗⃗⃗⃗⃗ and ⃗⃗⃗⃗⃗

(⃗⃗⃗⃗⃗ ⃗⃗⃗⃗⃗ ) denotes the angle between vectors ⃗⃗⃗⃗⃗ and ⃗⃗⃗⃗⃗

Vertex angles measure three things at once. equals,

1. The angle between the arcs and at ,


Module 2. Spherical Geometry 56

2. The angle between a vector ⃗⃗⃗⃗ that is tangent to at , and a vector

⃗⃗⃗⃗⃗⃗ that is tangent to at ,

3. The angle between the planes ̅̅̅̅̅̅ and ̅̅̅̅̅̅.

The first and second items in the above list are equal by definition. The second

and third items are equal because ⃗⃗⃗⃗ and ⃗⃗⃗⃗⃗⃗ are perpendicular to the line ⃡⃗⃗⃗⃗

where the planes ̅̅̅̅̅̅ and ̅̅̅̅̅̅ intersect. Similar statements holds for and

. To simplify, the notation that we will use,

⃗⃗⃗ ⃗⃗⃗⃗⃗ , ⃗⃗⃗ ⃗⃗⃗⃗⃗ , and ⃗⃗⃗ ⃗⃗⃗⃗⃗ .

We shall consider (⃗⃗⃗ ⃗⃗⃗ ) and (⃗⃗⃗ ⃗⃗⃗ ) as representing the same angle;

because our purposes angles of a spherical triangle have no orientation.

LEMMA 1. In a Spherical triangle, ,

1. The arc angles are:

( ⃗⃗⃗⃗ ⃗⃗⃗ )

( ⃗⃗⃗⃗ ⃗⃗⃗ )

( ⃗⃗⃗⃗ ⃗⃗⃗ )

2. The vertex angles are:

( ⃗⃗⃗ ⃗⃗⃗ ⃗⃗⃗⃗ ⃗⃗⃗ )

( ⃗⃗⃗ ⃗⃗⃗ ⃗⃗⃗⃗ ⃗⃗⃗ )

( ⃗⃗⃗ ⃗⃗⃗ ⃗⃗⃗⃗ ⃗⃗⃗ )

where is the “cross product”.


Module 2. Spherical Geometry 57

Figure 9. Vertex Angle

Proof. 1. This is just the restatement of the definition of the arc angles.

2. ⃗⃗⃗ x ⃗ is perpendicular to ̅̅̅̅̅̅, and ⃗⃗⃗ x ⃗⃗⃗ is perpendicular to ̅̅̅̅̅̅. This

almost proves that ( ⃗⃗⃗ ⃗⃗⃗ ⃗⃗⃗⃗ ⃗⃗⃗ ) since an angle between two planes

equals the angle between their normal vectors. The problem is that two

intersecting planes actually determine two supplementary angles, so we need to

make sure that ( ⃗⃗⃗ ⃗⃗⃗ ⃗⃗⃗⃗ ⃗⃗⃗ ) equals and not .

Let ⃗⃗⃗ be tangent to and let ⃗⃗⃗⃗⃗⃗ be tangent to at .

( ⃗⃗⃗ ⃗⃗⃗⃗⃗ )

Since ⃗⃗⃗ and ⃗⃗⃗⃗⃗⃗ are perpendicular to ⃗⃗⃗ the right hand rule says that ⃗⃗⃗ x ⃗⃗⃗

points in the direction obtained by rotating ⃗⃗⃗ ninety degrees to the right around

⃗⃗⃗ . Similarly, ⃗⃗⃗ x ⃗⃗⃗⃗⃗ points to the right of ⃗⃗⃗⃗⃗⃗ around ⃗⃗⃗ . Rotations does not

change angles, so

( ⃗⃗⃗ ⃗⃗⃗⃗⃗ ) (⃗⃗⃗ ⃗⃗⃗ ⃗⃗⃗ ⃗⃗⃗⃗⃗ ).

But, ⃗⃗⃗ x ⃗⃗⃗ points the same direction as ⃗⃗⃗ x ⃗⃗⃗ , and ⃗⃗⃗ x ⃗⃗⃗⃗⃗ points the same

direction as ⃗⃗⃗ x ⃗⃗⃗⃗ . Thus,

(⃗⃗⃗ ⃗⃗⃗ ⃗⃗⃗ ⃗⃗⃗⃗⃗ ) (⃗⃗⃗ ⃗⃗⃗ ⃗⃗⃗ ⃗⃗⃗ ).


Module 2. Spherical Geometry 58

This implies that,

(⃗⃗⃗ ⃗⃗⃗ ⃗⃗⃗ ⃗⃗⃗ ).

Similar arguments shows that (⃗⃗⃗ ⃗⃗⃗ ⃗⃗⃗ ⃗⃗⃗ )and (⃗⃗⃗ ⃗⃗⃗ ⃗⃗⃗ ⃗⃗⃗ ).

This completes the proof.

THEOREM 2. In a Spherical triangle ,

( )

in radians, where is the radius of the sphere.

In particular, .

Proof. Given a point on the sphere, let be the point on the opposite end of a

diameter from . Two great circles meeting at an angle at bound a sector of

the sphere with vertices and . Figure 10 shows that the ratio of the sectors’

area to the sphere’s area is the same as the ratio of the angle to an angle

subtending a full circle:

( )
( )

Since the area of the sphere is , it follows that,

( )

Figure 10.Area of a Sector = 2 x (angle of Sector)


Module 2. Spherical Geometry 59

Each vertex angle of subtends a sector on the sphere; call them the

“ ”, the “ ”, and the “ ”. Let be the hemisphere

that contains and is bounded by . contains both the “ ” and the

“ ” but only part of the “ ”. Figure 11 shows two views of the

three sectors. In the view on the right the sphere had been rotated so that is

the visible hemisphere; part of the wraps around to the back of the

sphere behind the visible hemisphere.

Figure 11. Rotating the Sphere

Based from Figure 11, we can see that,

( ) ( ) ( ( ) ) ( ( ) )

( ( )( ) )

( ) ( ) ( ( ) )

( ) ( ) ( ( ) )

( ) ( ) ( ( ))

Reflection through the center of the Sphere is an isometry that takes

each point on the Sphere to the point at the opposite end of a diameter.

( ( )) ( )( ) so, since is an isometry, it follows that

( ( )) ( ( )( ) )
Module 2. Spherical Geometry 60

( ) ( ) ( ( )( ) )

Comparing the sums of the previous equations, we find that

( ) ( ) ( )

( ) ( )

Since

( )

( )

( )

And

( ) ( *( ( ))

Then,

( )

Divide by , we get

( )

This completes the proof.

Figure 12.
Module 2. Spherical Geometry 61

COROLLARY 1. It is impossible to have an isometry between a region on a

Sphere and a region in the Plane, so long as the region in the Sphere includes at

least one geodesic triangle together with its’ interior.

Proof. Isometries preserve distance so they map shortest curves to shortest

curves. Therefore, such an isometry would map geodesic triangles on the Sphere

to geodesic triangles on the Plane. But isometries also preserve angles.

Therefore the sum of the angles of the Spherical triangle would have to equal the

sum of the angles of a planar triangle, and this contradicts theorem on isometries

mentioned in Module 1.

DISCUSSION BOARD
We will now try to interact with each other in a two way

process at a least possible way! I will post a question/s and try to answer it on

your own.

1. How do we describe the Six Angles of a Spherical Triangle?

2. What can you say about area of a sector on a Sphere?

3. What advise could you give to your fellow students who were taking the

same course to make this topic easier to understand?

POST COMPETENCY CHECKLISTS


Let us try to determine if you learned something from our topic. Answer the
following Self-Assessment Question.
Module 2. Spherical Geometry 62

SELF – ASSESSMENT QUESTION 2.3


1. Create your own example of Spherical Triangle. Label the six angles.
2. Construct your own example of sector on a Sphere.

EXPLORE!!!!

We had examined the Six Angles of a Spherical Triangle and

Sector on a Sphere. At this point in time, we will continue on another topic related

to Spherical Triangle – The Law of Cosines for Sides.

THE LAW OF COSINES FOR SIDES

Let be a spherical triangle with side opposite vertex , side

opposite vertex , and side opposite vertex .

PROPOSITION 1. The Law of Cosines for Sides.

Proof. We will use the dot product to compute for ,

(⃗⃗⃗ ⃗⃗⃗ ⃗⃗⃗ ⃗⃗⃗ ), so

(⃗⃗⃗ ⃗⃗⃗ ) (⃗⃗⃗ ⃗⃗⃗ ) |⃗⃗⃗ ⃗⃗⃗ ||⃗⃗⃗ ⃗⃗⃗ | ,

Also (⃗⃗⃗ ⃗⃗⃗ ) and (⃗⃗⃗ ⃗⃗⃗ ) so

|⃗⃗⃗ ⃗⃗⃗ | |⃗⃗⃗ ||⃗⃗⃗ |

|⃗⃗⃗ ⃗⃗⃗ | |⃗⃗⃗ ||⃗⃗⃗ | .

Thus,

(⃗⃗⃗ ⃗⃗⃗ ) (⃗⃗⃗ ⃗⃗⃗ ) (|⃗⃗⃗ ||⃗⃗⃗ | )(|⃗⃗⃗ ||⃗⃗⃗ | )


Module 2. Spherical Geometry 63

since |⃗⃗⃗ | |⃗⃗⃗ | |⃗⃗⃗ | . A standard identity form vector algebra enables us to

simplify the left – hand side of the equation:

⃗⃗⃗ ⃗⃗⃗ ⃗⃗⃗ ⃗⃗⃗


(⃗⃗⃗ ⃗⃗⃗ ) (⃗⃗⃗ ⃗⃗⃗ ) ( *
⃗⃗⃗ ⃗⃗⃗ ⃗⃗⃗ ⃗⃗⃗

|⃗⃗⃗ | |⃗⃗⃗ ||⃗⃗⃗ |


( +
|⃗⃗⃗ ||⃗⃗⃗ | |⃗⃗⃗ ||⃗⃗⃗ |

. /

Simplifying result,

Divide both sides by ,

Add to both sides,

This completes the proof.

Important Note: In using the Law of Cosines to solve a triangle, the accuracy of

the results could be ruined by rounding off, so it is best to carry out calculations

to at least six decimal places.

DISCUSSION BOARD
We will now try to interact with each other in a two way
Module 2. Spherical Geometry 64

process at a least possible way! I will post a question/s and try to answer it on

your own.

1. How do we write the formula for Law of Cosines for Sides?

2. What advise could you give to your fellow students who were taking the

same course to make this topic easier to understand?

POST COMPETENCY CHECKLISTS


Let us try to determine if you learned something from our topic. Answer the
following Self-Assessment Question.

SELF – ASSESSMENT QUESTION 2.4


Using the congruence postulates (SAS) and (SSS), solve and illustrate the
following Spherical triangle if,
1. , ,
2. , ,
3. , ,
4. , ,

EXPLORE!!!!

We are done with Law of Cosines for Sides of Spherical

Triangle. I hope that you are still okay at this moment in time! Before we

continue, I advice you to do some physical exercises so that your blood would

circulate properly through your body!


Module 2. Spherical Geometry 65

At this juncture, we wil continue on – The Dual Spherical Triangle.

THE DUAL SPHERICAL TRIANGLE

One good thing about the dual ( or “polar”) triangle is the fact that its’

vertices correspond to the sides of the original triangle and its’ sides correspond

to the vertices of the original triangle. Several theorem about dual triangle were

given that would be an advantage for us to easily grasp the lesson.

Definition 2.4 Dual Spherical Triangle. Let 𝐴𝐵𝐶 be a


Spherical Traingle. The dual triangle * 𝐴𝐵𝐶 is the
Spherical Triangle 𝐴 𝐵 𝐶 where,
⃗⃗⃗ 𝑥 ⃗⃗⃗
𝐵 𝐶
⃗⃗⃗⃗
𝐴
𝑅 𝑠𝑖𝑛 𝑎
⃗⃗⃗
𝐶 𝑥 ⃗⃗⃗
𝐴
⃗⃗⃗⃗
𝐵
𝑅 𝑠𝑖𝑛 𝑏
⃗⃗⃗
𝐴 𝑥𝐵 ⃗⃗⃗
⃗⃗⃗⃗
𝐶
𝑅 𝑠𝑖𝑛 𝑐

Here, is the radius of the Sphere, and ⃗⃗⃗⃗ ⃗⃗⃗⃗⃗⃗⃗ , ⃗⃗⃗⃗ ⃗⃗⃗⃗⃗⃗⃗⃗ and ⃗⃗⃗⃗ ⃗⃗⃗⃗⃗⃗⃗

where is the center of the Sphere. The purpose of the scalars ,

and in the definition is to adjust the lenghts of the vectors so that

|⃗⃗⃗⃗⃗ | |⃗⃗⃗⃗⃗ | |⃗⃗⃗⃗⃗ | .

⃗⃗⃗⃗ is perpendicular to the plane ̅̅̅̅̅̅ containing side of the original

triangle. Likewise, ⃗⃗⃗⃗ is perpendicular to the plane containing and ⃗⃗⃗⃗ is


Module 2. Spherical Geometry 66

perpendicular to the plane containing . In this sense the vertices of the dula

triangle correspond to the sides of the original triangle.

PROPOSITION 2.Let , , be the vertices of the dual triangle. Let be the

side opposite vertex , be the side opposite vertex and be the side

opposite vertex . Then,

⃗⃗⃗⃗ ⃗⃗⃗⃗
⃗⃗⃗ ( )

⃗⃗⃗⃗ ⃗⃗⃗⃗
⃗⃗⃗ ( )

⃗⃗⃗⃗ ⃗⃗⃗⃗
⃗⃗⃗ ( )

where

[⃗⃗⃗ ⃗⃗⃗ ⃗⃗⃗ ]

Proof.

⃗⃗⃗ ⃗⃗⃗ ⃗⃗⃗ ⃗⃗⃗


⃗⃗⃗⃗ ⃗⃗⃗⃗ ( ) ( )

(⃗⃗⃗ ⃗⃗⃗ ) (⃗⃗⃗ ⃗⃗⃗ )


⃗⃗⃗⃗ ⃗⃗⃗⃗

Use vector identity to multiply out the numerator on the right side,

(⃗⃗⃗ ⃗⃗⃗ ) (⃗⃗⃗ ⃗⃗⃗ ) .(⃗⃗⃗ ⃗⃗⃗ ) ⃗⃗⃗ / ⃗⃗⃗ .(⃗⃗⃗ ⃗⃗⃗ ) / ⃗⃗⃗

(⃗⃗⃗ ⃗⃗⃗ ) (⃗⃗⃗ ⃗⃗⃗ ) ⃗⃗⃗ [⃗⃗⃗ ⃗⃗⃗ ⃗⃗⃗ ]

Hence,
Module 2. Spherical Geometry 67

[⃗⃗⃗ ⃗⃗⃗ ⃗⃗⃗ ]


⃗⃗⃗⃗ ⃗⃗⃗⃗ ⃗⃗⃗ ( )

So,

⃗⃗⃗ (⃗⃗⃗⃗ ⃗⃗⃗⃗ ) ( )


[⃗⃗⃗ ⃗⃗⃗ ⃗⃗⃗ ]

⃗⃗⃗⃗ ⃗⃗⃗⃗
Thus, ⃗⃗⃗ points the same direction as ( *.

( ). Moreover,

|⃗⃗⃗ | and

|⃗⃗⃗⃗ ⃗⃗⃗⃗ |

since |⃗⃗⃗⃗ | |⃗⃗⃗⃗ | and (⃗⃗⃗⃗ ⃗⃗⃗⃗ ) . Therefore,

⃗⃗⃗⃗ ⃗⃗⃗⃗
⃗⃗⃗ ( )

The other equations follow in a similar way. This completes the proof.

COROLLARY 2. The dual of the Dual Triangle.

The dual of * is,

[⃗⃗⃗ ⃗⃗⃗ ⃗⃗⃗ ]


*( ) {
( )( )( ) [⃗⃗⃗ ⃗⃗⃗ ⃗⃗⃗ ]

COROLLARY 3. *( ) .

Proof. By Corollary 2. either *( ) equals or it equals the reflection of

through the center of the sphere.


Module 2. Spherical Geometry 68

COROLLARY 4. Let , and be the vertices of * . Let be the side

opposite vertex , the side opposite vertex , and the side opposite vertex

. Then,

Proof. By Definition 2.4,

(⃗⃗⃗ ⃗⃗⃗ )

(⃗⃗⃗ ⃗⃗⃗ ⃗⃗⃗ ⃗⃗⃗ )

(⃗⃗⃗⃗⃗⃗⃗ ⃗⃗⃗ ⃗⃗⃗ ⃗⃗⃗ )

(⃗⃗⃗ ⃗⃗⃗ ⃗⃗⃗ ⃗⃗⃗ )

Thus

Applying this result to the dual ** ** ** of * * *, we have

Where is the side opposite . But, , for Corollary 3 says that

** ** ** is congruent to . Hence,

This proves the first equation. The other equations follows in similar way.
Module 2. Spherical Geometry 69

DISCUSSION BOARD

We will now try to interact with each other in a two way

process at a least possible way! I will post a question/s and try to answer it on

your own.

1. How do we describe Dual Spherical Triangle?

2. Can you give your own example of Dual Spherical Triangle?

3. What advise could you give to your fellow students who were taking the

same course to make this topic easier to understand?

POST COMPETENCY CHECKLISTS


Let us try to determine if you learned something from our topic. Answer the
following Self-Assessment Question.

SELF – ASSESSMENT QUESTION 2.5


1. Complete the proof for the remaining two equations in Corollary 4.
√ √ √
2. Let ( ), . /, . / on the unit sphere centered
at ( ) in
a. Find the vertices of the dual triangle of
b. Sketch and *

EXPLORE!!!!

I hope that you are not overwhelmed by the concepts,


Module 2. Spherical Geometry 70

theorems, corollary and proof about The Dual Spherical Triangle! We will again

continue on another concepts – The Law of Cosines for Angles.

THE LAW OF COSINES FOR ANGLES

The next result were derived from Law of Cosines for Sides by exploiting

the duality.

COROLLARY 5. The law of Cosine for Angles

Let be a spherical triangle with side opposite vertex , side opposite

vertex , and side opposite vertex . Then,

Proof. Apply the Law of Cosines for Sides to the dual triangle * * * * :

where is the side opposite of , is the side opposite of , and is the

side opposite of . By Corollary 4, , and

. Plug these into the previous equation, then use the fact that

( ) and ( ) for every angle to get the

previous equation in Corollary 5. This completes the proof.

Given its vertex angles one can completely solve a spherical triangle by

using the Law of Cosines, two spherical triangles with the same vertex angles

are congruent. In plane geometry, one can say only that tow triangles with the

same angles are similar.

Example 2.4. Find the sides , , of a spherical triangle if its’ vertex angles

are , and .

Solution: by the Law of cosines for Angles,


Module 2. Spherical Geometry 71

( )

Similarly,

( )

And

( )

When we combine several spherical angles having the same vertex, the

angles add up in the same way as they do in the plane. For example,

in Figure13. The reason is that the spherical angle between two

curves is, by definition, equal to the angle between tangents to the curves, so if

several angles have the same vertex they combine as do angles between

vectors in a single tangent plane.


Module 2. Spherical Geometry 72

Figure 13.

DISCUSSION BOARD

We will now try to interact with each other in a two way

process at a least possible way! I will post a question/s and try to answer it on

your own.

1. How do we write the formula for Law of Cosines for Angles?

2. What advise could you give to your fellow students who were taking the

same course to make this topic easier to understand?

POST COMPETENCY CHECKLISTS


Let us try to determine if you learned something from our topic. Answer the
following Self-Assessment Question.

SELF – ASSESSMENT QUESTION 2.6


Use (ASA) and (AAA) Congruence Postulates to solve the following Spherical
Triangles.
Given the spherical triangle if:
1. , ,
2. , ,
Module 2. Spherical Geometry 73

EXPLORE!!!!

So far, we had examined the Law of Cosines for Angles of

Spherical Triangles. I assumed that you are all doing well! If not so, that’s fine!

Just keep on reading and reading until such time that you find it so clear on your

mind. Meanwhile, we will continue on Law of Sines for Spherical Triangles.

PROPOSITION 3. The Law of Sines for Spherical Triangles

Let be a spherical triangle with side opposite vertex , side opposite

vertex , and side opposite vertex . Let , and be the vertices of the

dual triangle. Then,

[⃗⃗⃗ ⃗⃗⃗ ⃗⃗⃗ ]


( )
[⃗⃗⃗⃗ ⃗⃗⃗⃗ ⃗⃗⃗⃗ ]

where

is the sign of the [⃗⃗⃗ ⃗⃗⃗ ⃗⃗⃗ ]

Proof. Let be the radius of the sphere,

[⃗⃗⃗⃗ ⃗⃗⃗⃗ ⃗⃗⃗⃗ ] ⃗⃗⃗⃗ (⃗⃗⃗⃗ ⃗⃗⃗⃗ )

⃗⃗⃗ ⃗⃗⃗
[⃗⃗⃗⃗ ⃗⃗⃗⃗ ⃗⃗⃗⃗ ] ( ) (⃗⃗⃗ *

⃗⃗⃗ (⃗⃗⃗ ⃗⃗⃗ )


[⃗⃗⃗⃗ ⃗⃗⃗⃗ ⃗⃗⃗⃗ ] ( *( )

[⃗⃗⃗ ⃗⃗⃗ ⃗⃗⃗ ]


( *( )
Module 2. Spherical Geometry 74

But, since + = . Substitute this to the previous

equation,

[⃗⃗⃗ ⃗⃗⃗ ⃗⃗⃗ ]


[⃗⃗⃗⃗ ⃗⃗⃗⃗ ⃗⃗⃗⃗ ]

Because , this proves thr first part of the Proposition 3; the rest follows in

similar way. This completes the proof.

DISCUSSION BOARD

We will now try to interact with each other in a two way

process at a least possible way! I will post a question/s and try to answer it on

your own.

1. How do we write the formula for Law of Sines for Spherical Triangles?

2. What advise could you give to your fellow students who were taking the

same course to make this topic easier to understand?

POST COMPETENCY CHECKLISTS


Let us try to determine if you learned something from our topic. Answer the
following Self-Assessment Question.

SELF – ASSESSMENT QUESTION 2.7


In the following sets of questions, are vertex angles and are arc
angles of Spherical Triangles.
1. Given , , . Determine whether the following
angles are acute or obtuse:
Module 2. Spherical Geometry 75

a. ( )
b. ( )
c. ( )
2. Given , , . Which of the following angles are
acute and which are obtuse?
a. ( )
b. ( )
c. ( )

EXPLORE!!!!

We had examined the Law of Sines for Spherical Triangle. I

hope that you were not confused with a lot of formulas and concepts that were

introduced to you! For the meantime, we will continue on exploring Some

Navigation Problems where you can apply the concepts of Spherical Triangle.

NAVIGATION PROBLEMS

As what Jennings had mentioned, spherical trigonometry is commonly

used to solve problems arising from astronomy and navigation.

LATITUDE, LONGITUDE and BEARINGS

The Latitude of a point is the angle between the point and the equator,

measured along a great circle passing through the point and the poles. For

example a point at latitude is north of the equator and a point at latitude

is south of the equator.

The Longitude of a point is the angle between two great circles, one

connecting the point to the north and south poles and the other connecting the
Module 2. Spherical Geometry 76

north and south poles to the observatory at Greenwich, England. A point at

longitude is west of Greenwich; a point at longitude is east of

Greenwich.

Bearings measure directions on the earth’s surface. For example the

direction bearing points to the east of due north and the direction

points to the west of due south.

A Nautical Mile is the length of arc on the earth’s surface that is subtended

by an angle of one minute . / with vertex at the center of the earth.

A Statute Mile ( ) is the unit of distance commonly used on land.

The radius of the earth is approximately 3,960 statute miles.

( *( *( )

Speed is sometimes measured in knots. One knot is one nautical mile per hour.

Example 2.5. Find the distance from New Orleans to New York and the bearing

(direction) from each city to the other. Use the following data:

City latitude longitude

New Orleans

New York

Solution: Set up a spherical triangle whose three vertices are New York, New

Orleans, and the North Pole. Since New York is located at


Module 2. Spherical Geometry 77

latitude, the angle between New York and the North Pole is

. The angle between New Orleans and the North Pole is .

The angle between the arc connecting New Orleans to the North Pole and the

arc connecting New York to the North Pole is the diffence between the longitudes

of New York and New Orleans, .

Figure 14. A Geographical Spherical Triangle

Let be the arc connecting New Orleans to New York. By Law of

Cosines for Sides,

So,

( )

radians.

Multiply this by the radius of the earth to get the distance from New York to New

Orleans.

( )( )

The vertex angle at New Orleans measures the direction from New

Orleans to New York. By Law of Cosines for Sides,


Module 2. Spherical Geometry 78

( )

Since New York lies to the east of New Orleans, the bearing form New Orleans

to New York is approximately.

A similar computation shows that the vertex angle at New York is

approximately. According to the standard bearing notation, all angles

should measure between and form north or south, so this angle should be

described as bearing ( ) west from south:

The bearing from New York to New Orleans is approximately.

DISCUSSION BOARD

We will now try to interact with each other in a two way

process at a least possible way! I will post a question/s and try to answer it on

your own.

1. As soon to be teachers, how do we apply Navigation Problems in your

field of expertise?

2. What advise could you give to your fellow students who were taking the

same course to make this topic easier to understand?


Module 2. Spherical Geometry 79

POST COMPETENCY CHECKLISTS


Let us try to determine if you learned something from our topic. Answer the
following Self-Assessment Question.

SELF – ASSESSMENT QUESTION 2.8


A. Find the distances between the following places, also the bearing of each
from the other. Refer to the values on the table below.

Place Latitude Longitude


Cape of Good Hope
Chicago
Moscow
New York
Paris
Rio de Janeiro
San Francisco
Sydney

1. New York and San Francisco


2. New York and Paris
3. New York and Cape of Good Hope
4. San Francisco and Sydney
5. San Francisco and Rio de Janeiro
6. New York and Rio de Janeiro
7. Rio de Janeiro and Sydney
8. Moscow and San Francisco

B. Surf the internet and get the coordinates of your place of residence both
latitude and longitude. Search also for the coordinates of your school
(CBSUA). Compute the distance and bearing from your place of residence
to your school.
Module 2. Spherical Geometry 80

EXPLORE!!!!

We had explored the applications of Spherical Triangles to

Navigation Problems. I hope that you can still process the information that was

introduced to you. In not, well it’s time for you to take a short break! Meanwhile,

we will continue on Mapmaking.

MAPMAKING

As stipulated in the book of Jennings, it is impossible to make a map of

any part of a spherical earth on a flat sheet of paper without introducing some

type of distortion. The challenge to mapmaker is to control the distortion so that

the information he wanted to depict was displayed as clearly and accurately as

possible. The kind of distortion that is acceptable depends on the map’s intended

use. There are four map projections that will be introduced in this section.

Central Projection. A map that uses lines to represent geodesics.

Figure 15. Central Projection: latitude to

Cylindrical Projection. A map that preserves areas.


Module 2. Spherical Geometry 81

Figure 16. Cylindrical Projection: to

Mercator Projections and Stereographic Projections. Maps that preserves

angles.

Figure 17. Mercator Projections: latitude to


Module 2. Spherical Geometry 82

Figure 18. Stereographic Projections: latitude to

CENTRAL PROJECTIONS

A central projection maps a hemisphere onto a plane by projecting

along lines extending radially from the center of the hemisphere. Let be a

plane that does not contain and is parallel to the great circle formin gthe

boundary of the hemisphere. The projection ( ) of a point is the

point where the line ⃡⃗⃗⃗⃗⃗ intersects .

( ) ⃡⃗⃗⃗⃗

Figure 19. Central Projection

All mappings from the sphere to the plane distort distances. With central

projection the distortion is minimal near the point on where the tangent plane
Module 2. Spherical Geometry 83

is parallel to . Distortion becomes increasingly extreme near the edge of the

hemisphere.

PROPOSITION 4. Let be a curve on . ( ) is a line segment if and only if

is an arc of a great circle.

Proof. If is an arc of a great circle then lies in the plane containing . Thus

the line connecting any point on to lies in . It follows that the entire

projection ( ) is contained in . Therefore, ( ) is a line segment.

Conversely, if ( ) is a line segment, then is contained in the plane

joining the line segment to the center of the sphere. Hence, is an arc of the

geat circle.

Figure 20.Projecting a Great Circle into a Line

CYLINDRICAL PROJECTIONS

A cylindrical projection maps a sphere minus two diametrically opposite

points into a cylinder by projecting out along lines extending radially out from a

diameter of the sphere.

Figure 21. Cylindrical Projection


Module 2. Spherical Geometry 84

Let be a sphere and a cylinder whose axis contains a diameter of . To

project a point into the cylinder, extend a ray from a point on the diameter

through , perpendicular to the diameter. The projection ( ) is the point where

the ray intersects the cylinder.

Once the sphere is projected out onto the cylinder a flat map can be

produced by slitting the cylinder from end to end, unrolling it, and laying it out flat.

Distortions are least along the great circle where tangent planes on the sphere

are parallel to the tangent planes on the cylinder.

PROPOSITION 5. If the radii of the cylinder and sphere are equal the cylindrical

projection preserves areas. In other words, if is any region on the sphere then,

( ).

Proof. It is enough to prove that

( ) ( ( ))

whenever is an infinitisimal rectangle, since any area can be computed by

summing up areas of infinitisimal rectangles.

Set up a system of latitude and longitude coordinates ( ) on the surface

of the sphere. is the latitude, measured up from the equator where the cylinder

is tangent to the sphere, and is the longitude, measured around the axis of the

cylinder.

Latitude and longitude meridians are perpendicular to each other.

Therefore the infinitisimally nearby points given in latitude and longitude

coordinates by

( ) ( )
Module 2. Spherical Geometry 85

( ) ( )

are vertices of an infinitisimal rectangle on the sphere. Its area is

( ) ( ) ( )

And the area of its projection is,

( ( )) ( ( )) ( ( ))

With the approximations becoming exact in the limit as the lengths of the sides

approach zero.

Let

( )

The arc subtends an angle at a distance of , measured from the

axis of the cylinder. Thus,

( ) .

( ) subtends an angle at a distance from the axis. Thus

( ( )) .

It follows that

( ( ))
( )

On the other hand also is the angle between the tangent to arc at and the

cylinder, so

( ( ))
( )

Simplifying the result, we get

( ( ))
( )
.
Module 2. Spherical Geometry 86

Figure 22. Equal Areas

with equality in the limit as the lenghts of the sides approach zero. This

completes the proof.

CONFORMAL MAPS

We will try to introduce one of the related concepts to Cylindrical

Projections, the Conformal Maps.

Definition 2.5 Conformal Mapping. A conformal mapping


is a function 𝑓 that preserves angles. 𝑓 is a conformal
mapping if, whenever 𝐶 and 𝐶 are curves that meet in the
domain of 𝑓, then the angle subtended by 𝐶 and 𝐶 is
congruent tot eh angle subtended by 𝑓 (𝐶 ) and 𝑓(𝐶 ).

Example 2.6. An example of conformal mapping in is “dilation by a factor a”:

( ) ( ),

where is some constant. Such a map simply scales everything up a factor

, mapping each triangle to a similar triangle ( ).

If is an infinitesimal triangle in the domain of a conformal map ,

then is similar to ( ). Thus every conformal map acts like dilation at


Module 2. Spherical Geometry 87

the infinitesimal level. Conversely, it can be shown that this condition is enough

to ensure that is conformal. We will state it in the following form:

If acts like dilation on infinitesimally smalll rectangles then is a

conformal map.

Conformal mappings are quite special ut they are not as rare as we might

suppose. For example, every complex differentiable function is

conformal.

MERCATOR PROJECTIONS

As mentioned by George A. Jennings, the most common conformal map is

the Mercator projection, invented by Gerhard Kremer (1512 – 1594), also known

as Mercator. His idea was to alter the distortion in a cylindrical projection by

stretching the cylinder along its axis until the maps becomes conformal.

As previously mantioned,

( ( )) ( )

While.

( ( )) ( )

So in order to make cyclindrical projection conformal we needs to stretch the

length of ( ( )) by a factor of

Set up coordinates and on the surface of the cylinder, with

measuring the distance around the cylinder and measuring the distance up the

cylinder. When the cylindre is unrolled and laid out flat, and becomes a

system of rectangular coordinates on the resulting planar map. Applying


Module 2. Spherical Geometry 88

trigonometry, if ( ) is a point on the sphere given in latitude – longitude

coordinates, then it’s image under cylindrical coordinate projection is the pooint

( ) given by the following formulas:

Differentiating the second equation,

Hence, to stretch the height of an infinitesimal rectangle by a factor of , we

must replace by a function whose derivatives is times as large as the

derivatives of :

It follows that

| |

for some constant (which shall be taken as zero.). The result,

( )

( ) | |

where is an arbitrary positive constant, gives a formula for the Mercator

projection.

A line on a Mercator projection map represents a curve with a constant

heading relative to latitude and longitude lines; the angle between the curve and

a longitude meridian is the same at every point on the curve. Such a point on the
Module 2. Spherical Geometry 89

earth is called a rhumb line or loxodrome, it is particularly easy for a vessel to

follow since pilot simply needs to keep a constant heading on the compass.

STEREOGRAPHIC PROJECTIONS

A stereographic projection maps a sphere minus one point to a plane by

projecting along lines through the missing point. Let,

a sphere

a point on

a plane distinct from but parallel to the tangent plane at .

Figure 23. Stereographic Projection

If ( * +) then it’s stereographic projection ( ) is,

( ) ⃡⃗⃗⃗⃗ .

Distortion is least near points on the sphere where its tangent plane is parallel to

. To prove that stereographic projection is conformal, we will refer to the

following two simple lemmas:

LEMMA 2. Let , , , be planes in . If is parallel to then the lines

and subtend the same angle as the lines and .

Proof. Let be the point where the two lines meet in and be the point where

the other two lines meet in . Translation by ⃗⃗⃗⃗⃗ maps one set of lines to the

other.
Module 2. Spherical Geometry 90

Figure 24.Equal Angles

LEMMA 3. Let and be a pair of circle in intersecting in two points and

. Then the circles subtend the same angle at as they do at .

Proof. Reflection in the plane that forms the perpendicular bisector of ̅̅̅̅ maps

the angle at to the angle at .

Figure 25.Equal Angles

PROPOSITION 6.Stereographic projection is a conformal mapping.

Proof. Let be a point on the sphere, and let ⃗⃗⃗ , ⃗⃗⃗⃗ be tangent vectors

forming an angle at . Let and be planes conaining and such that,

is tangent to ⃗⃗⃗

is tangent to ⃗⃗⃗⃗

And set,

The cricles and meet at in the same angle as do ⃗⃗⃗ and ⃗⃗⃗⃗ , and

their projections meet in the projection of this angle. Thus, we need to show that
Module 2. Spherical Geometry 91

the angle formed by their projections at ( ). But the angle between circles at

is congruent to the angle between the circles at by Lemma 3, and the angle

between the circles at is congruent to the angle between their projections by

Lemma 2. Therefore, the angle between the circles is congruent to the angle

between their projections.

Figure 26.Stereographic projection is conformal

DISCUSSION BOARD

We will now try to interact with each other in a two way

process at a least possible way! I will post a question/s and try to answer it on

your own.

1. How do we describe Mapmaking?

2. Can you reiterate the map projections previously mentioned and their

similarities and differences?

3. What advise could you give to your fellow students who were taking the

same course to make this topic easier to understand?


Module 2. Spherical Geometry 92

POST COMPETENCY CHECKLISTS


Let us try to determine if you learned something from our topic. Answer the
following Self-Assessment Question.

SELF – ASSESSMENT QUESTION 2.9


1. Let be the unit sphere centered at the origin in .
a. Show that stereographic projection of from the point ( ) into
the plane is given by the formula
( ) . / for ( ) in .
b. Show that the inverse function is

( ) ( )

for ( ) in the plane.

2. Let , , and be as in (# 1), and let be a curve in . Show that


a. is a circle and if and only if ( ) is a line in .
b. is a circle and if and only if ( ) is a circle in .

EXPLORE!!!!

We had examined Mapmaking and explored several theories

and concepts related to it. I hope that you are still okay at this time! We need one

more step to finish our journey on Spherical Geometry. For the meantime, let us

continue on Applications of Stereographic Projection.

APPLICATIONS OF STEREOGRAPHIC PROJECTION

I will emphasized the discussion of George A. Jennings in this section

about applications of stereographic projection.


Module 2. Spherical Geometry 93

Figure 27. Stereographic Projection in the Plane

Figure 27 illustrates a stereographic projection of the circle in

from the point ( ) to the – axis. If ( ) is a point on the circle and ( ) is

its projection onto the – axis , then

since is the slope of the line connecting ( ) to ( ). We can solve for ( )

in terms of .

( )

Plug this into the equation of the circle, we get

( )

Subtract 1 from both sides,

( ) ( )

Then factor out ( )

, -,( ) ( )-

Assuming ( ) ( ) we have

( ) ( )

Solve for
Module 2. Spherical Geometry 94

Then plug back into the equation ( ) of the line,

( ) ( )

Application #1. Pythagorean Triples

A Pythagorean Triple is a sequence ( ) of three integers such that

(1)

for instance, ( ) and ( ) are Pythagorean triples.

We will find a formula that produces all the Pythagoren triples. If ( )

is a Pythagorean triple then ( ) is also, so it is enough to produce either one

of them. Also, if ( ) is a Pythagorean triple and is any integer, then

( ) is also a Pythagoren triple so it is enough to find all the Pythagorean

triples ( ) where and have no common factors (other than ). In

particular, and are not both zero; it follows that , too.

Divide (1) by and get,

(2)

Set

and (3)

and are rational nonzero numbers since and are nonzero integers.

Plug equations (3) into (2) and get,

Hence ( ) is a point on the unit circle, with rational coordinates.

Using stereographic projection, project ( ) from ( ) to the – axis.

By the previous equation, the projection of ( ) is the point ( ) where


Module 2. Spherical Geometry 95

(4)

Hence is a rational number, so we can write it as a fraction reduced to lowest

terms:

(5)

where and are integers with no common factor other than .

To get a formula for the Pythagorean triples,we simply reverse these steps

by solving for and in terms of and . By previous equations,

( ) ( )

Plug equation (5)

( * ( ,

. / . / (6)

These equations are satisfied if we set

( ) ( ) (7)

Equation (7) produces a Pythagorean triple for every pair of integers and .

However, there could be possibly go wrong with for they

might have a common factor which cancels out (7), thereby leading to a

solution,

( ) . / (8)

not covered by (7).


Module 2. Spherical Geometry 96

If were a common prime factor of , then

must divide evenly into ( ) ( ) and also into (

) ( ) . It follows that either is a common factor of and or

else .

By hypothesis, and have no common factors other than so must

be 2. Since divides evenly into , it follows that either and are both

even numbers or both odd numbers. In either case both and are

both even, so

and

are integers. Setting in (8), we have

( ) ( )

where,

( ) ( )

This equation has the same form as (7), except that the formula for and are

switched. The following claim was proved.

CLAIM 1. ( ) is a Pythagorean triple if and only if it can be written in the

form

( ) ( ) or ( )

where,

( ) ( )

and and are integers.

Application # 2. Integrals of Rational Trigonometric Functions


Module 2. Spherical Geometry 97

Consider an integration problem of the form

∫ ( ) (9)

where ( ) is a rational function.

(10)

(11)

where ( ) ( ) is an arbitrary point on the unit circle.

Differentiating (11), we obtain

( )

( )( *

( *

where the last line follows that

(12)

Simplifying the formula on the original equation,

∫ . / (13)

The integrand is a rational function of which can be integrated by partial

fractions.

Example 2.7. To compute

∫ ∫

Substitute (12) and (13)


Module 2. Spherical Geometry 98

∫( )( * ∫

∫0 1 by partial fraction

| | | | (14)

to get a solution to the original problem, rewrite this as function of .

Combine the previous equations to get,

And substitute this to | | | | , resulting to

∫ | | | |

DISCUSSION BOARD

We will now try to interact with each other in a two way

process at a least possible way! I will post a question/s and try to answer it on

your own.

1. Can you reiterate the Applications of Stereographic Projection?

2. What advise could you give to your fellow students who were taking the

same course to make this topic easier to understand?

POST COMPETENCY CHECKLISTS


Let us try to determine if you learned something from our topic. Answer the
following Self-Assessment Question.
Module 2. Spherical Geometry 99

SELF – ASSESSMENT QUESTION


2.10
1. Use the result of your answer to SAQ 2.9 ( 1- a) to find formulas for all
quadruples ( ) of integers such that .
2. Use the technique in this section to evaluate

ASSIGNMENT

1. Solve the following spherical triangles.

a. , ,

b. , ,

c. , ,

2. Find the perimeter of the spherical triangle in which ;

, and the radius of the sphere is 5 inches.

3. A triangle whose sides are and lies on a sphere of radius 10

inches. Find the difference between the area of this triangle and that of an

equilateral triangle having the same perimeter.

4. A triangle whose sides are and lies on a sphere of radius 10

inches. Find the difference between the area of this triangle and that of an

equiangular triangle having the same perimeter.

For item number 5 to 9, refer to the following table

Place Latitude Longitude


Cape of Good Hope
Module 2. Spherical Geometry 100

Chicago
Moscow
New York
Paris
Rio de Janeiro
San Francisco
Sydney

5. A ship sailed due east from New York to a point on the meridian of

near Portugal. Find the distance it would have saved if it had sailed along

the arc of a great circle.

6. A ship sails from New York to Cape of Good Hope along the arc of a great

circle. Find its course (direction) when it crosses the equator.

7. Find the area of the triangle whose vertices are New York, San Francisco

and Rio de Janeiro.

8. An airplane flies from New York to Chicago in 3 hours and 45 minutes.

What is the average rate of speed in statute miles per hour?

9. An airplane flew from Chicago to San Francisco at an average speed of

180 statute miles per hour. How long did the flight take?

10. Find all the Pythagorean triples with and .


Module 3. Conics 101

3
Conics
We are familiar with parabola, hyperbola, circle and ellipse during our high
school mathematics. The concepts, theorems and postulates that we had learned in high
school about these conics sections are dependable, that is, what is true about these conic
sections are still true at present. In this section, we will try to recall one of the familiar
topics related to modern geometry- Conics Section. Take note that we will be using a
more concise and axiomatic approach.

SELF-CHECK!!!

After finishing this module, you are expected to achieve the following
objectives
1. Describe Conics Sections.
2. Determine the Foci of an Ellipse and Hyperbola.
3. Identify the Eccentricity and Directrix.
4. Determine the Tangent Lines.
5. Describe Properties of Conics Sections.
6. Review Exercises for Standard Equations for Smooth Curves.
7. Define LORAN Navigation.
8. Describe Kepler’s Law of Planetary Motion
Module 3. Conics 102

LEARNING RESOURCES
For further readings, you may refer to the following book;
George A. Jennings Modern Geometry With Applications
(Universitext)

Or watch video at www.youtube.com/moderngeometry

EXPLORE!!!!

We will now continue expounding our understanding about

Modern Geometry. We had already explored Euclidean Geometry in Module 1

and Spherical Geometry in Module 2. In this module we will try to review our

understanding about Conic Sections or simply Conics.

CONICS SECTIONS

All throughout these module, I will adapt the discussion made by George

A. Jennings in his book entitled Modern Geometry with Applications. If you take

two intersecting lines and in and revolve around then the rotating line

will sweep out a right circular cone (if the lines are perpendicular, the rotating

lines sweep out plane). All line that is obtained by rotating around is called a

generator of the cone. The line is called the cone’s axis, the point where

and intersect is the cone’s vertex, and we call the angle between and the

“vertex half – angle” of the cone ( ).


Module 3. Conics 103

Figure 1. Right Circular Cone

A conic section or simply “a conic” is obtained by intersecting a right circular cone

with a plane. For this entire module, we will use the following notation:

a right circular cone with vertex at

a plane

, a conic section

the vertex half – angle of

the angle between and the axis of

Figure 2. Side View

The overall shape of the conic depends on two things: the relative sizes of

the angles and , and whether or not . is

a. Smooth if ,

b. Degenerate or, equivalently, singular if , and it is,

i. Elliptic if

ii. Parabolic if
Module 3. Conics 104

iii. Hyperbolic if

Thus, there are 6 types of conics. A singular hyperbolic consists of two lines

intersecting at , a singular parabolic is a single line, and a singular elliptic is a

single point.

Figure 3. Smooth Conics

Figure 4.Singular Conics

DISCUSSION BOARD
We will now try to interact with each other in a two way
Module 3. Conics 105

process at a least possible way! I will post a question/s and try to answer it on

your own.

1. How do we describe Conic Section?

2. Can you reiterate the six types of conics?

3. What particular subject/s that you had already taken that you find helpful

in learning this topic?

POST COMPETENCY CHECKLISTS


Let us try to determine if you learned something from our topic. Answer the
following Self-Assessment Question.

SELF – ASSESSMENT QUESTION 3.1


1. Differentiate Smooth Conics from Singular Conics.
2. The common sundial is consists of a horizontal plane and a vertical
pointer or gnomen. As the sun travels across the sky it causes the
shadow of the gnomen to move across the plane. The position of the
shadow tells the observer of time of day.
The tip of the moving shadow traces out a curve whose shape
depends on two things: 1) the latitude of the place where the sundial is
located (this determines the angle between the gnomen and the
earth’s axis of revolution), and 2) the season (this determines the
position of the earth in its’ orbit around the sun, and hence the angle
between the earth’s axis and the rays of the light that come from the
sun.
What curve is traced out by the tip of the shadow on the sundial,
and how is its shape affected by the season and the latitude? What will
its shape be if the sundial is located,
a. At the north pole,
b. At latitude north,
c. On the equator,

On,
Module 3. Conics 106

i. The summer solstice (the day when the sun is highest in the
sky),
ii. The winter solstice (the day when the sun is lowest in the sky)
iii. The equinox (the day when the sun passes over the equator)?

Figure 5. Four Seasons


3. If you will be given the opportunity to teach this topic, how are you
going to do it?

EXPLORE!!!!

We had examined the introductory part of this module, the

Conic Section or simply Conics. I assumed that we set ourselves ready for the

continuation of our lesson. We will continue on Foci of Ellipses and Hyperbolas.

FOCI OF ELLIPSES AND HYPERBOLAS

The properties relative to focus (foci in plural form) of conics are

necessary in practical applications. They had been studied since the time of

Greeks by Appolonius of Perga (262 – 190 B.C). In this section, we will adapt the

modern line of argument due to the Belgian mathematician G. P. Dandelin in

1822.

Dandelin’s constructions use spheres that are inscribed in the cone and

also tangent to the plane . If the conic is an ellipse or a hyperbola exactly two
Module 3. Conics 107

inscribed spheres are tangent to , but if the conic is a parabola only one

inscribed sphere has this property.

Let be a sphere that is inscribed in and tangent to . If there are two

such spheres call the other one . A point where or is tangent to is a

focus of the conic . Set

and .

PROPOSITION 1. If is an ellipse then is the same for every point

- constant

Figure 6. Ellipse

Proof. Let be an arbitrary point on the ellipse. ⃡ is tangent to at and ⃡

is tangent to at since and are tangent to at these points. Let and

be the circles where and intersect the cone, and set

⃡ and

⃡ .

Since and are tangent to along and , it follows that ⃡ is

tangent to at and ⃡ is tangent to at . Hence,

and .

Therefore,

.
Module 3. Conics 108

But , which is the distance between the circles and .

Since the distance between and does not depend on , it follows that

is the same for all . This completes the proof.

Figure 7. Dandelin’s construction of Ellipse

PROPOSITION 2. If is a hyperbola then | | is the same for all .

| | = constant

Figure 8. Hyperbola

DISCUSSION BOARD
We will now try to interact with each other in a two way

process at a least possible way! I will post a question/s and try to answer it on

your own.
Module 3. Conics 109

1. How do we describe the foci of ellipse and hyperbola?

2. What tips/advise could you give to your fellow students who were taking

the same course to make this topic easier to learn?

POST COMPETENCY CHECKLISTS


Let us try to determine if you learned something from our topic. Answer the
following Self-Assessment Question.

SELF – ASSESSMENT QUESTION 3.2


1. Prove Proposition 2 by modifying Proposition 1.
2. Show how to construct sphere(s) that are inscribed in a cone and
tangent to a plane by revolving inscribed circles around the axis of the
cone.

EXPLORE!!!!

We had explored the Foci of Ellipses and Hyperbolas. I hope

that you are still okay at this moment. Let us try to continue on Eccentricity and

Directrix.

ECCENTRICITY AND DIRECTRIX; THE FOCUS OF A PARABOLA

As mentioned by Jennings, every noncircular smooth conic has at least

one directrix. Let be a sphere that is inscribed in and tangent to .

intersects in a circle. Every circle lies in a plane, so let be the plane that

contains . The line


Module 3. Conics 110

is a directrix of , and the focus is its’ associated focus.

and

Figure 9. Dandelin’s Construction of Directrix

If is a circle then is parallel to , so the directrix does not exist. The

eccentricity of the conic is the ratio

In particular,

if is a hyperbola

if is a parabola

if is a noncircular ellipse

if is a circle.

PROPOSITION 3. If is a noncircular smooth conic with eccentricity , directrix

, and associated focus , then

( )( )

for every point .


Module 3. Conics 111

Proof. Let be the plane containing , and let be an arbitrary point on .

Choose points and so that

and ̅̅̅̅ is perpendicular to ,

and ̅̅̅̅ is perpendicular to , and

̅̅̅̅ is parallel to the axis of the cone since both ̅̅̅̅ and the axis of the

cone are perpendicular to . Since ̅̅̅̅ is perpendicular to and lies in , it

follows that ̅̅̅̅ is perpendicular to . Since ̅̅̅̅ is also perpendicular to , it

follows that the plane ̅̅̅̅̅̅ is perpendicular to . Since lies in ,

the plane ̅̅̅̅̅̅ is perpendicular to .

Therefore,

because ̅̅̅̅ is parallel to the axis of the cone, and

for the same reason. Thus

But because ̅̅̅̅ and ̅̅̅̅ are tangent to at and , and

because ̅̅̅̅ is perpendicular to . Hence

Divide through by to complete the proof.

COROLLARY 1. If is a parabola with focus and directrix then

for all (1)

Proof. If is a parabola then .


Module 3. Conics 112

Figure 10. Parabola

DISCUSSION BOARD
We will now try to interact with each other in a two way

process at a least possible way! I will post a question/s and try to answer it on

your own.

1. How do we describe eccentricity and directrix?

2. What tips/advise could you give to your fellow students who were taking

the same course to make this topic easier to learn?

POST COMPETENCY CHECKLISTS


Let us try to determine if you learned something from our topic. Answer the
following Self-Assessment Question.

SELF – ASSESSMENT QUESTION 3.3


1. Construct your own example of parabola and ellipse. Identify the directrix

and eccentricity.

2. Show that every point on the curve traced out in Fgure 11 satisfies the

equation
Module 3. Conics 113

Figure 11. Apparatus for drawing parabola

EXPLORE!!!!

We had examined Eccentricity and Directrix. I am hoping that

you are doing well at this moment! If not, that’s fine! Learning might not happen

overnight. Just keep on reading and studying. Meanwhile, we will study Tangent

Lines.

TANGENT LINES

During our study in Analytic Geometry, we had examined tangent lines.

No line can intersect a smooth conic section in more than two points. It is evident

if we substitute the equation of the line ( ) into the equation of

the conic and solve for . The result will be a quadratic equation

for coordinate of the intersection. Such equations have at most two solutions.

Figure 12. Lines intersecting a Smooth Conic


Module 3. Conics 114

Lines that intersect the conic in exactly two points are secant lines. A line that

intersects the conic in only one point is either:

a. Tangent to the conic, or

b. A line that is parallel to one of its asymptotes if the conic is a hyperbola, or

c. A line that is parallel to its axis if the conic is a parabola.

PROPOSITION 4. Let be a smooth conic, a point on and a focus. If is

a parabola let be the directrix, otherwise let ̃ be the other focus,

a. If is an ellipse then the line that bisects the angle between the vectors

and is tangent to at .

b. If is a hyperbola then the line that bisects the angle between and ̃

is tangent to at .

c. If is a parabola let be the point such that ̅̅̅̅ is perpendicular to .

Then the line that bisects the angle between and is tangent to at

Proof. Let ⃡ be a line through , with and on the opposite sides of .

a. Assume that is an ellipse and let ̃ . By Proposition 1 there

exist a constant such that

̃ for all .

Since ̃ , it implies that ̃ is the shortest path from to ⃡

to ̃ . Hence,

̃ if ⃡ and .
Module 3. Conics 115

In other words, ⃡ bisects only at . This proves part (a).

Figure 13. Tangent to an Ellipse

b. Assume that is a hyperbola and ̃ . By Proposition 2, there

exist a constant such that | ̃| for all .

Let be the point on the ray such that ̃ . Then

| ̃ |, so

Since ⃡ bisects ̃ it follows that ⃡ is the perpendicular bisector of

the base of the isosceles triangle . Thus if is any other point

on ⃡ then ̃ also is isosceles triangle, in particular

̃.

Clearly, and since and are not

collinear. Hence,

Subtract and get , so

| | .

But, ̃ and , so

| ̃| (2)

for all in ⃡ .
Module 3. Conics 116

This almost proves that ⃡ is tangent to ; it only remains to

eliminate the possibility that ⃡ is parallel to an asymptotic line. The

hyperbola divides the plane into three connected open regions: a middle

region between the two branches of the hyperbola, and the two outer

regions, one containing and the other containing ̃ . Define a function

on the plane by ( ) ̃. on the hyperbola, and it is easy

to see that on the region containing , on the middle

region, and on the region containing ̃ . It was shown in (2) that

everywhere on ⃡ , so ⃡ never crosses over the hyperbola

into either of the outer regions. It follows that ⃡ is tangent to the

hyperbola.

Figure 14. Tangent to a Hyperbola

c. Assume is a parabola and . By Corollary 1

if and only if .

Since ̅̅̅̅ is perpendicular to , we have

so is an isosceles triangle. Let ⃡ . is an isosceles

triangle, in particular

If then ̅̅̅̅ is not perpendicular to , so . Hence,


Module 3. Conics 117

It follows that ⃡ intersects the parabola only at .

The parabola bounds two open, connected regions in the plane,

one containing and the other containing . Define a function by

( ) . on the region containing the focus and on

the region containing the directrix. Since everywhere on ⃡ , it

follows that ⃡ never crosses over the parabola from one region to the

other. Therefore, ⃡ is a tangent line.

Figure 15. Tangent to a Parabola

DISCUSSION BOARD
We will now try to interact with each other in a two way

process at a least possible way! I will post a question/s and try to answer it on

your own.

1. How do we describe tangent line?

2. Does it make a difference if a tangent line crosses an ellipse, a parabola

or a hyperbola?
Module 3. Conics 118

3. What tips/advise could you give to your fellow students who were taking

the same course to make this topic easier to learn?

POST COMPETENCY CHECKLISTS


Let us try to determine if you learned something from our topic. Answer the
following Self-Assessment Question.

SELF – ASSESSMENT QUESTION 3.4


1. Identify a real life situation/event where you can apply the concept of

tangent lines.

2. Prove that if an ellipse and a hyperbola have the same foci then they are

perpendicular to each other at each point where they intersect.

EXPLORE!!!!

We are done dealing with Tangent Lines. We will continue

examining Properties of Conics. Perhaps, some concepts were familiar to you

and some are not. So, let’s get back to work!

PROPERTIES OF CONICS

Like what I mentioned in the previous modules, we will again adapt the

discussion of George A. Jennings relative to this topic. Light and sound reflect off

smooth curved surfaces in the same direction as they would reflect off a plane

that is tangent to the surface, following the rule that the angle of incidence equals
Module 3. Conics 119

the angle of reflection. Because of the special relation between the foci of smooth

conics and their tangents, mirrors formed by revolving an ellipse or a hyperbola

around the line through its foci, or a parabola around its axis of symmetry, have

unique focusing properties that are useful in applications.

PARABOLIC MIRRORS

The light entering a parabolic mirror in a direction parallel to its axis of

symmetry will reflect into the focus of the parabola. since they are

vertical angles, and by Proposition 1. Hence, ,

which says that the angle of incidence equals the angle of reflection.

Figure 16.Parabolic Reflector

Reflecting telescopes use parabolic mirrors because the ability of parabolic

mirrors to gather a great deal of light into one spot enables astronomers to see

objects that otherwise too dim to detect. Parabolic mirrors are also used in solar

collectors, long distance microphones and receiving antennas.

Energy radiating out from the focus of a parabolic mirror reflects into a

beam that is parallel to the axis, which makes parabolic reflectors ideal for

constructing headlights, spotlights and directional transmitting antennas.

HYPERBOLIC MIRRORS

Light aimed at one focus of hyperbolic mirror reflects off the mirror toward
Module 3. Conics 120

the other focus. ̃ , by Proposition 1 and the congruence of

vertical angles.

Figure 17. Hyperbolic Reflector

Some reflecting telescopes use secondary hyperbolic mirror in addition to the

main parabolic reflector to redirect the light from the main focus to a more

convenient point. In figure 18, both the parabola and hyperbola have the same

focus ̃ . Light entering the parabolic mirror reflects toward ̃ ,then bounces off the

hyperbolic mirror and travels toward the other focus of the hyperbola.

Figure 18. Compound Parabolic – Hyperbolic Reflector

ELLIPTIC MIRRORS

Another consequence of Proposition 1 is that energy radiating out from

one focus of an elliptic reflector reflects toward the other focus. Dentist’s lamp

use elliptic reflectors to focus light at one spot in the patient’s mouth. The Capitol

building in Washington, D.C. contains an elliptically shaped “whispering gallery”,


Module 3. Conics 121

designed so that a whisper uttered at one focus can be heard at the other focus

but not in the other parts of the room.

Figure 19. A dentist’s elliptical lamp

DISCUSSION BOARD
We will now try to interact with each other in a two way

process at a least possible way! I will post a question/s and try to answer it on

your own.

1. How do we describe properties of conics?

2. What tips/advise could you give to your fellow students who were taking

the same course to make this topic easier to learn?

POST COMPETENCY CHECKLISTS


Let us try to determine if you learned something from our topic. Answer the
following Self-Assessment Question.

SELF – ASSESSMENT QUESTION 3.5


1. Identify a real life situation/event where you can apply the properties of

conics. Draw and identify what properties of conics were applied.


Module 3. Conics 122

EXPLORE!!!!

So far we had examined Properties of Conics. We will continue

on Review Exercises for Standard Equations for Smooth Conics. We will go

through several exercises related to smooth conics.

REVIEW EXERCISES FOR STANDARD EQUATIONS FOR SMOOTH CONICS

In this section, I will present several exercises adapted from the book of

George A. Jennings. Exercises were classified according to the relevant topics.

EQUATION OF A PARABOLA

Let be the parabola in the plane with focus ( ) and directrix

the line . Show that satisfies the equation

EQUATIONS FOR HYPERBOLAS AND ELLIPSES

Let be a hyperbola or ellipse in the plane with foci ( ) on

the axis, .

a. Show that there are constants and such that

for all , (1)

where,

b. Deduce that satisfies the equation

(2)

where ( ).
Module 3. Conics 123

c. If is a hyperbola, show that the asymptotes have slope .

d. Let be the eccentricity of the conic and the directrix associated with

the focus ( ). Show that

and

{( ) }

EQUATION FOR A SMOOTH CONIC IN POLAR COORDINATES

Let . Show that a smooth conic with focus ( ), associated

directrix the line , and eccentricity is parametrized in polar coordinates

by,

(3)

where .

DISCUSSION BOARD
We will now try to interact with each other in a two way

process at a least possible way! I will post a question/s and try to answer it on

your own.

1. How concepts in Analytic Geometry do apply to this topic?

2. What tips/advise could you give to your fellow students who were taking

the same course to make this topic easier to learn?


Module 3. Conics 124

POST COMPETENCY CHECKLISTS


Let us try to determine if you learned something from our topic. Answer the
following Self-Assessment Question.

SELF – ASSESSMENT QUESTION 3.6


Choose at least one (1) problem from the given Review Exercises for Standard

Equations for Smooth Conics. Answer the problem that you choose and submit

your answers with the corresponding solutions.

EXPLORE!!!!

We had exposed several Review Exercises related to

equations of conic. For the meantime we will continue on LORAN Navigation.

LORAN NAVIGATION

The Long Range Navigation (LORAN) system enables the navigator of a

ship or airplane to find its position without relying on visible landmarks. Radio

stations at and simultaneously broadcast signals that are received by the

ship at . The navigator measures the interval

between the time when he receives the signal sent by , and the time

when he receives the signal sent by . If is the total amount of time it takes

the signal from to reach the ship, and is the total amount of time it takes the
Module 3. Conics 125

signal from to reach the ship, then the difference between the distance from

the ship to and the distance from the ship to is

where is the speed of light.

The navigator cannot measure and directly without knowing

precisely when the signals were sent. But he or she can accurately measure the

difference between the times when the signals were received, which is

enough to determine that the ship lies at some point on the hyperbola whose

equation is

| | .

The navigator can locate the ship’s location exactly if he receives signals from

three stations . Each pair of stations gives a hyperbola containing the

ship, so its exact position must lie at the point where the three hyperbolas

intersect. The navigator could find this position on a map by plotting the three

hyperbolas and intersecting them, or by setting up coordinates and computing

their intersections algebraically. (In real life it would be necessary to correct for

the curvature of the earth, and to take into account the possibility that the radio

signals may have been reflected and other potential source of error.).

(distances are in miles)


Figure 20. Carson Bay and vicinity
Module 3. Conics 126

DISCUSSION BOARD
We will now try to interact with each other in a two way

process at a least possible way! I will post a question/s and try to answer it on

your own.

1. How do we describe LORAN system?

2. What tips/advise could you give to your fellow students who were taking

the same course to make this topic easier to learn?

POST COMPETENCY CHECKLISTS


Let us try to determine if you learned something from our topic. Answer the
following Self-Assessment Question.

SELF – ASSESSMENT QUESTION 3.7

1. An oil tanker heading for Carson Bay in dense fog receives signals that

were broadcast simultaneously from three radio stations , located

on the line with in the middle. If the ship receives the signal from

7.22 x sec. later than signal from , where is the ship and in which

direction should the helmsman steer to avoid running aground? (The

speed of light is approximately 186,000 mi./sec)

EXPLORE!!!!
We had examined LORAN System that sailors used on their
Module 3. Conics 127

journey from one point to another. We are now about to tackle the last subtopic

of Module 3 the Kepler’s Law of Planetary Motion. Buckle up and fasten your

seatbelts as we travel at full speed in this section!

KEPLER’S LAW OF PLANETARY MOTION

As mentioned by Jennings, the elliptical shape of planetary orbits as

discovered by Johannes Kepler (1571 – 1630), through careful analysis of the

astronomical observations of Tycho Brahe (1546 – 1601). It is difficult to

appreciate fully the magnitude of Kepler’s achievement. Not only did he have to

calculate planetary orbits by hand from Tycho Brahe’s raw data, but he had to

correct for the fact that Brahe’s observations were taken from a moving platform

(the earth) which was also travelling along an unknown path. And he did all this

work at a time when most astronomers believed that the earth was immovably

fixed at the center of the universe, with all heavenly bodies travelling in

complicated paths around it.

Kepler’s Laws

1. Each planet travels in an elliptical orbit with one focus at the center of mass of

the system formed by the planet and the sun.

2. The vector pointing from this focus to the planet sweeps out equal areas in

equal interval of time.

3. The cube of the period of the orbit (that is, the cube of the length of the

planet’s “year”) is proportional to the square of the length of the orbit’s major

axis.
Module 3. Conics 128

Figure 21.Equal Areas in Equal Times

More generally, any object in orbit around the sun travels in an orbit that has the

shape of a conic section with one focus near the sun. Objects that follow closed

orbits travel in ellipses; objects that are travelling fast enough to escape from the

sun travel along hyperbolas or parabolas.

Figure 22.Orbits around a massive object

Kepler’s Laws provided evidence that enabled Sir Isaac Newton (1643 –

1727) to formulate and confirm his famous Laws of Motion and Universal Law of

Gravitation (the “inverse – square law” of gravitational force). In this section we

will use Newton’s Law to derive Kepler’s Law.

Newton’s Laws of Motion

1. A body that is experiencing no forces moves in a straight line at a constant

speed.

2. The force on a body that is the product of the body’s mass and its

acceleration (“ ”).
Module 3. Conics 129

3. When two bodies exert forces on each other, the forces are always equal in

magnitude and opposite in direction (“every action has an equal and opposite

reaction”).

Implicit in Newton’s Laws is the assumption that the motion of the body is

measured in an unaccelerated, or “inertial”, coordinate system. In an inertial

coordinate system, the and axes may move through space as time passes

but their acceleration is zero. One of the fundamental observations of Newtonian

physics is that the physical properties of objects are the same when they are

measured in an inertial coordinate system as they are in a coordinate system that

is at rest.

Consider an isolated system of two masses and , travelling freely

through space, affected by no forces except each other’s gravity. Let

the vector that points from 0 to (1)

the force on

the vector that points from 0 to

the force on

and let

| |, | |, | |, | |

be the magnitudes of these vectors. (Forces are represented by vectors since

they have both magnitude and direction).

The center of mass of two body system is the point

Newton’s Second Law of Motion says that


Module 3. Conics 130

(2)

where primes denotes derivatives with respect to . Newton’s Third Law

says that

(3)

Hence,

so the center of mass is not accelerating. Therefore to simplify our calculations

we may change to an inertial system of coordinates whose origin is located at the

center of mass. In these coordinates

(4)

With this assumption

(5) and

(6)

It remains only to calculate and .

According to Newtonian inverse square law of gravitation, the magnitude

of the gravitational force between the two bodies is inversely proportional to the

square of the distance separating the bodies and directly proportional to their

masses:

( )

(7)
( )
Module 3. Conics 131

Here is constant (the “gravitational constant”), and the second line in (7)

follows from the first equation by (1). The gravitational force on the planet

points from towards :

( ) (8)

To derive Kepler’s second law, consider the area swept out by as

varies over an infinitesimally small time interval . The infinitesimal area

swept out by during this time interval is approximately equal to the area of the

triangle whose sides are and , or one half of the area of the parallelogram

whose adjacent edges are and :

| |

where ( ) ( )

If is very small then

So

| |

with equality in the limit as . Thus the area swept out by changes at the

rate of

| |. (9)

Kepler’s second law asserts that is constant. To see this, differentiate

in (9)

( )
Module 3. Conics 132

( ) (10)

(the last equation comes from Newton’s second law). Equation (8) says that is

parallel to . Since it follows that

( )

Thus is a constant vector.

In particular, = | | is constant, which proves Kepler’s Second

Law.

Set

(11)

points from 0 to , and is perpendicular to . Therefore lies in the plane

that is perpendicular to and passes through the origin. Since is constant

this plane does not change with time, and hence it contains the entire orbit of .

Since the orbit of lies in a single plane we may set up coordinates in

such a way that the plane containing the orbit of the is the plane. Let ,

and be unit vectors pointing in the and directions, respectively.

Write,

( ) ( )

( ) (12)

where is the angle between and the – axis. Differentiate twice with respect

to :

( ) ( ) (13)
Module 3. Conics 133

And

( )( ) ( )( ) (14)

Then combine (10), (11), (12) to obtain

( ) .

In particular, is constant. Setting | |, we have

(15)

Where is a constant equal to twice the rate of change of the area swept out by

Differentiate to obtain . Plug this result into (14)

to obtain

( )( ).

Plug this into Newtons’ Second Law and combine with (3) and (4) to

obtain a second order differential equation governing the motion of :

( ) (16)
( )

To solve this equation we use the chain rule and (14) to replace derivatives with

respect to with derivatives with respect to .

( ) ( )

( ) ( )

( ) ( )

( ) ( )
Module 3. Conics 134

( )

Thus, by equation (16)

( )
( )

Multiply through by to get a second order differential equation in and ,

governing the orbit of .

( ) (17)
( )

Equation (17) can be rewritten in the following way,

( ) (18)
( )

a linear second order ordinary differential equation in ( ) and with constant

coefficients! Its general solution is

( )
( )

( )
(19)

where and are constants, and

( )
, and

(20)

Compare (19) with previous formulas in Conics, it says that the conic with focus

at the origin, parameter , eccentricity , and directrix parallel to the – axis is

given in polar coordinates by the equation


Module 3. Conics 135

Equation (19) describes a conic in the plane with focus at the origin; the

additional constant indicates that the directrix of the conic is tilted at an angle

relative to the - axis. This proves Kepler’s first law.

It remains to prove Kepler’s third law. If one writes as a function of ,

then as runs from to the change in is

By ( ) ( ) so the time it takes to make one full revolution, i.e. the

period, is

So

( )

The area inside the ellipse is

, where

The lengths of major and minor axis of an ellipse with eccentricity and

parameter are easily calculated using previous equations in Conics and using

the fact that the endpoints of the major axis occur when , and the
Module 3. Conics 136

endpoints of the minor axis occur where has a maximum or a

minimum, that is, when . One finds that

and

Thus, the area inside the orbit is

( )

( )

And the period is

( )

( )

( )

Where the last line follows form the definition of . This proves Kepler’s Third

Law.

The Second Body

The orbit of the second body is

Since they differ only by scale factor , the orbits have similar shapes and

equal periods.
Module 3. Conics 137

DISCUSSION BOARD
We will now try to interact with each other in a two way

process at a least possible way! I will post a question/s and try to answer it on

your own.

1. How do we describe Kepler’s Law of Planetary Motion?

2. What are the implications of these laws in our daily living?

3. What tips/advise could you give to your fellow students who were taking

the same course to make this topic easier to learn?

POST COMPETENCY CHECKLISTS


Let us try to determine if you learned something from our topic. Answer the
following Self-Assessment Question.

SELF – ASSESSMENT QUESTION 3.8

(Adapted from Jennings). Pretend that you live at the time of Newton and you
know about Kepler’s Laws. Your aim is to use them to measure the gravitational
force in two – body system. Let be as defined in the
previous equations mentioned in this section. Put the origin at the center of mass
of the two – body system and treat the gravitational forces and as
unknowns.
a. From Kepler’s Second Law and the fact that the orbit of lies in a plane,
deduce that always points directly toward (or away from) . (Hint: Use
Newton’s Second Law of Motion ( )).
b. Deduce the inverse square law
Module 3. Conics 138

Where is a constant, form Kepler’s First Law. (Hint: Let be polar


coordinates in the plane containing orbit).

ASSIGNMENT
1. Locate the focus, find the equation of the directrix and sketch the curve of the
parabola .
2. Find the lengths of the major axis and minor axes, the latus rectum, the
eccentricity, the coordinates of the foci and vertices and the directrices of the
ellipse .
3. Find the center, vertices, foci and asymptotes of the hyperbola whose
equation is . Sketch the graph.
4. The cable of a horizontal suspension bridge is supported by two towers 150 ft
apart and 50 ft high. If the cable is 5 ft above the floor of the bridge at the
center, find the equation of the parabola using the midpoint of the bridge as
the origin.
5. The two towers of a suspension bridge are 300 meters and extended 80
meters above the road surface. If the cable (in the shape of parabola) is
tangent to the center of the bridge, find the height of the cable above the road
at 50 meters from the center of the bridge.
6. Assume that water is issuing from the end of a horizontal pipe, 25 ft above the
ground, describe a parabolic curve, the vertex of the parabola being at the
end of the pipe. If at point 8 ft below the line of the pipe, the flow of water has
curved outward 10 ft beyond a vertical line though the end of the pipe, how far
beyond this vertical line will the water strike the ground?
7. The arch of an underpass is a semi ellipse 80 ft wide and 30 ft high. Find the
clearance at the edge of a lane if the edge is 30 ft from the middle.
8. The earth’s orbit is an ellipse with the sun at one focus. The length of the
major axis is 186,000,000 miles and the eccentricity is 0.0167. Find the
distance from the ends of the major axis to the sun. These are the greatest
and the least distances from the earth and the sun.

The Energy of a Two Body System


The total energy of the Keplerian two body system is

where
Module 3. Conics 139

| | and | |

are the kinetic energies of the bodies and

is the potential energy of the system. is the constant representing the amount
of energy system would have if the bodies were infinitely far apart and were
travelling at zero velocity. Using little algebra, one can show that

( ).
( )

9. If the orbit is closed, show that

( )
where is the length of the major axis.
10. A 1,000 kilogram satellite is launched into an elliptical orbit whose highest and
lowest points are, respectively, 35,000 km and 15,000 km above the earth’s
surface. How much extra energy would be required to boost the satellite into
a circular orbit 27,000 km high? (Hint: If is the mass of the satellite and is
the mass of the earth then . To compute use Newton’s Second
Law of Motion, the inverse square law, and the fact that the acceleration due
to gravity at the earth’s surface is about . The mean radius of the
earth is about 6,371 km.)
Module 4. Projective Geometry 140

4
Projective Geometry
We are familiar with perspectives, sketching and solving during our high school
mathematics. The concepts, theorems and postulates that we had learned in high school
about Projective Geometry are dependable, that is, what is true about these concepts are
still true at present. In this section, we will try to explore one of the unfamiliar topics
related to modern geometry- Projective Geometry. Take note that we will be using a
more concise and axiomatic approach.

SELF-CHECK!!!

After finishing this module, you are expected to achieve the following
objectives
1. Describe Perspective Drawing.
2. Define Projective Space.
3. Describe Desargues Theorem.
4. Determine Cross Ratio.
5. Describe Projections in Coordinates
6. Determine Homogenous Coordinates and Duality.
7. Describe Homogenous Polynomials, Algebraic Curves.
8. Identify Tangents.
9. Determine Dual Curves.
10.Describe Pascal’s and Brianchon’s Theorems.
Module 4. Projective Geometry 141

LEARNING RESOURCES
For further readings, you may refer to the following book;
George A. Jennings Modern Geometry With Applications
(Universitext)

Or watch video at www.youtube.com/moderngeometry

EXPLORE!!!!

We had explored Euclidean Geometry in Module 1, Spherical

Geometry in Module 2 and Conics in Module 3. In this module, we will study

another unfamiliar filed under Modern Geometry, the Projective Geometry. Again,

like what I mentioned in the previous modules, I do not own all the knowledge in

this universe so I have to ask for help from other individual who were experts in

this field. In this entire module, we will adapt the discussion from the book of

George A. Jennings entitled “Modern Geometry with Applications”

Projective Geometry was originally created by artists to analyse

perspectives during the Renaissance period. During the eighteenth and

nineteenth centuries it blossomed into a complete and independent field of

geometry. Recently it has provided the setting for the modern study of algebraic

equations, and has even played role in Physics in the mathematics of quantum

field theory.

PERSPECTIVE DRAWING

When you look at a scene your eye responds to the light rays it receives

from points in the scene. To make a correct perspective drawing the artist first
Module 4. Projective Geometry 142

projectivizes the scene by extending an imaginary line to his eye from each point

in the scene. He then projects the scene into a plane by intersecting the plane

with each of the imaginary lines. Taken together these intersections form an

image that looks just the same as the original scene to the artists eye.

Figure 1. Projecting a Scene into a Plane

Projective Geometry is the study of the properties of geometric figures that

are not altered by projections. There are two basic projections, namely

1. Central Projection. Given a point and a plane with , define a

projection function by the formula

( ) ⃡

Figure 2. Central Projection

for point such that ⃡ is not parallel to . is called projection from into

; is the center of the projection .


Module 4. Projective Geometry 143

2. Parallel Projection. Let be a nonzero vector and a plane that is not

parallel to . For each point let be the line through that is parallel to

. Define a projection function by the formula

( )

Figure 3. Parallel Projection

is called parallel projection into along the direction .

Parallel projection acts like central projection whose center is infinitely far away.

Some examples of properties that are preserved by projections are: the

property of being a point, the property of being a line, and the property of being a

conic section. Properties that are not preserved include length, the size of

angles, area and the property of being a circle.

Projectivization. From now on, we will imagine that the artist has only one eye,

and that is located at the origin, . A radial line or plane is one that passes

through . The projectivization of a scene is the set of all radial lines that pass

through the points in the scene, together with all radial lines that are

infinitesimally close to lines passing through points in the scene (nobody’s eye is

sharp enough to distinguish between lines that are infinitesimally close to each

other.)
Module 4. Projective Geometry 144

As it views a scene your eye does not respond directly to the objects in

the scene. Instead it responds to a projectivization of the scene, namely the

projectivization that consists of all the light rays that travel along lines from points

in the scene to your eye. This fact has important consequences:

1. Radial lines looks like points, and radial planes looks like lines because

they are being viewed “edge on” by the eye at the origin

2. Radial dimensions are lost because radial lines looks like points.

3. Non – radial lines acquire an extra “point at infinity”. The projectivization

of a non-radial line is the set of radial lines in the plane ̅̅̅̅ that connects

with the eye. Only one radial line in the plane ̅̅̅̅ does not connect a point on

to the eye. That one exception is the radial line that is parallel to . We shall

call this exception line , the “point at infinity” on . To the eye appears to

be a point at infinity on , because it is the limit of lines ⃡ connecting the eye

to points as approaches infinity.

Figure 4. A point at infinity

4. Non – radial planes acquire an extra “line at infinity”. Let be a non –

radial plane. As goes to infinity the line ⃡ tends towards the radial

plane that is parallel to . We call this plane , the line at infinity of .

{ ⃡ }.
Module 4. Projective Geometry 145

To the eye, points in look like they lie “at infinity” on the horizon of .

Figure 5. is a “line at infinity”

As a general rule any figure that extends off to infinity will acquire extra

“points at infinity” when it is projectivized.

Vanishing Points

A perspective drawing is created by intersecting the projectivization of a

scene with a plane. The plane, which shall call the viewplane, is the artist’s

canvass. The image of the scene is the intersection of the projectivization of

the scene with the viewplane. A vanishing point is an image of a point at

infinity.

Figure 6. Parallel lines have the same vanishing points

Parallel lines are parallel to the same radial line, so they have the same

point at infinity. Therefore, the images of parallel lines all pass through the

same vanishing point. The only exception to this rule occurs when the lines
Module 4. Projective Geometry 146

are all parallel to the viewplane. In that case their point at infinity does not

intersect the viewplane, so the lines have no vanishing points and their

images are parallel. (Of course their common point at infinity is still visible to

the eye, but it does not appear in the picture.)

A plane’s horizon is the image of its line at infinity. If the plane contains

some parallel lines then their common vanishing point is on the plane’s

horizon.

The next figures show the four views of a rectangular box. The first is a

“three point perspective” all three vanishing points are present in the

viewplane.

Figure 7. Three Point Perspectives

The second view is a “two point perspective” – only two vanishing points are

present and four edges have parallel images with no vanishing point.

Figure 8. Two Point Perspectives


Module 4. Projective Geometry 147

The third view is a “one point perspective”. The viewplane is parallel to a

whole side of the box.

Figure 9.One Point Perspective

It is impossible to have a true “zero point perspective” drawing of a

rectangular box since the viewplane cannot be parallel to all its edges at

once. However if you move the artists eye off to infinity then central projection

through the artist’s eye becomes a parallel projection, the images of parallel

lines become parallel, and the image of the box has no vanishing points. The

fourth view is a parallel projection.

Figure 10. Parallel Projection

Parallel projections generally are preferred in technical and scientific

applications, even though they look less natural than true perspective

drawings, because there is simpler relationship between distances in the

drawing and distances in the original scene with parallel projections than

there is with perspective drawings. This makes parallel projections easier to

create, and simplifies the task of calculating the exact measurements of the

original object from data that are given in drawings.


Module 4. Projective Geometry 148

DISCUSSION BOARD
We will now try to interact with each other in a two way

process at a least possible way! I will post a question/s and try to answer it on

your own.

1. How do we describe perspectives?

2. Can you reiterate the four views mentioned and their short description?

3. What tips/advise could you give to your fellow students who were taking

the same course to make this topic easier to learn?

POST COMPETENCY CHECKLISTS


Let us try to determine if you learned something from our topic. Answer the
following Self-Assessment Question.

SELF – ASSESSMENT QUESTION 4.1


1. If you are looking at a drawing of a rectangular box in there point
perspective, where should you place your eye so that the picture will look
the same to you as the original box did to the artist? Let and be
the three vanishing points. Show that you should put your eye at a point
such that the lines ⃡ ⃡ and ⃡ are all perpendicular to each other.
Let be the sphere with diameter ̅̅̅̅̅̅ , the sphere with diameter ̅̅̅̅̅̅,
and the sphere with diameter ̅̅̅̅̅̅. Show that . There
are two points in this intersection, one on each side of the viewplane.
Where should you put your eye if the box is drawn in two point
perspective? In one point perspective?
2. If you will be given the opportunity to teach this topic, how are you going to
do it?
Module 4. Projective Geometry 149

EXPLORE!!!!

In the previous section, we had examined the Perspective

Drawing. I know that your talent in drawing will be of greater use in that section!

For the meantime, we will continue to study another concept related to Projective

Geometry the – Projective Space.

PROJECTIVE SPACE

I will present the definition here about Projective Space exactly the same

manner as how it was presented by Jennings on his book.

Definition 4.1. A projective point (notation:𝑃𝑜 ) is a radial


line. A projective line (notation:𝑃 ) is the set of all radial
lines in a radial plane. A projective plane (notation:𝑃 ) is
the set of all radial lines in a radial three dimensional space.

Generalizing the above statements, there are two equivalent definitions for

projective space.

Definition 4.2. An 𝑛 – dimensional projective space 𝑃𝑛 is


the set of all radial lines in 𝑅𝑛+

Definition 4.3. An 𝑛 – dimensional projective space 𝑃𝑛 is


obtained by starting with 𝑅𝑛 and completing it by adding on
its “points at infinity”

To see why the two directions are equivalent, recall that


Module 4. Projective Geometry 150

+ *( ) +

+
Regard as the set of the points in with coordinate equal to 1:

*( ) +

+
a kind of viewplane in .

Set ( ). Every point ( ) ( ) in lies on exactly

one radial line, namely, the line ( ) consisting of all scalar multiples of the

vector ( ):

( ) *( ) +

Radial lines

( ) *( ) +

whose points have components equal to zero, are parallel to ; they

+
represent “points at infinity” on . Thus, every radial line in can be

matched up either with a point ( ) in or with a point at infinity on .

This produces a one – to – one correspondence

* + + * +

Figure 11. * +

which shows that Definition 4.2 and 4.3 are equivalent.


Module 4. Projective Geometry 151

The fact that projective spaces contain “points at infinity” is an important

difference between projective spaces and Euclidean spaces. Nevertheless two

kinds of space look the same to the eye, so they usually are depicted in the same

way in diagrams.

A theorem in projective space can be interpreted as a theorem about a

Euclidean space of the same dimension, provided that lines meeting at a point at

infinity are interpreted as parallel lines, planes meeting along a line at infinity are

interpreted as parallel planes, and so on.

We need to note that in projective space all points look exactly the same,

all lines look exactly the same, and all planes look exactly the same. In other

words there is nothing special about points, lines, or planes at infinity. This

means that any point, line or plane in can be regarded as a point, line or plane

at infinity, provided that this is done in a consistent way: the line at infinity in

must contain all points at infinity, the plane at infinity in must contain all the

lines at infinity, and so on.

To avoid confusion, from now on we will drop the word “projective”

wherever possible, and call projective points, projective lines and projective

planes simply “points”, “lines” and “planes”.

PROPOSITION 1.

1. Every pair of points in lies on exactly one line.

2. Every pair of lines in intersects in exactly one point.

3. Every pair of planes in intersects in exactly one line.


Module 4. Projective Geometry 152

+
Proof of 1. According to Definition 2, a pair of radial lines in lies in exactly

one radial plane. This is clear because any pair of intersecting lines lies in a

unique plane.

Figure 12. Two points determine a Line

Proof of 2. Again using Definition 2, two radial planes in must intersect in

exactly one radial line. This is true because two radial planes cannot be parallel

as both contain the origin.

Figure 13. Two coplanar lines determine a point

Proof of 3. A rigorous proof of (3) requires one to think about intersecting three

dimensional radial subspaces of . Let and be planes in . Let be a

third plane in , different from the other two. Regard as plane at infinity, and

as a copy of . The set of all “finite points” on is a plane in

. Likewise the set of all finite points on is a plane in . These two

Euclidean planes either intersect in a Euclidean line or else they are parallel and

have the same line at infinity. In either case it follows that and intersect in a

line when points at infinity are included.

Figure 14. Two planes in determines a line

Similar arguments establish the next proposition.


Module 4. Projective Geometry 153

PROPOSITION 2. Any three noncollinear points in lie in exactly one plane.

Three planes in must either intersect in exactly one point or else they contain

a common line. A line and a plane in intersect in exactly one point unless the

line lies in the plane. If two lines in do not lie in common plane then they are

skew (and do not intersect).

DISCUSSION BOARD
We will now try to interact with each other in a two way

process at a least possible way! I will post a question/s and try to answer it on

your own.

1. How do we describe Projective Space?

2. Differentiate Projective Space from Euclidean Space.

POST COMPETENCY CHECKLISTS


Let us try to determine if you learned something from our topic. Answer the
following Self-Assessment Question.

SELF – ASSESSMENT QUESTION 4.2


1. Identify at least one application of Projective Space in our daily living.
Include pictures or illustrations if any.
2. As a future teacher, if you will be given the chance to teach this topic, how
are you going to do it? Please explain.
Module 4. Projective Geometry 154

EXPLORE!!!!

So far, we had explored Projective Space all its related

concepts. I hope that you are still okay at this moment! We will continue

analysing Desargues’ Theorem.

DESARGUES’ THEOREM

In this section, we will study the theorem introduced by Girard Desargues

(1591 – 1661). According to him, a set of lines is coincident if all the lines

intersect at the same point. A triangle is the union of three intersecting but

noncoincident lines

⃡ ⃡ ⃡

Triangles and are in perspective if the lines ⃡ ⃡ ⃡ that join

corresponding vertices on the two triangles, are coincident.

THEOREM 1. If the triangles and are in perspective then the

points

⃡ ⃡ , ⃡ ⃡ , ⃡ ⃡

with their corresponding sides intersect, are collinear.

Proof. Let

⃡ ⃡ ⃡

Figure 15. Desargues Theorem


Module 4. Projective Geometry 155

Case 1. Suppose that and do not lie in the same plane. In this case

we must first check that the intersections ⃡ ⃡ ,⃡ ⃡ and ⃡ ⃡ are

nonempty. The fact that the triangles are in perspective implies that each of the

following pairs of lines is contained in a plane:

⃡ ⃡ ̅̅̅̅̅̅

⃡ ⃡ ̅̅̅̅̅̅

⃡ ⃡ ̅̅̅̅̅̅

Therefore each pair of lines must intersect somewhere, so the points and

do, in fact, exist.

Clearly

⃡ ⃡ ⃡ ̅̅̅̅̅̅

⃡ ⃡ ⃡ ̅̅̅̅̅̅̅̅̅

Hence,

̅̅̅̅̅̅ ̅̅̅̅̅̅̅̅̅

This proves Desargues Theorem in Case 1.

Case 2. Suppose that and do lie in the same plane.

Figure 16. Desargues Theorem


Module 4. Projective Geometry 156

If , and there is nothing to prove. Therefore without loss of

generality we may assume that the two triangles differ on at least one edge. Let

us assume ⃡ ⃡ .

Let

a plane containing

⃡ , with and all distinct

⃡ and ⃡ both lie in a plane ̅̅̅̅̅̅ , so we can define

⃡ ⃡

By construction ⃡ ⃡ ⃡ so and are in

perspective. Moreover they do not lie in the same plane, for is the only plane

that contains ⃡ and ⃡ , but does not contain or .

By Case1 it follows that

⃡ ⃡ , ⃡ ⃡ and are collinear.

Projections map lines to lines. Since projection from into maps to

and to while fixing every point in , it must map ⃡ ⃡ to ⃡

⃡ and ⃡ ⃡ to ⃡ ⃡ while leaving fixed. Therefore since

and are collinear, their images and must also be collinear. This

completes the proof for Desargues theorem.


Module 4. Projective Geometry 157

DISCUSSION BOARD
We will now try to interact with each other in a two way

process at a least possible way! I will post a question/s and try to answer it on

your own.

1. How do we describe Desargues Theorem?

2. What advise/tips could you give to your fellow students who were taking

the same course to make this topic easier to understand?

POST COMPETENCY CHECKLISTS


Let us try to determine if you learned something from our topic. Answer the
following Self-Assessment Question.

SELF – ASSESSMENT QUESTION 4.3


1. Lines that intersect at Remote Points. Desargues theorem enables one
to draw a set of lines that intersect a point located off the drawing paper.
Prove that the following construction produces a line that
contains the intersection of two given lines and and passes
through a given point .

Construction. Given two lines and and a point , draw three coincident
lines and with . Let

⃡ ⃡ ⃡ ⃡

⃡ ⃡
Module 4. Projective Geometry 158

Now set

(Hint: , and are in perspective). The three coincident lines


and may be parallel, that is they may meet at a point at infinity.

Figure 17. Lines intersecting at a remote point

EXPLORE!!!!

We are done dealing with Desargues Theorem and all other

related concepts. For the meantime we will continue on Cross Ratios. I advise

you to review your notes in Algebra and Trigonometry for that will be helpful in

dealing with these topics.

CROSS RATIOS

We cannot calculate the exact distances between objects in the scene

from data obtained by measuring a perspective drawing of the scene because

the drawing does not depict radial distances. However, using cross ratio one can
Module 4. Projective Geometry 159

find the relative distances between there or more collinear points in the scene,

provided that the points are not on the same radial line and one knows the

location of the vanishing point of the line that contains them.

Notation: the ratio of two parallel vectors in Euclidean Space is

if and .

Definition 4.4 The Cross Ratios of Four Points on a


Euclidean Line. The cross ratio ,𝐴 𝐵 𝐶 𝐷 - of four distinct
points 𝐴 𝐵 𝐶 𝐷 on a Euclidean line is

𝐴𝐶 𝐵𝐷
,𝐴 𝐵 𝐶 𝐷 -
𝐴𝐷 𝐵𝐶

Example 4.1. The cross ratio of four numbers on a number line is

, -

If you rearrange the points their cross ratio may change. For example,

, - , -. The cross ratio is significant in projective geometry

because it is not changed by projections.

PROPOSITION 2. Let and be four points on a Euclidean line, and a

point that is not on that line. Then

1.

, - (1)

2. Let be the projections, respectively of from onto

another line. Then


Module 4. Projective Geometry 160

, - , -

Figure 18. , - , -

It is important to keep track of the signs. The angles in part 1 of the Proposition 2

are oriented angles, with a direction of rotation chosen so that an angle

indicates a rotation between and carrying to . The sines of two of

these angles have the same sign if and only if the angles rotate in the same

direction.

Proof. Taking into account the orientations of the angles, it is easy to check that

the left and right sides of equation in Part 1 have the same signs. It remains to

show that their magnitudes are equal.

The magnitude of their cross ratio is

|, -|

|, -|

( )( ) ( )( )
|, -| | || |
( )( ) ( )( )

|, -| | |

This proves part 1 of the Proposition.


Module 4. Projective Geometry 161

Part 2 is an immediate consequence of part 1 since and so on.

This completes the proof.

If one of the points , - in Definition 4.4 is a point at infinity, we can

still compute the cross ratio by taking a limit. For example, if , then

, - , -

and so on. The same result is obtained simply by defining and .

Example 4.2. If are numbers on a number line,

, -

, -

The conclusion of part 2 of the Proposition remains true if is a point

at infinity. In particular if is a point at infinity then “projection from ” is parallel

projection, and the proof of part 2 is a simple application of similar triangles.

The next Corollary shows how to compute the ratio of distances between

points in a scene by using the cross ratio of their images in a perspective

drawing.

COROLLARY 1. Let be three points on a Euclidean line, let be

their images in a perspective drawing and let be the vanishing point of the line.

Then

, -

Proof. By Proposition 2

, - , -
Module 4. Projective Geometry 162

, -

Figure 19. Equal lengths

Definition 4.5 The Cross Ratios of Four Points on a


Projective Line. Let 𝑃 𝑃 𝑃 𝑃 be four points on a
projective line, let 𝐻 be a Euclidean viewplane, and let
𝑃 𝑃 𝐻,,,, 𝑃 𝑃 𝐻 be the images of 𝑃 𝑃 in 𝐻.
Then,
,𝑃 𝑃 𝑃 𝑃 - ,𝑃 𝑃 𝑃 𝑃 -

This definition would be useless if intersecting with different planes

gave different cross ratios, but part 2 of Proposition 2 guarantees that this never

happens.

COROLLARY 2. In projective space, if are four points on a line ,

and are their images under a central projection mapping to another line

, then

, - , -
Module 4. Projective Geometry 163

Proof. If you intersect everything with a viewplane then Corollary 2 becomes part

2 of Proposition 2.

Definition 4.6 The Cross Ratios of Four Coincident Lines


in a Plane. Let 𝐿 𝐿 𝐿 𝐿 be four coincident lines in a
plane. If 𝑃 𝐿 , 𝑃 𝐿 ,𝑃 𝐿 and 𝑃 𝐿 are any four
collinear points, define
,𝐿 𝐿 𝐿 𝐿 - ,𝑃 𝑃 𝑃 𝑃 -

This definition applies in both projective and Euclidean spaces. In either case,

part 2 of Proposition 2 guarantees that the cross ratio of the four lines is the

same regardless of the choice of the points .

PROPOSITION 3. Cross ratios of lines are not changed by projections. In or

let be four coincident lines in a plane . If is another plane and

is a central projection from to , then

, ( ) ( ) ( ) ( )- , -

Proof. Let be a line in . For each set . Then,

, - , -

, - , ( ) ( ) ( ) ( )-

, - , ( ) ( ) ( ) ( )-

Figure 20. Cross ratios are not changed by projections


Module 4. Projective Geometry 164

A conic in the projective plane is the projectivization of a conic in the Euclidean

plane. The projective conic is “smooth” if the corresponding Euclidean conic is

smooth.

PROPOSITION 4. Let be four points on a smooth conic . Then for all

[⃡ ⃡ ⃡ ⃡ ] [⃡ ⃡ ⃡ ⃡ ]

Proof. Clearly it is enough to prove the proposition for a Euclidean cone, for

every projective cone can be made into a Euclidean cone by intersecting it with a

viewplane.

Every smooth Euclidean conic is a section of right circular cone. Let be

the vertex of the cone. If you project from into a plane that is perpendicular to

the axis of the cone the image of will be a circle, and the images of the lines

⃡ ,,,,⃡ will be lines through that circle. Since cross ratios are not changed by

projections the proposition is true for if and only if it is true for the circle.

Therefore it is enough to prove the proposition in case is a circle.

Assume is a circle,

[⃡ ⃡ ⃡ ⃡ ]

and

[⃡ ⃡ ⃡ ⃡ ]

If is a circle,

,
Module 4. Projective Geometry 165

It follows immediately that

[⃡ ⃡ ⃡ ⃡ ] [⃡ ⃡ ⃡ ⃡ ]

This completes the proof of Proposition 4.

Figure 21. The cross ratios are equal

Definition 4.7 The Cross Ratios of Four Points on a


Smooth Conic. The cross ratio ,𝐴 𝐵 𝐶 𝐷 - of four points
𝐴 𝐵 𝐶 𝐷 on a smooth conic is defined by the formula

,𝐴 𝐵 𝐶 𝐷 - ⃡ ⃡𝑃𝐵 ⃡𝑃𝐶 ⃡𝑃𝐷 ] where 𝑃 is any point on 𝐾.


[𝑃𝐴

By previous propositions, , - is the same regardless of the point that is

used to compute it.

DISCUSSION BOARD
We will now try to interact with each other in a two way

process at a least possible way! I will post a question/s and try to answer it on

your own.

1. How do we describe Cross Ratio?

2. Can you reiterate the cross ratios mentioned previously?


Module 4. Projective Geometry 166

3. What advise/tips could you give to your fellow students who were taking

the same course to make this topic easier to understand?

POST COMPETENCY CHECKLISTS


Let us try to determine if you learned something from our topic. Answer the
following Self-Assessment Question.

SELF – ASSESSMENT QUESTION 4.4


1. Draw a scene showing a row of equally spaced houses along a street that

runs towards the horizon.

2. Draw a checkerboard that extends toward the horizon in all direction

3. Let be four points in a plane, no three of which lie on a line. The

complete quadrilateral is a quadrilateral ⃡ ⃡ ⃡ ⃡

together with its diagonals ⃡ and ⃡ . Let

⃡ ⃡ , ⃡ ⃡

⃡ ⃡ , ⃡ ⃡

Show that if one takes ⃡ to be the line at infinity then and are

vertices of a parallelogram in Euclidean plane ⃡ . Use this fact to

show that , - .

Figure 22. A complete quadrilateral


Module 4. Projective Geometry 167

EXPLORE!!!!
We had examined Cross Ratios and its several kinds. I hope

that you are not yet overwhelmed by a lot of theorems, corollaries and

propositions previously introduced to you. At this juncture, we will study

Projections in Coordinates.

PROJECTIONS IN COORDINATES

In this particular subtopic, ( ) are standard coordinates in .


Example 4.3. Given two parallel lines

* +

* +

contained in the plane

* +

project and from the origin into the viewplane

* +

Solution. Points ( ) and ( ) lie on the same radial line if and

only if

( ) ( )

for some scalar , that is, if and only if

, and .

The third equation says that ; using this the two equations become

and (2)
Module 4. Projective Geometry 168

( ) lies on if and only if . Hence ( ) lies on the projection of

if and only if . Multiplying through by to clear the fractions, we get

the equation for the projection of :

Similarly, the equation for the projection of is , i.e.

Figure 23. Images of parallel lines meeting at the horizon

The horizon of is the intersection of the viewplane with the plane ,

which is parallel to . Thus the horizon is the line in . The projections of

and meet at the common vanishing point, ( ) ( ), of and in

which lies on the horizon of .

Example 4.4. Project the parabolas

2 3 and

{ }

from the plane * + into the plane.


Module 4. Projective Geometry 169

Solution. By using formula (2) from previous example and plug into the formula

for , we obtain a formula ( )( ) for the projection of .

Multiplying by to clear the fractions, we have

To find out what this equation represents, complete the square then multiply by

four:

. / .

This is the equation of an ellipse in with center . /, minor axis 1 unit

long, parallel to the - axis, and major axis 2 units long, parallel to the – axis.

The ellipse is tangent to the horizon of at the point ( ), which is the

vanishing point of the axis of symmetry of the parabola . From the projective

point of view the parabola is simply an ellipse that is tangent to the horizon

because these curves look identical to an eye at the origin.

The image of is computed in similar way. We plug the formulas into the

formula for to get ( )( ) , then multiply through by to clear

fractions:

This is the equation of a parabola that is tangent to the horizon of at ( ).

The image of is a parabola instead of an ellipse because the radial line

through the point ( ) is parallel to the viewplane. As points on

approach ( ), their projections in the viewplane go off to infinity.


Module 4. Projective Geometry 170

Figure 24.Projecting a parabola onto an ellipse

DISCUSSION BOARD
We will now try to interact with each other in a two way

process at a least possible way! I will post a question/s and try to answer it on

your own.

1. How do we illustrate Projections in Coordinate?

2. What advise/tips could you give to your fellow students who were taking

the same course to make this topic easier to understand?

POST COMPETENCY CHECKLISTS


Let us try to determine if you learned something from our topic. Answer the
following Self-Assessment Question.

SELF – ASSESSMENT QUESTION 4.5


Project

a. The parabola

b. The circle

c. The hyperbola
Module 4. Projective Geometry 171

in the plane from the origin into plane. Give a formula for the

projected curve (circle, ellipse, parabola, etc.) it is, and locate the points,

(if any) where it intersects the horizon of the plane.

EXPLORE!!!!
We are done dealing with Projections in Coordinates. In the

next section, we try to examine Homogenous Coordinates and Duality. As a

piece of advice, it is much better if you have a short break before you deal with

this!

HOMOGENOUS COORDINATES AND DUALITY

I will directly present the definition introduced by Jennings on his book

Modern Geometry with Applications.

Definition 4.8 Homogenous Coordinates. The


homogenous coordinates of a radial line in 𝑅 are the
Euclidean coordinates ,𝑋 𝑌 𝑍- of any nonzero point on that
line.

Homogenous coordinates serve as coordinates for points on the projective plane.

Note that , - must be nonzero. Square brackets and capital letters are used

to distinguished homogenous coordinates from Cartesian coordinates.

There is an important difference between homogenous coordinates on

and Cartesian coordinates on , while Cartesian coordinate triples ( ) are

in one to one correspondence with points in , a single projective point has

infinitely many sets of homogenous coordinates.


Module 4. Projective Geometry 172

PROPOSITION 5. , - and , - are homogenous coordinates for the

same point in if and only if

, - , -

for some nonzero scalar .

Proof. ( ) and ( ) lie on the same radial line if and only if one of

them is a scalar multiple of the other.

Definition 4.9 The Dual Projective Plane. The Dual


projective Plane 𝑃 is the set of all lines in 𝑃 . A point in
𝑃 is a line in 𝑃

It turns out that is a projective plane just like . Recall that a line in is a

radial line in , that is, a line is the set of all points , - satisfying a

linear equation of the form

(3)

where are constants, at least one of them nonzero. One can think of

, - as a set of homogenous coordinates for the line. They are “homogenous”

because if is a nonzero scalar then , - and , - represent the same

line, for if and only if . Thus from a formal

algebraic point of view, is just a projective plane whose homogenous

coordinates are represented by letters at the beginning of the Alphabet instead of

letters at the end.


Module 4. Projective Geometry 173

Similarly, a point in is a line in . If we regard , - as

homogenous coordinates of a point in , then the point , - lies on the line

whose coefficients are , -. Thus , - becomes a line in .

Table 1. Duality

Algebraic Statement Interpretation Dual Interpretation

, - Point in Line in

, - Line in Point in

The point , - lies on The line , - contains

the line , - the point , -

The points , - and The lines , - and

and , - lie on the line , - intersect at the

, - point , -

The lines , - and The points , -

and , - intersect at the and , - lie on the

point , - line , -

The fact that there is no algebraic difference between and besides

the position of their coordinates in the Alphabet means that any statement about

projective planes that can be expressed in algebraic language has two

interpretations: one where “points” and “lines” are points and line in and a dual

interpretation where “points” are points in (lines in ) and “lines” are lines in

(points in )
Module 4. Projective Geometry 174

Example 4.5. The Theorem of Pappus (Pappus of Alexandria, circa 320 AD).

In , given points on a line , and on a line , set

⃡ , ⃡ ⃡ , ⃡ ⃡

Then are collinear.

Figure 25. Theorem of Pappus

The Dual Theorem of Pappus, says

In , given lines passing through a point , and lines

passing through a point , set

⃡( )( ), ⃡( )( ), ⃡( )( )

Then, are coincident.

Figure 26. Dual Theorem of Pappus


Module 4. Projective Geometry 175

DISCUSSION BOARD
We will now try to interact with each other in a two way

process at a least possible way! I will post a question/s and try to answer it on

your own.

1. How do we illustrate Homogenous Coordinates? Duality?

2. What advise/tips could you give to your fellow students who were taking

the same course to make this topic easier to understand?

POST COMPETENCY CHECKLISTS


Let us try to determine if you learned something from our topic. Answer the
following Self-Assessment Question.

SELF – ASSESSMENT QUESTION 4.6


1. Show that the projectivization of the line has homogenous

coordinates , - in

2. Prove both Pappus theorem and its dual by proving the dual theorem.

EXPLORE!!!!
We are done examining Homogenous Coordinates and

Duality. In this section, we will explore Homogenous Polynomials and Algebraic

Curve. Just like the previous sections, we will adapt the discussion made by

Jennings.
Module 4. Projective Geometry 176

HOMOGENOUS POLYNOMIALS AND ALGEBRAIC CURVES

Like what we learned in high school mathematics, we define polynomial in

very similar manner.

Homogenous Polynomials. Every polynomial is a sum of one or more

“monomials”. A monomial is a product of a constant and a finite number of

variables with nonnegative integer exponents. The degree of a nonzero

monomial is the sum of the exponents. The degree of a polynomial is the largest

of the degree of its monomials. A nonzero polynomial is homogenous if all of its

monomials have the same degree.

Example 4.6. is a monomial of degree .

is a polynomial of degree

is a homogenous polynomial of degree 4.

PROPOSITION 6. If ( ) is a homogenous polynomial of degree then

( ) ( )

for all scalars .

Proof. If ( ) is a monomial of degree , then

( ) ( ) ( ) ( )

( ) + +

( ) ( )

for every scalar . The same holds for since all of its terms have the same

degree.
Module 4. Projective Geometry 177

A similar statement with a similar proof holds for polynomials in any

number of variables.

COROLLARY 3. If is homogenous and ( ) at some nonzero point

( ) , then everywhere on the radial line through the point ( ).

Proof. The radial line consists of scalar multiples ( ) of ( ), so by

Proposition 6 ( ) ( ) , where is the degree of the

homogenous polynomial .

Algebraic Curves

An algebraic curve in the Euclidean plane is the set of points ( )

satisfying a polynomial equation

( ) .

where is a constant polynomial. Examples include the circle ,

the line , the parabola and the “figure eight” (

) .

An algebraic curve in the projective plane is the set of points , -

satisfying a homogenous polynomial equation

( )

where is a nonconstant homogenous polynomial. Examples include the

projectivized circle , the projectivized line and

the projectivized parabola .

Proposition 6 says that the locus *( ) ( ) + of a

homogenous polynomial equation is a union of radial lines. In other words, it is a


Module 4. Projective Geometry 178

kind of “generalized cone” in with vertex at the origin. To an eye at the origin it

looks like a curve.

Figure 27. Generalized cone, with radial tangent plane

Homogenization. Projectivization is the process of converting a Euclidean figure

into a projective figure by replacing each of its points by a radial line.

Homogenization is the algebraic analog of projectivization. To homogenize a

polynomial ( ) of degree , introduce new variables

, then replace each , by and multiply through by

to clear fractions.

To recover the original polynomial from the homogenous polynomial

replace each by and by 1.

Example 4.7. Let ( )

( ) ( ) ( )( )

Multiply through by to get the homogenous polynomial

( )

To convert back into set and :

( )

( ) ( )
Module 4. Projective Geometry 179

DISCUSSION BOARD
We will now try to interact with each other in a two way

process at a least possible way! I will post a question/s and try to answer it on

your own.

1. How do we describe Homogenous Polynomial? Algebraic Curve?

2. What advise/tips could you give to your fellow students who were taking

the same course to make this topic easier to understand?

POST COMPETENCY CHECKLISTS


Let us try to determine if you learned something from our topic. Answer the
following Self-Assessment Question.

SELF – ASSESSMENT QUESTION 4.7


1. Find a homogenous polynomial equation for the cone with vertex at the

origin that passes through the circle where the plane

intersects the sphere .

2. Homogenize the following polynomials

a. ( )

b. ( )

c. ( )

d. ( )
Module 4. Projective Geometry 180

EXPLORE!!!!
We are done exploring the Homogenous polynomials and

Algebraic Curves. We will now study Tangents. Our previous discussions in

Module 3 about tangent will be of great help to you.

TANGENT

The projectivization of a Euclidean curve is a cone, and the

projectivization of a line that is tangent to the curve is a plane that is tangent to

the cone. If , - is a point on the projectivized curve ( ) then

the equation of its tangent plane at is

( ) ( ) ( ) (4)

0 1 are the homogenous coordinates of the projective line that is tangent

to the projectivized curve.

Lines that are asymptotic to the original Euclidean curve become tangents

at points at infinity on the projectivized curve when they are projectivized.

If all of the partial derivatives of are zero at , then is a singular point.

A curve may have multiple tangent lines at singular points.

Figure 28.Tangents at a singular point


Module 4. Projective Geometry 181

DISCUSSION BOARD
We will now try to interact with each other in a two way

process at a least possible way! I will post a question/s and try to answer it on

your own.

1. How do we illustrate Tangents?

2. What advise/tips could you give to your fellow students who were taking

the same course to make this topic easier to understand?

POST COMPETENCY CHECKLISTS


Let us try to determine if you learned something from our topic. Answer the
following Self-Assessment Question.

SELF – ASSESSMENT QUESTION 4.8

1. Consider the hyperbola in the plane in . Find

homogenous equations for the projectivization of the hyperbola and for the

projectivizations of its asymptotic lines. Verify directly that the projectivized

asymptotic lines are tangent to the projectivized hyperbola where it

intersects the line at infinity.

2. Show that the projectivization of the parabola in the plane is

tangent to the line at infinity.


Module 4. Projective Geometry 182

EXPLORE!!!!
We are done studying Tangent. I hope that you are still okay at

this time. For the meantime, we will try to explore Dual Curves.

DUAL CURVES

For every curve there is a dual curve . A point on is a

line that is tangent to .

Definition 4.10 The Dual Curves.

𝐶 {𝑃 𝑃 𝑃 𝑖𝑠 𝑡𝑎𝑛𝑔𝑒𝑛𝑡 𝑡𝑜 𝐶}

The Dual of the Dual Curve. The dual of the dual curve consists of lines that

are tangent to the dual curve. Since a line in is a point in , is a curve in

PROPOSITION 7.

Proof. Suppose that and are smooth curves. For each and let

( ) line tangent to at

( ) line tangent to at

To prove the proposition it is enough to show that

( ) if and only if ( )

for arbitrary and .

Let ( ), ( ) where . By definition

( ) ⃡ (5)
Module 4. Projective Geometry 183

is the limit of the secant lines ⃡ as goes to on . Dualizing, ( )

becomes a point in ,

( ) ( ) ( ) ( ) ( ) .

Figure 29.Dual curves

The second equality follows from the fact that as . A similar

argument shows that ( ) . This proves Proposition 7.

The Equation of a Dual Curve. Although it is very rare, calculation of the

equation of the dual curve involves complicated algebra. The following example

will show how it can be done.

Example 4.8. Let be the projectivization of the Euclidean hyperbola

To compute the dual of we start by finding all the lines that are tangent to .

Let be the line whose equation is

is tangent to the hyperbola if and only if it intersect the hyperbola at only one

point and is not parallel to one of its asymptotes. To find where intersects the

hyperbola, plug the equation for into the equation for the hyperbola:
Module 4. Projective Geometry 184

( )

Multiply out:

Factor out:

. / . / . / (6)

Equation (6) takes the form , where

. / . / . / (7)

The asymptotic lines of the hyperbola have slope √ . If is not

parallel to an asymptotic line it follows that , so we can use the quadratic

formula to solve for , (solution is left for you to verify)

There is exactly one solution if and only if

Plugging in the previous equation, one finds that the line is tangent to

the hyperbola if and only if

(8)

Equation (8) determines a conic in the – plane, and is its projectivization.

PROPOSITION 8. The dual of a smooth conic in is a smooth conic in .

Proof. Use the method applied in Example 4.8, starting with the general equation

for a conic, then the rest was left for you to verify.
Module 4. Projective Geometry 185

DISCUSSION BOARD
We will now try to interact with each other in a two way

process at a least possible way! I will post a question/s and try to answer it on

your own.

1. How do we describe Dual Curves?

2. What advise/tips could you give to your fellow students who were taking

the same course to make this topic easier to understand?

POST COMPETENCY CHECKLISTS


Let us try to determine if you learned something from our topic. Answer the
following Self-Assessment Question.

SELF – ASSESSMENT QUESTION 4.9


1. Find an equation for the dual of the circle .

2. Find an equation for the dual of the parabola .

EXPLORE!!!!
We are done exploring Dual Curves. We are not about to

tackle the last subtopic for this module. I hope that you find it worthwhile dealing

with Projective Geometry. As we end with this module, it points out that we will

again start with a new one. Meanwhile, we will try to study Pascal’s and

Brianchon’s Theorem.
Module 4. Projective Geometry 186

PASCAL’S AND BRIANCHON’S THEOREM

A hexagon is the union of six lines in .


THEOREM 2. PASCAL’S THEOREM (Blaise Pascal , -)

If a hexagon is inscribed in a smooth conic, the intersections of opposite sides of

the hexagon are collinear.

In other words, if points lie in a smooth conic, then

⃡ ⃡ ⃡ ⃡ ⃡ ⃡

are collinear.

Figure 30. Pascal’s Theorem

Proof. Let ⃡ ⃡ and ⃡ ⃡ . Projecting from we see that

, - [⃡ ⃡ ⃡ ⃡ ]

by definition of cross ratio of four points on a conic. Also,

, - [⃡ ⃡ ⃡ ⃡ ]

by definition of cross ratio of four points on a line. Hence,

, - , - (9)

If we project from , it shows that

, - [⃡ ⃡ ⃡ ⃡ ]

and

, - [⃡ ⃡ ⃡ ⃡ ]
Module 4. Projective Geometry 187

So,

, - , -

Combining this with equation (9), we have

, - , - (10)

Now let ⃡ be the line at infinity, and regard the rest of as Euclidean plane. To

show that and are collinear it suffices to show that is a point at infinity, or

in other words that ⃡ is parallel to ⃡ in the Euclidean plane. Since and

are points at infinity, ⃡ is parallel to ⃡ and ⃡ is parallel to ⃡ in the

Euclidean plane. Also since and are points at infinity, then

Therefore, since each of the triangles has one side on ⃡ and another

side on ⃡ , it follows that these two triangles are similar. Hence ⃡ is parallel to

⃡ . In particular ⃡ ⃡ is a point at infinity, so it lies on ⃡ . This

completes the proof.

Figure 31

The next theorem was derived by dualizing Pascal’s theorem.


Module 4. Projective Geometry 188

THEOREM 3. BRIANCHON’S THEOREM (Charles Julien Brianchon)

If a hexagon is circumscribed around a smooth conic, the lines connecting

opposite vertices all pass through the same point.

In other words, given six lines tangent to a smooth conic

the lines,

⃡( )( )

⃡( )( )

⃡( )( )

all intersect at the same point.

Proof. Dualize Pascal’s theorem, using the fact that the dual of a smooth conic is

a smooth conic.

Figure 32. Brianchon’s Theorem

Tangents to a Smooth Conics, Revisited.

We had constructed tangents to a circle in Module 1. The construction in

the following claim enables us to construct tangents to any smooth conic.

CLAIM 1. Let be a smooth conic and a point outside . Draw lines

through , each intersecting in two points. Set

* +
Module 4. Projective Geometry 189

* +

* +

Let

⃡ ⃡

⃡ ⃡

Then ⃡ is tangent to at .

Figure 33. Tangent to a conic

DISCUSSION BOARD
We will now try to interact with each other in a two way

process at a least possible way! I will post a question/s and try to answer it on

your own.

1. How do we describe Pascal’s Theorem? Brianchon’s Theorem?

2. What advise/tips could you give to your fellow students who were taking

the same course to make this topic easier to understand?


Module 4. Projective Geometry 190

POST COMPETENCY CHECKLISTS


Let us try to determine if you learned something from our topic. Answer the
following Self-Assessment Question.

SELF – ASSESSMENT QUESTION 4.10


1. Prove Claim 1 in the following steps:

Set

⃡ ⃡

⃡ ⃡

⃡ ⃡

⃡ ⃡

Prove that all six points are collinear.

2. In #1, prove that if ⃡ ⃡ , so

ASSIGNMENT

1. Draw your dream house. Identify the perspective use. Identify the radial

points, lines and planes.

2. Surf the net and search about short biography of Gerard Desargues.

Identify also his other contributions in mathematics.

3. Find the cross ratio of the numbers 3, 7, 10, 15 in the number line.
Module 4. Projective Geometry 191

4. Project the parabola from the plane into plane.

5. Surf the net and search about short biography of Pappus. Identify also his

other contributions in mathematics.

6. Homogenize the function ( ) .

7. Solve the system of homogenous equations,

8. Let be the projectivization of the Euclidean hyperbola

find the dual of .

9. Surf the net and search about short biography of Blaise Pascal. Identify

also his other contributions in mathematics.

10. Surf the net and search about short biography of Charles Julien

Brianchon. Identify also his other contributions in mathematics.


Module 5. Special Relativity 192

5
Special Relativity
We are familiar with several scientific theories during our high school
mathematics. Theses scientific theories that we had learned in high school about are
dependable, that is, what is true about these theories are still true at present. In this
section, we will try to explore one of the unfamiliar topics related to modern geometry-
Special Relativity. Take note that we will be using a more concise and axiomatic
approach.

SELF-CHECK!!!

After finishing this module, you are expected to achieve the following
objectives
1. Define Spacetime.
2. Describe Galilean Transformation.
3. Identify the Failure of Galilean Transformation
4. Describe Lorentz Transformation
5. Describe Relativistic Addition of Vectors
6. Define Lorentz-FitzGerald Contraction
7. Describe Minkowski Geometry
8. Identify Why The Slowest Path is a Line
9. Determine Hyperbolic Angles and Velocity Addition Formulas
Module 5. Special Relativity 193

LEARNING RESOURCES
For further readings, you may refer to the following book;
George A. Jennings Modern Geometry With Applications
(Universitext)

Or watch video at www.youtube.com/moderngeometry

EXPLORE!!!!

We had discussed Euclidean Geometry in Module 1, Spherical

Geometry in Module 2 and Conics in Module 3 and Projective Geometry in

Module 4. Again, like what I mentioned in the previous modules, I do not own all

the knowledge in this universe so I have to ask for help from other individual who

were experts in this field. In this entire module, we will adapt the discussion from

the book of George A. Jennings entitled “Modern Geometry with Applications”

SPACETIME

Let us start with the statement of Albert Einstein, and it goes this way,

“The whole of science is nothing more than a refinement of everyday thinking”.

“Time is nature’s way of preventing everything from happening all at once”.

It is clear that the topic on this entire module was Einstein’s special

relativity theory and what it says about geometry of flat spacetime. This is not

difficult because it involves knowledge little beyond high school mathematics. A

spacetime is simple the mathematical version of a universe that has dimensions

both of space and time. A flat spacetime is a spacetime with no gravity, since
Module 5. Special Relativity 194

gravitation tends to “bend” a spacetime. Flat spacetimes are the simplest kind of

spacetimes; they stand in the same relation to curved spacetime as a flat

Euclidean plane does to a curved surface.

The difference of a flat spacetime from a Euclidean space is the existence

of a time dimension. In the two dimensional case there is just one space

dimension and one time dimension, we will focus on this case for it is the

simplest and yet it illustrates the key elements of the theory.

We will tackle lengths of vectors and angles between vectors in two

dimensional spacetime, and to have an analogue of the Pythagorean Theorem. It

is not surprising when these turn out to behave differently than lengths and

angles in Euclidean space. After all, what does it mean to speak of “the length of”

or “the angle between” vectors that are pointing forward in time?

Event and Worldliness

In mechanics one studies objects moving through space over intervals of

time. Motion along a line is represented by the graph of a function ( ) in the

plane, a two dimensional spacetime where represents time and ( )

represents the position of the object at time . The local slope of the graph

is the velocity of the particle.

Figure 1. A two – dimensional spacetime


Module 5. Special Relativity 195

An event is a point in a spcetime. A worldline is a curve ( ) that shows the

position of a particle as a function of time. The event at time in the life of the

particle occurs at the point ( ) in the spacetime. The plane has only one

space dimension, so worldliness in the plane represent the motion of particles

along a line. Motions of particles in higher dimensional spaces require higher

dimensional spacetimes. The worldline of a particle moving in the plane is

the graph of the function ( ) ( ( ) ( )) in a three – dimensional

spacetime; for each the event at the point ( ( ) ( )) is the event in the life of

the particle at time . The worldline of a particle moving in three – dimensional

space exists in a spacetime with four independent dimensions .

Nature does not equip her spacetimes with a set of coordinate axes. In

real life coordinates like and must be artificially defined and measured by

observers who live within the spacetime. Different observers may set up different

coordinate systems and disagree on the locations and times of events, so it is

important to find a way to relate these different measurements with each other.

The search for a solution to this problem leads to the Special Theory of Relativity.

Imagine two observers, and ̃ , living in a two dimensional spacetime.

Observer assigns coordinates ( ) to each event by measuring

the time (according to ) when the event occurred, and

the distance (according to ) between himself and the event at the moment

when it occurred.

Observer ̃ assigns coordinates, which he calls ( ̃ ̃ ), to each event

̃ the time (according to ̃ ) when the event occurred, and


Module 5. Special Relativity 196

̃ the distance (according to ̃ ) between himself and the event at the moment

when it occurred.

A Fundamental Problem. What is the relation between the coordinates ( )

and ( ̃ ̃ ) of two different observers in the same spacetime?

A basic difficulty arises from the fact that the two observers are likely to be

moving. Since their state of motion affects their measurements, each observer

needs to be able to measure his own motion in order to correct for it effects. But

without a natural system of coordinates to refer to, it is physically impossible for

those observers to measure, in any absolute sense, exactly where they are or

how they are moving, or even whether or not they are moving at all!

Imagine that both observers are coasting along in deep space. If an

observers turn on his rocket engine he will be pressed back in his seat. He

knows that he is moving because he feels the acceleration. If he is not

accelerating he will feel no motion even though he may be moving with great

speed. Though he can measure his position and velocity relative to the other

objects in the universe he cannot determine his absolute position or velocity

since the absolute positions and velocities of these other objects are also

unknown.

In the end one is forced to conclude that position and velocity are

physically meaningful only in relation to other objects. “Absolute position” and

“absolute velocity” are meaningless abstractions. As long as an observer is not

accelerating then as far as he or we can tell (if we are moving with him) he might

as well be standing still.


Module 5. Special Relativity 197

An inertial observer is an observer who is not accelerating. His or her

coordinates are inertial coordinates. The basic premise of relativity – that nature

does not come equipped with a special set coordinates and all motion is relative

– means that the universe looks the same to all inertial observers: if two inertial

observers do the same experiment then they will get the same results. This is:

The Principle of Relativity. The laws of Physics are the same in any inertial

coordinate system.

DISCUSSION BOARD
We will now try to interact with each other in a two way

process at a least possible way! I will post a question/s and try to answer it on

your own.

1. How do we define Spacetime?

2. Can you describe the Principle of Relativity?

3. How can you apply the Principle of Relativity to your daily living?

POST COMPETENCY CHECKLISTS


Let us try to determine if you learned something from our topic. Answer the
following Self-Assessment Question.

SELF – ASSESSMENT QUESTION 5.1

1. Describe the Spacetime involved in your daily living from the moment you

wake up until the time that you go to school. (Cite at least three events)
Module 5. Special Relativity 198

2. If you will be given the opportunity to teach this topic, how are you going to

do it?

EXPLORE!!!!

We are done talking about Spacetime and a little about

Principle of Relativity. For the meantime, we will continue exploring Galilean

Transformation.

GALILEAN TRANSFORMATIONS

From now on and ̃ will be inertial observers moving with a constant

relative velocity so that each observer sees the other moving away from him

with velocity . It was shown on the figure what spacetime looks like to in his

own system of coordinates.

Figure 2.Spacetime in coordinates

Since measures the distance from to an event, points where are

events on own worldline. The event ( ) is the event in the life of at time

,( ) is the event in the life of at ,( ) is the event in the life of at

time , etc. Taken together, the points on worldline form the – axis.
Module 5. Special Relativity 199

The – axis is the set of all events with . It is the snapshot of the

universe at time . Vertical lines .. are snapshots of the

universe at times .

̃ is travelling with a constant velocity away from , so his coordinate at time

is

(1)

where is a constant. The line is ̃ worldline expressed in

coordinates; its slope is the velocity of ̃ relative to .

Before Einstein, physicists assumed that and ̃ would get the same

result whenever they measured the interval of time between two events. It follows

that the time of clock can differ from the time on ̃ clock only by a constant,

̃ (2)

One cannot expect and ̃ to agree on the positions of events that occur

at different times because each observer measures position on his own ruler,

which moves with him to the other observer. For the same reason, a person

standing on the side of a road would not measure the length of a moving car by

comparing the position of its rear bumper at one time with the position of its front

bumper one minute later.

Nevertheless it was assumed that the observers would agree on the

distances between simultaneous events. To measure the length of the moving

car, you compare the positions of its front and rear bumpers at the same time.
Module 5. Special Relativity 200

Let be an event with ̃ coordinates ( ̃ ̃ ). is ̃ units of distance away

from ̃ when ̃ clock reads . At that moment, clock reads ̃ . At

time the distance from to ̃ is ( ) and the distance from ̃ to is

̃. Both observers agree on these measurements since they were made

simultaneously, so we can combine them to find distance at that moment from

to :

( ̃ ) ( ̃)

Combining (1) and (2) together, we have

PROPOSITION 1. The Galilean Transformations

Let and ̃ be inertial observers moving with relative velocity , and let ( )

and ( ̃ ̃ ) be their inertial coordinates respectively. If and ̃ agree on the time

interval between any two events and on the distances between them

simultaneous events, then

̃ and

where and are constants.

Figure 3. Galilean Transformation


Module 5. Special Relativity 201

Figure 3 compares the two coordinates system. The vertical lines are

constant lines (respectively, ̃ constant lines)) and the horizontal lines are

constant (respectively ̃ constant)) lines.

Galilean Spacetimes

Definition 5.1.Galilean Spacetime. A Galilean Spacetime


is a spacetime in which the coordinates of inertial observers
are related by Galilean Transformations. Galilean observers
are inertial observers in Galilean Spacetime

Galilean observers agree on the elapsed time between events and on the

distance between simultaneous events, and they have a particularly simple

formula for the addition of velocities.

PROPOSITION 2. Addition of Velocities in Galilean Spacetimes.

Let and ̃ be Galilean observers and an object in a Galilean

spacetime. If ̃ is travelling with velocity relative to ,and is travelling in the

same direction with velocity relative to ̃ , then is travelling with the velocity

relative to .

In other words, if a child is running forward with a speed of 10 miles per

hour inside a train which is traveling 50 miles per hour, then the child’s total

speed relative to the ground is 60 miles per hour.

Proof. Since is travelling with constant velocity relative to ̃ , worldline

satisfies an equation of the form

̃ ̃ ̃
Module 5. Special Relativity 202

in ̃ coordinates, where ̃ is a constant. Substitute the Galilean

Transformations into this equation and get

( ) ̃

in coordinates. Solve for to find the position of at time according to :

( ) ( ̃ )

( )

Hence, velocity is according to .

DISCUSSION BOARD
We will now try to interact with each other in a two way

process at a least possible way! I will post a question/s and try to answer it on

your own.

1. How do we describe Galilean Transformation?

2. Can you describe the Galilean Spacetime?

3. How does vector addition relates to Galilean Transformation?

POST COMPETENCY CHECKLISTS


Let us try to determine if you learned something from our topic. Answer the
following Self-Assessment Question.

SELF – ASSESSMENT QUESTION 5.2

1. Compare Galilean Transformation with Transformation in Euclidean plane.


Module 5. Special Relativity 203

2. If you will be given the chance to teach this topic, how are you going to do

it?

EXPLORE!!!!

We are done exploring Galilean Transformations and other

related concepts. In this section, we will try to find out why Galilean

Transformation had failed.

FAILURE OF GALILEAN TRANSFORMATIONS

It ended up that we do not live in a Galilean spacetime. Observers in our

universe do not agree on the time interval between events, they do not agree on

distances between events that are simultaneous according to some observer,

and velocities do not add in the simple way described in Proposition 2. It took a

long time for scientists to realize this because the assumption that our universe is

Galilean produces only negligible errors at low relative speeds. But at speeds

close to the speed of light the errors are too large to ignore.

By the end of the nineteenth century scientists had measured the speed of

light to great accuracy under many different conditions. Experiments show that all

observers get the same result, about 186,282 miles per second, when they

measure the speed of light in a vacuum regardless of the velocity of the observer

or the velocity of the light source.

Suppose a jet fighter is hurtling along at a speed of one mile per second.

The pilot measures the speed of light travelling forward in his cockpit to be
Module 5. Special Relativity 204

186,282 miles per second. According to Galilean velocity addition, a person

standing on the ground should see the same light flashing by at the speed of

186,282 +1 = 186,283 miles per second. But he does not! In fact, he gets exactly

the same figure as the pilot on the plane. The velocity addition formula simply

does not work, at least when it is applied to light. Since the velocity addition

formula is derived from the assumption that different observers could agree on

distances between simultaneous events and on time, this assumption must also

be false.

DISCUSSION BOARD
We will now try to interact with each other in a two way

process at a least possible way! I will post a question/s and try to answer it on

your own.

1. How does Galilean Transformation failed?

2. What can you say about the assumptions made against Galilean

Transformation?

POST COMPETENCY CHECKLISTS


Let us try to determine if you learned something from our topic. Answer the
following Self-Assessment Question.

SELF – ASSESSMENT QUESTION 5.3


In not less than 100 words, discuss your reaction on the failure of Galilean

Transformation.
Module 5. Special Relativity 205

EXPLORE!!!!

So far, we are done exploring the reason why Galilean

Transformation failed. We will try to explore another concepts related to Special

Relativity – the Lorentz Transformations.

LORENTZ TRANSFORMATIONS

We will try to study a strange world of relativistic physics by replacing the

Galilean transformations with transformations that are consistent with the fact

that the speed of light is the same for all inertial observers.

Simultaneity and the Relativity of Time.

As illustrated by Jennings, imagine a pair of flashing lights mounted at

points and a certain distance, say one mile, apart. Place a detector at a point

halfway between the two lights and measure whether or not the light from

arrives at the detector at the same time as the light from . Both signals travel

the same distance at the same velocity, so both take the same amount of time to

reach the detector. Therefore, we know that flashed first if the signal from

arrives at the detector before the signal from , flashed first if the signal from

arrives at the detector later than the signal from , or and flshed

simultaneously if the two signals arrive together.

Figure 4. Simultaneous events.


Module 5. Special Relativity 206

Definition 5.2.Simultaneity of Distant Events. Let 𝑂 be an


observer with inertial coordinate system (𝑡 𝑥 ). two events
occur simultaneously according to 𝑂 if and only if two light
signals, one originating at each event, would arrive together
at the point exactly halfway between the events.

The exact location of the point halfway between the events is determined by ,

using his own ruler.

Comments on the definition.

1. The events in the definition are not actually required to send out any light

signals, but whatever means are used to assign times to the events must

give the same results as would have been obtained if light signals had

been sent.

2. The measurement of simultaneity depends on the observer. It is entirely

possible that another observer, carrying out the same measurement, will

get different results. Time, as well as position, may be different for

different observers.

Let and ̃ be inertial observers with inertial coordinate systems ( )

and ( ̃ ̃ ), respectively, moving apart with relative velocity . We seek functions

and which are consistent with the principle of relativity, the definition of

simultaneity, and the fact that the speed of light is constant, such that

̃ ( ) and ̃ ( )
Module 5. Special Relativity 207

Assume that and have the same form that is similar to Galilean

transformations, that is, that there are constants such that

( ) and

( ) (3)

Besides the fact that this assumption simplifies our task considerably,

there are other reasons for asserting that it should be so. It is equivalent to the

statement that the partial derivatives ̃ ̃ ̃ and ̃ are all

constant. If the partials were not constant then we might expect to find a point

( ) where the partial derivatives were especially nice, say where ̃ has a

minimum or something of that sort. Such a point would be a specially marked

point in the spacetime, and we could base a special coordinate system upon it.

But the principle of relativity says that there are no specially marked points or

other natural features in the universe on which to base a special “natural” system

of coordinates, so we conclude that such point does not exist. Hence the partial

derivatives should be constant.

In any case let us agree to accept equation (3). Then

( ) and

( ) (4)

Our job is to find the constants .

To simplify our calculations let us suppose that and ̃ have some

common event in their lives, that is, that their worldliness intersects at some

event . They might as well agree to synchronize their clocks at this event, so we

may assume that both observers assign to the time


Module 5. Special Relativity 208

( ) ̃( )

Figure 5.

Also, because lies on both of their worldlines, they both assign it the position

( ) ̃( )

Thus, and ̃ coordinate systems have a common origin at the event .

Substituting the coordinates ( ) ( ) ( ̃ ̃ ) of into equation (3), we

deduce that

(5)

Let us assume that and ̃ agree to orient their – axis so that if a light

beam is travelling in the positive direction according to , then it is also

travelling in the positive ̃ direction according to ̃ .

Figure 5 is a picture of spacetime in coordinates. The axis in

worldline and the line is ̃ worldline. Since ̃ on ̃ worldline, we

have

̃ and (6)

Plug (5) and (6) into the second of equation (4), to get
Module 5. Special Relativity 209

(7)

Hence, . Substitute this and (5) back in (4) to obtain the system

̃ (8)

The next step is to locate the ̃ axis. (The ̃ axis is ̃ worldline). The ̃ axis

contains all the events with ̃ coordinate equal to zero, so to find it we must locate

another event (other than the origin ) that has ̃ coordinate equal to zero. Let

( ), in ̃ coordinates, be such an event. According to ̃, is

simultaneous with since they both occur at time ̃ , so light signals sent by

and will meet at a point halfway between them. lies in the worldline ̃ ,

lies in the worldline ̃ , so the point halfway in between lies on the worldline

̃ . Thus to find all we need to do is follow a light beam from the origin

until it crosses the line ̃ , then follow another light beam back from there

until it intersects the line ̃ .

Rewrite the equations for these lines in coordinates:

̃ becomes

̃ becomes

̃ becomes (9)

By (8), let

A light signal travelling from the origin towards ̃ follows the line ,

and meets the halfway point ̃ at the event


Module 5. Special Relativity 210

( ) ( )
( ) ( )

(To see this, solve the equations simultaneously with the second in (9))

A light beam travelling in the opposite direction through this event moves

along a line with a slope ; satisfies the equation

( )
( ) ( )

and meets the line ̃ at the event

( ) ( )
( ) ( )

by third of (9).

This event is the event . Since the line connecting to the origin is the ̃

axis, it follows that the ̃ axis satisfies the following

̃ : (10)

Returning to the calculation of constants in (4) we have

̃ if and only if

̃ if and only if

by formulas (8) and (10). Hence , so , and (8) becomes

̃ (11)

Now,

̃ ̃ if and only if (12)

because the speed of light equals for both observers. Substituting in (11), (12)

becomes
Module 5. Special Relativity 211

if and only if (13)

Solve the left hand side for :

Comparing the left and right sides of (13), we have

Multiply through by to clear the fractions, then simplify to get:

( ) ( )

since no physical observer can travel at the speed of light. Hence

and (11) becomes

̃ (14)

It remains to calculate .

Imagine an experiment where each observer holds up a ruler that is one

foot long for the other observer to measure as he passes by. Both of the

observers perform the same experiment: measure the length of a one-foot long

ruler that is moving with velocity . By the principle of relativity both observers

should get the same result.

To measure the length of the moving ruler each observer records the

positions of its endpoints at particular time, and then finds the difference between

them. Since ruler is moving it is important that he record the location of the
Module 5. Special Relativity 212

endpoints at one particular time, for if he recorded where the endpoints were at

different times the difference in the positions would reflect how far the ruler had

moved in the interim in addition to its length.

Suppose that measures the length of ̃ ruler at time . The

endpoints of ̃ ruler at ̃ and ̃ . In ̃ coordinates these equations

become

and

(by 14). Substituting in we find that the endpoints of ̃ ruler are located at

and

when . Thus according to

the length of ̃ ruler is (15)

Next we calculate the length of ruler when it is measured by ̃ at time ̃ .

The endpoints of ruler are at and . First substitute ̃ into the

first equation of (14) and get ( ) , so

Then plug this result back into the second of (14) and get

̃ ( )

Thus when we have ̃ and when we have ̃ ( ), so

the length of ruler is ( ) (16)

Since both observers performed the same experiment the principle of relativity

says that their results (15 and 16) must be the same. Hence
Module 5. Special Relativity 213

( )

It follows that

(17)

Plugging (17) into (14) we arrive at our final result:

THEOREM 1. Lorentz Transformations.

Let and ̃ be a pair of inertial observers travelling with relative velocity . Let

( ) and ( ̃ ̃ ) be their respective inertial coordinate systems. Then there are

constants and such that

̃

̃

The constants and in the statement of Theorem 1 take care of the possibility

that and ̃ might not have the same origin, and takes care of the fact that

they might not orient their axes the same way .Figure 6 shows how and ̃

compare when they are related by a Lorentz transformation.

Figure 6. Lorentz transformations


Module 5. Special Relativity 214

DISCUSSION BOARD
We will now try to interact with each other in a two way

process at a least possible way! I will post a question/s and try to answer it on

your own.

1. How do we describe Lorentz Transformations?

2. How do we compare Lorentz Transformations and Galilean

Transformation?

POST COMPETENCY CHECKLISTS


Let us try to determine if you learned something from our topic. Answer the
following Self-Assessment Question.

SELF – ASSESSMENT QUESTION 5.4

In not less than 100 words, discuss your own understanding about Lorentz

Transformations

EXPLORE!!!!

We are done exploring Lorentz Transformations and all related concepts. For the

meantime, we will study Relativistic Addition of Velocities


Module 5. Special Relativity 215

RELATIVISTIC ADDITION OF VELOCITIES

Definition 5.3.Minkowski Spacetime. A Minkowski


spacetime is a spacetime in which coordinates of different
inertial observers are related by Lorentz transformations. A
Minkowski observer is an inertial observer in a Minkowski
spacetime.

PROPOSITON 3. Addition of Velocities in a Minkowski Spacetime.

Let and be objects in a Minkowski spacetime. If is travelling with

velocity relative to and is travelling with velocity relative to then is

travelling with velocity

relative to .

Proof. We regard as Minkowski observers. Let ( ) and ( ̃ ̃ ) be the

inertial coordinates of and respectively. Since is travelling with velocity

relative to , worldline satisfies an equation of the form

̃ ̃ ̃

in coordinates, where ̃ is a constant. Plug in Lorentz transformations and

this becomes

√ √
( )

Simplifying, we get an equation for the worldline of in coordinates:


Module 5. Special Relativity 216

( ) ( )

DISCUSSION BOARD
We will now try to interact with each other in a two way

process at a least possible way! I will post a question/s and try to answer it on

your own.

1. How do we describe Relativistic Addition of Velocities in Minkowski

Spacetime?

2. What tips/advice could you give to your fellow students who were taking

the same course to make this topic easier to understand?

POST COMPETENCY CHECKLISTS


Let us try to determine if you learned something from our topic. Answer the
following Self-Assessment Question.

SELF – ASSESSMENT QUESTION 5.5

1. A train is moving at 60 mph. A child on the train runs forward at a speed of

10 mph relative to the train. How fast is she moving relative to the ground?

2. Rework #1 assuming that the train is moving forward at 50% of the speed

of light and the child is running forward at 90% of the speed of light

relative to the train.


Module 5. Special Relativity 217

EXPLORE!!!!

We are done studying Relativistic Addition of Velocities in

Minkowski Spacetime. I hope that you are not overwhelmed by theorems,

propositions and proofs indicated in this module! Meanwhile, we will continue our

study on another topic –Lorentz – FitzGerald Contractions

LORENTZ – FITZGERALD CONTRACTIONS

In this section we will try to analyse the contractions of objects as

proposed by Irish physicists George FitzGerald (1851 – 1901).

Moving Clocks Runs Slow

One of the strange effects predicted by special relativity is that if two

observers are in motion relative to one another then each perceives the other’s

clock to be moving slower that his own.

Example 5.1. The Twin Paradox

On their twenty first birthday Paula leaves her twin brother Peter and

embarks on a trip at the terrific speed of ( ) . After travelling for seven years

she turns back and returns to Peter at the same speed. Paula is ( )

years old when she rejoins her twin brother. How old is Peter?

Solution. We shall assume that Peter and Paula are inertial observers

throughout the first leg of Paula’s trip. Let ( ) be Peter’s coordinate system and

let ( ̃ ̃ ) be Paula’s. Let ( ) ( ) ( ̃ ̃ ) at the event of Paula’s departure,

so that in the Lorentz transformations.

The first leg of Paula’s trip ends at ( ̃ ̃ ) ( ). Plug ( ̃ ̃ ) ( ) and

( ) into Lorentz transformations:


Module 5. Special Relativity 218

√ ( )

√ ( )

So,

( ) ( )

at the end of the leg of Paula’s first trip. Therefore the first leg of Paula’s trip

takes twenty five (of Peter’s) years. The second leg takes equal amount of time

so Peter is ( ) years old when Paula returns.

Figure 7. Twin Paradox

Although it seems strange that twin could have different ages, there is nothing

contradictory about the “twin paradox”. Experiment shows that the world really

works this way. One might ask, why can’t the same argument be used to prove

that Paula is older than Peter, since she sees him ravelling away from her? The

answer is that the relation between Peter and Paula is not really symmetrical:

Paula experiences a terrific acceleration when she turns around and heads back,

but Peter does not.


Module 5. Special Relativity 219

Moving Rulers are Short.

Another effect predicted by special relativity is the contraction of space in

the direction of motion. It was shown that if a ruler feet long is moving with

velocity relative an observer, then the observer will measure its length to be

feet long.

Example 5.2. The Einstein Train. A train whose length is 200 m. according to

an observer on the train passes through a station whose length is 100 m.

according to an observer in the station. The speed of the train is (√ ) . To a


man in the station the length of the train is ( ) (√ ( ) ) . So

the train has the same length as the station and the two fit together perfectly. But


to a man in the train the station is ( ) (√ ( ) ) long. This is only

one quarter of the length of the train, so the station is too short to hold the train.

DISCUSSION BOARD
We will now try to interact with each other in a two way

process at a least possible way! I will post a question/s and try to answer it on

your own.

1. How do we describe Lorentz-FitzGerald Contractions?


Module 5. Special Relativity 220

2. Can you cite at least 3 applications of Lorentz-FitzGerald Contractions in

our daily living?

3. What tips/advice could you give to your fellow students who were taking

the same course to make this topic easier to understand?

POST COMPETENCY CHECKLISTS


Let us try to determine if you learned something from our topic. Answer the
following Self-Assessment Question.

SELF – ASSESSMENT QUESTION 5.6

Assume that it took Paula ( in Example 5.1) a negligible amount of time to

reverse her direction of travel and start back.

a. An instant before Paula turned back, how old was she according to Peter

and how old was he according to her?

b. An instant after Paula turned back, how old was she according to Peter

and how old was he according to her?

c. According to Paula how much did Peter age while Paula was turning

around? How much did Paula age according to Peter?

EXPLORE!!!!

We had studied Lorentz-FitzGerald Contractions in the

previous section. And, for this section, we will explore Monkowski Geometry. As

a piece of advice, it is much better to scan your notes in Linear Algebra for it

would help you learn this topic easier.


Module 5. Special Relativity 221

MINKOWSKI GEOMETRY

We will start with the short review on dot product of vectors.

The Dot Product

The dot product of two vectors ⃗⃗⃗⃗ ( ) and ⃗⃗⃗⃗ ( ) in is

defined by the formula

⃗⃗⃗⃗ ⃗⃗⃗⃗ (18)

The dot products compute lengths and angles:

|⃗⃗⃗⃗ | √⃗⃗⃗⃗ ⃗⃗⃗⃗ (19)

⃗⃗⃗⃗ ⃗⃗⃗⃗
(⃗⃗⃗⃗ ⃗⃗⃗⃗ ) ( )
|⃗⃗⃗⃗ ||⃗⃗⃗⃗ |

and it is preserved by Euclidean isometeries. In fact it is not hard to prove that a

function is a Euclidean isometry if and only if ⃗⃗⃗⃗⃗⃗⃗⃗⃗⃗⃗⃗⃗⃗⃗⃗⃗⃗⃗⃗


( ) ( ) ⃗⃗⃗⃗⃗⃗⃗⃗⃗⃗⃗⃗⃗⃗⃗⃗⃗⃗⃗⃗
( ) ( )

⃗⃗⃗⃗⃗ ⃗⃗⃗⃗⃗ for all points .

The dot product is so fundamental for the Euclidean geometry of that it

is often said that Euclidean geometry is simply the geometry of the dot product.

Minkowski Product

Minkowski Geometry is the geometry of the Minkowski product.

Definition 5.4.Minkowski Product. The minkowski product


⃗⃗⃗⃗
𝑥 ∗ ⃗⃗⃗⃗
𝑦 of two vectors ⃗⃗⃗⃗
𝑥 (𝑠 𝑥 𝑥 𝑥𝑛 ) and ⃗⃗⃗⃗
𝑦
(𝑠 𝑥 𝑥 𝑛+
𝑥𝑛 ) in 𝑅 is defined by
⃗⃗⃗⃗
𝑥 ∗ ⃗⃗⃗⃗
𝑦 𝑐 𝑠𝑡 𝑥 𝑦 𝑥 𝑦 𝑥𝑛 𝑦𝑛
where 𝑐 is a constant.
Module 5. Special Relativity 222

+ +
A dimensional Minkowski space is together with the minkowski

product. Physically and are regarded as inertial coordinates. From

now on we will concentrate on .

Definition 5.5. Generalized Lorentz Transformation. A


Generalized Lorentz Transformation on 𝑀 is a function of
the form

𝑣
𝑡 𝑥 𝑥 𝑣𝑡
𝑓 (𝑡 𝑥 ) 𝛼 𝑐 𝑡 𝛽 𝑥
𝑣 √ 𝑣 𝑐

( 𝑐 )
where 𝑡 𝑥 , and 𝑐 < 𝑣 < 𝑐 are constants and
𝛼 𝛽

PROPOSITION 4. Generalized Lorentz Transformations are Minkowski

Isometries. If are any events in and is a generalized Lorentz

transformation then,

⃗⃗⃗⃗⃗⃗⃗⃗⃗⃗⃗⃗⃗⃗⃗⃗⃗⃗⃗⃗
( ) ( ) ∗ ⃗⃗⃗⃗⃗⃗⃗⃗⃗⃗⃗⃗⃗⃗⃗⃗⃗⃗⃗⃗
( ) ( ) ⃗⃗⃗⃗⃗ ∗ ⃗⃗⃗⃗⃗

Proof. Set

⃗⃗⃗⃗⃗ ( ) ⃗⃗⃗⃗⃗ ( )

⃗⃗⃗⃗⃗⃗⃗⃗⃗⃗⃗⃗⃗⃗⃗⃗⃗⃗⃗⃗
( ) ( ) ( ̃ ̃) ⃗⃗⃗⃗⃗⃗⃗⃗⃗⃗⃗⃗⃗⃗⃗⃗⃗⃗⃗⃗
( ) ( ) (̃ ̃)

If ( ) and ( ) then ⃗⃗⃗⃗⃗ so

( ) ( )

If is a generalized Lorentz transformation


Module 5. Special Relativity 223

⃗⃗⃗⃗⃗⃗⃗⃗⃗⃗⃗⃗⃗⃗⃗⃗⃗⃗⃗⃗
( ) ( ) ( ) ( ) (20)

⃗⃗⃗⃗⃗⃗⃗⃗⃗⃗⃗⃗⃗⃗⃗⃗⃗⃗⃗⃗
( ) ( )
√ √
( )

for some constants and . (The constants and cancel out when

you subtract ( ) from ( ). Similarly

⃗⃗⃗⃗⃗⃗⃗⃗⃗⃗⃗⃗⃗⃗⃗⃗⃗⃗⃗⃗
( ) ( ) ( ) (21)
√ √

Plugging in (20) and (21) and multiplying out, we have

̃ ̃ ̃̃ (22)

which proves the Proposition.

COROLLARY 1. The minkowski product has the same form in any inertial

coordinate system.

Proof. This is equation (22)

Whereas there is only one type of nonzero vector in Euclidean geometry,

Minkowski geometry has three:

Definition 5.6. A nonzero vector ⃗⃗⃗⃗


𝐴 𝑀

i. spacelike if ⃗⃗⃗⃗𝐴 ∗ ⃗⃗⃗⃗


𝐴 >
ii. lightlike if ⃗⃗⃗⃗
𝐴 ∗ ⃗⃗⃗⃗
𝐴
iii. timelike if ⃗⃗⃗⃗
𝐴 ∗ ⃗⃗⃗⃗
𝐴 <

Every timelike vector has a time orientation which determines whether it

points toward the future or toward the past.


Module 5. Special Relativity 224

Definition 5.7. Time Orientation. A timelike vector (𝑡 𝑥 )


is
i. future pointing if 𝑡 >
ii. past pointing if 𝑡 <

For example the vector ( ) is spacelike, ( ) is lightlike, and ( ) is

timelike and future pointing. More generally, a vector is timelike if it is parallel to

the time axis of some observer, spacelike if it is parallel to space axis of some

observer and lightlike if it is parallel to the worldline of a light ray.

Figure 8.Time Orientation

Definition 5.8. The worldline of a material particle in


Minkowski space is a parametrized curve whose velocity
vectors all are timelike and future pointing.

Material particles have timelike velocity vectors ⃗⃗⃗⃗ ( ) since their

speed | | is less than | |.


Module 5. Special Relativity 225

Definition 5.9. Lenght. The length of a vector ⃗⃗⃗⃗


𝐴 in

Minkowski space is |⃗⃗⃗⃗


𝐴| √|⃗⃗⃗⃗
𝐴 ∗ ⃗⃗⃗⃗
𝐴|

(This is a funny kind of length since lightlike vectors have zero length even

though they are not zero)

The length of a spacelike vector measures distance, but the length of a

timelike vector measures time.

PROPOSITION 5. Let .

a. If ⃗⃗⃗⃗⃗⃗ is parallel to the time axis of some observer then ( )|⃗⃗⃗⃗⃗⃗ | is the

time between the two events for that observer.

b. If ⃗⃗⃗⃗⃗⃗ is parallel to the space axis of some observer then |⃗⃗⃗⃗⃗⃗ | is the

distance between two events for that observer.

Proof. If ⃗⃗⃗⃗⃗⃗ is parallel to the ̃ axis then ⃗⃗⃗⃗⃗⃗ ( ) for some ̃ , and its length

is √| ̃ | | ̃ |. If is parallel to the ̃ axis then ⃗⃗⃗⃗⃗⃗ ( ̃ ) for some ̃

and its length is √| ̃ | | ̃ |.

Definition 5.10. ⃗⃗⃗⃗


𝐴 is perpendicular to ⃗⃗⃗⃗
𝐵 in 𝑀 if ⃗⃗⃗⃗
𝐴 ∗
⃗⃗⃗⃗
𝐵
Module 5. Special Relativity 226

Figure 9. Minkowski - perpendicular

DISCUSSION BOARD
We will now try to interact with each other in a two way

process at a least possible way! I will post a question/s and try to answer it on

your own.

1. How do we describe Minkowski Geometry?

2. What tips/advice could you give to your fellow students who were taking

the same course to make this topic easier to understand?

POST COMPETENCY CHECKLISTS


Let us try to determine if you learned something from our topic. Answer the
following Self-Assessment Question.

SELF – ASSESSMENT QUESTION 5.7

1. Show that if ⃗⃗⃗⃗ ⃗⃗⃗⃗ ⃗⃗⃗⃗ are timelike future pointing vectors then

⃗⃗⃗⃗ ⃗⃗⃗⃗ ⃗⃗⃗⃗ also is a timelike future pointing vector.


Module 5. Special Relativity 227

2. Prove that if ⃗⃗⃗⃗ is a timelike future pointing vector then the ̃ coordinate of is

positive in any inertial coordinate system. (Hint: Use Corollary 1)

3. Minkowski Pythagorean Theorem. Let ⃗⃗⃗⃗ ⃗⃗⃗⃗ ⃗⃗⃗⃗ be the sides of a triangle in

Minkowski spacetime. Suppose ⃗⃗⃗⃗ is timelike, ⃗⃗⃗⃗ is spacelike and ⃗⃗⃗⃗ is

perpendicular to ⃗⃗⃗⃗ . Show that

| | | | | |

Figure 10. Minkowski Pythagorean Theorem

EXPLORE!!!!

We had explored Minkowski Geometry in the previous

section. In this section, we will study the idea behind why The Slowest Path is a

Line.

THE SLOWEST PATH IS A LINE

We will start our discussion with the Length of a Worldline.

The Length of a Worldline


Module 5. Special Relativity 228

Let ( ) ( ( ) ( )) , parametrize the worldline of a (not

necessarily inertial) observer. As ranges over an infinitesimally small interval

, the event ( ) is displaced by an amount

( ) ( )

Over a small enough interval we can treat the observer as an inertial observer

since his velocity changes by a negligible amount. The amount of time that

elapses on the observer’s clock over the interval is

| |

| ( )|

with equality in the limit as . Hence the total time that elapses on the

observer’s clock between the event ( ) where and the event ( ) where

is

∫ | ( )| (23)

( )

Time Maximization

The worldline of an inertial observer is a straight line in Minkowski space.

In Euclidean Geometry straight line minimize length, but in Minkowski geometry

straight lines pointing in a timelike direction maximize time.

PROPOSITION 6. The Generalized Twin Paradox. An inertial observer takes

longer to get from one event to another than any other observer.
Module 5. Special Relativity 229

Proof. Let and ̃ be observers whose worldlines contain the events and .

Assume that is an inertial observer and ̃ is not. In coordinates let

( ) and ( )

with < . In ̃ coordinates, let

(̃ ) and (̃ ).

We must prove that

| |>| ̃ ̃ |.

To compute |̃ ̃ |, let ( ) ( ( ) ( )), , be a

parametrization of ̃ worldline between and . Then

( ) and ( )

and, since the velocity vector ( ( ) ( )) of the worldline of a material particle is

a future pointing vector, we also have

( )> for all .

By the length formula

|̃ ̃ | ∫ | ( )|

|̃ ̃ | ∫ | ( ) ( )|

Because ̃ cannot travel faster than light,

< < ,

So,

( ) >( )

Hence,
Module 5. Special Relativity 230

| ( ) ( )| √| ( ( )) ( ( )) |

| ( ) ( )| √ ( ( )) ( ( ))

| ( ) ( )| | ( )|

| ( ) ( )| ( )

for every . It follows that

|̃ ̃ | ∫ | ( ) ( )|

|̃ ̃ | ∫ ( )

|̃ ̃ | ( ) ( )

|̃ ̃ |

with equality on the second line only if ( ) for all . Thus takes longer to

travel from to than ̃ does. This completes the proof.

DISCUSSION BOARD
We will now try to interact with each other in a two way

process at a least possible way! I will post a question/s and try to answer it on

your own.

1. Why do we consider that the Slowest Path is a Line?

2. What tips/advice could you give to your fellow students who were taking

the same course to make this topic easier to understand?


Module 5. Special Relativity 231

POST COMPETENCY CHECKLISTS


Let us try to determine if you learned something from our topic. Answer the
following Self-Assessment Question.

SELF – ASSESSMENT QUESTION 5.8

1. Let and be any two events such that ⃗⃗⃗⃗⃗⃗ is a timelike, future pointing

vector. Show that, given any positive number , there is an observer travelling

slower than the speed of light (although not necessarily at constant speed)

who gets from to in less than units of time, as measured on the

observers own clock.

Thus, travelling slower than the speed of light, you can get from here to Alpha

Centauri, 4.3 light years away, in less time than it takes to read this sentence

(if you are willing to endure some pretty wild accelerations).

EXPLORE!!!!

We had explored the idea behind why The Slowest Path is a

Line in the previous section. In this section, we will study the last topic in Special

Relativity the Hyperbolic Angles and the Velocity Addition Formula. I hope and

pray that after this last topic, you had gained at least the minimum competency

that was expected for you to have. If not, don’t worry, there is always a room for

improvement!
Module 5. Special Relativity 232

HYPERBOLIC ANGLES AND THE VELOCITY ADDITION FORMULA

In this section, I will directly present the definition mentioned by Jennings

relative to this topic.

Definition 5.11. A pseudocircle in Minkowski space is the


set of vectors ⃗⃗⃗⃗
𝐴 satisfying an equation of the form

|⃗⃗⃗⃗
𝐴| 𝑟

where is a constant.

If the pseudocircle consists of a pair of hyperbolas

The branch of the hyperbola where > is parametrized by

A radial vector ( ) is a timelike and future pointing if it extends out to a point on

this branch of hyperbola. The hyperbolic tangent measures the velocity of a

particle whose worldline is parallel to the vector ( )

(24)

Definition 5.12. Let ⃗⃗⃗⃗


𝐴 and ⃗⃗⃗⃗
𝐵 be timelike, future pointing
vectors. The hyperbolic angle 𝜑 between ⃗⃗⃗⃗
𝐴 and ⃗⃗⃗⃗
𝐵 is
defined up to a sign by the formula
⃗⃗⃗⃗
𝐴 ∗ ⃗⃗⃗⃗
𝐵
𝑐𝑜𝑠 𝜑
𝑐 |⃗⃗⃗⃗
𝐴 ||⃗⃗⃗⃗
𝐵|
Module 5. Special Relativity 233

Figure 11. Pseudocircles

Lorentz transformations preserve hyperbolic angles because they

preserve lengths and Minkowski products that are used to define them. If ⃗⃗⃗⃗ and

⃗⃗⃗⃗ are each tangent to the worldline of an observer at some event then (24)

says that is times their relative velocity, because one can always

regard the coordinates in (24) as being the inertial coordinates of one of the

observers. Given objects and let

the hyperbolic angle between and worldline

the hyperbolic angle between and worldline

the hyperbolic angle between and worldline

The Lorentz velocity addition formula says that

( )

where the second equation comes from the addition formula for hyperbolic

tangents. Thus velocity addition formula boils down to the statement that
Module 5. Special Relativity 234

In this way we recover something very much like the simplicity of Galilean

velocity addition formula.

DISCUSSION BOARD
We will now try to interact with each other in a two way

process at a least possible way! I will post a question/s and try to answer it on

your own.

1. How do we describe Hyperbolic Angles? Velocity Addition Formula?

2. What tips/advice could you give to your fellow students who were taking

the same course to make this topic easier to understand?

POST COMPETENCY CHECKLISTS


Let us try to determine if you learned something from our topic. Answer the
following Self-Assessment Question.

SELF – ASSESSMENT QUESTION 5.9

Prove the following addition formulas for hyperbolic functions

1. ( )

2. ( )

+
3. ( )
+
Module 5. Special Relativity 235

ASSIGNMENT

1. Surf the net and search about short biography of Galileo. Identify also his

other contributions in mathematics.

2. Surf the net and search about short biography of Albert Einstein. Identify

also his other contributions in mathematics.

3. Surf the net and search about short biography of Hendrik Antoon Lorentz.

Identify also his other contributions in mathematics.

4. Surf the net and search about short biography of Hermann Minkowski.

Identify also his other contributions in mathematics.

5. The Minkowski “Triangle Inequality”. Let ⃗⃗⃗⃗ and ⃗⃗⃗⃗ be future pointing

timelike vectors. Prove that

|⃗⃗⃗⃗ ⃗⃗⃗⃗ | |⃗⃗⃗⃗ | |⃗⃗⃗⃗ |

with equality if and only if ⃗⃗⃗⃗ and ⃗⃗⃗⃗ are parallel.

Figure 12. Minkowski Triangle Inequality

6. What would be different about in #5 if the triangle were in Euclidean

space?

7. What does the triangle inequality have to do with the twin paradox?

8. Search from a book (or via internet) all the formulas for hyperbolic

functions.

9. In not less than 100 words, write what you learned in Special Relativity.
Module 5. Special Relativity 236

10. And as final assignment, in not more than 500 words, write a reflection on

what you learned in Modern Geometry. (You may also include your

suggestions for further improvement)


Modern Geometry 237

References

Book:

George A. Jennings Modern Geometry With Applications (Universitext)

Electronic Resources:

http://www.youtube.com
Modern Geometry 238

About the Author

MARBEN ALVARADO OROGO is an instructor II at


Central Bicol State University of Agriculture –
Pasacao Campus.

He is currently taking up Doctor of Philosophy in


Education major in Mathematics (24 units) at
University of the Philippines Open University ,
Los Baños Laguna.

He took up his masters’ degree at Naga College Foundation Inc., Naga City
with the degree Master of Arts in Education major in Mathematics (2012-
2014) where he graduated with Academic Excellence and Excellence in Oral
Defense awards.

He finished his bachelor’s degree at Central Bicol State University of Agriculture


– Pasacao Campus, Sta. Rosa Del Norte, Pasacao Camarines Sur with the
degree Bachelor of Secondary Education major in Mathematics (2007-2011)
where he graduated with academic awards as Cum Laude and Best in Practice
Teaching as well as Leadership Award.

You might also like